The test that exonerates CO2

By Javier Vinós

This post has been translated into German by Christian Freuer here.

Most people don’t have a clear understanding of the greenhouse effect (GHE). It is not complicated to understand, but it is usually not well explained. It is often described as “heat-trapping,” but that is incorrect. Greenhouse gases (GHG) do not trap heat, even if more heat resides within the climate system due to their presence in the atmosphere. The truth is that after adjusting to a change in GHG levels, the planet still returns all the energy it receives from the Sun. Otherwise, it would continue warming indefinitely. So, there is no change in the energy returned. How do GHGs produce GHE?

GHGs cause the atmosphere to be more opaque to infrared radiation. As solar radiation heats mainly the ocean and land surface of the planet, GHGs absorb thermal emission from the surface at the lower troposphere and immediately pass that energy along to other molecules (typically N2 and O2) through collisions that occur much faster than the time it would take to re-emit the radiation. This warms the lower troposphere. The density and temperature decrease rapidly through the troposphere, so molecules are colder and more separated at the upper troposphere. Now GHGs have a chance to emit IR radiation so when they finally collide with another molecule, they are colder so GHGs have a cooling effect in the upper troposphere and stratosphere.

Because GHGs make the atmosphere more opaque to IR radiation, when they are present the emission to space from the planet normally does not take place from the surface (as happens in the Moon). Part of it still takes place from the surface through the atmospheric window, but most of it takes place from higher in the atmosphere. We can define a theoretical effective emission height as the average height at which the Earth’s outgoing longwave radiation (OLR) is being emitted. The temperature at which the Earth emits is the temperature at the effective emission height in the atmosphere. That temperature, when measured from space is 250 K (-23°C), not 255 which is the calculated temperature for a theoretical blackbody Earth. That temperature corresponds to a height of about 5 km, which we call the effective emission height.

The last piece we need to understand the GHE is the lapse rate, which in the troposphere is positive, meaning that temperature decreases with height. Without a positive lapse rate, the GHE does not work. Since GHGs cause the planet to emit from a higher altitude, due to making the atmosphere more opaque to IR radiation, that altitude is colder due to the lapse rate. The Earth still needs to return all the energy received from the Sun, but colder molecules emit less. So, the planet will go through a period when it will emit less than it should, warming the surface and the lower troposphere until the new height of emission achieves the temperature necessary to return all the energy, at which point the planet stops warming.

The GHE simply states that the temperature at the surface (Ts) is just the temperature of emission (Te) plus the lapse rate (Γ) times the height of emission (Ze).


Ts = Te + ΓZe

Held & Soden (2000) illustrated it in figure 1:



This is how the GHE actually works. An increase in CO2 means an increase in the height of emission. Since the temperature of emission must remain the same, the temperature from the surface to the new height of emission must increase. The increase is small but significant. As Held and Soden say:



“The increase in opacity due to a doubling of CO2 causes Ze to rise by ≈150 meters. This results in a reduction in the effective temperature of the emission across the tropopause by ≈(6.5K/km) (150 m) ≈1 K.”

Held and Soden

So, the temperature at the surface must increase by 1K. That’s the direct warming caused by the doubling of CO2, before the feedbacks (mainly water vapor) kick in, further raising the height of emission.

This also has an interesting prediction. If the warming is due to an increase in CO2 when the increase takes place and the altitude of emission increases, the planet should emit less OLR as the new altitude is colder and a reduced OLR is the warming mechanism. Once the warming takes place, the OLR will become the same as before the GHG increase. It says so in Held and Soden’s figure 1 caption: “Note that the effective emission temperature (Te) remains unchanged.” Same Te, same OLR. So, if CO2 is responsible for the surface temperature increase, we should first expect less OLR and then the same OLR. If at any time we detect more OLR that would indicate another cause for the warming. Anything that makes the surface warmer, except GHGs, will increase the temperature of emission, increasing OLR.

So, this is the test:

– Surface warming but less or same OLR: CO2 is guilty as charged

– Surface warming and more OLR: CO2 is innocent

And the test results can be evaluated for example with Derwitte and Clerbaux 2018:



“decadal changes of the Outgoing Longwave Radiation (OLR) as measured by the Clouds and Earth’s Radiant Energy System from 2000 to 2018, the Earth Radiation Budget Experiment from 1985 to 1998, and the High-resolution Infrared Radiation Sounder from 1985 to 2018 are analyzed. The OLR has been rising since 1985, and correlates well with the rising global temperature.

Derwitte and Clerbaux 2018

CO2 is innocent. Its fingerprint is not found at the crime scene. Something else is warming the planet and causing the increase in OLR.

Bibliography:

Dewitte, S. and Clerbaux, N., 2018. Decadal changes of earth’s outgoing longwave radiation. Remote Sensing, 10(10), p.1539.
https://www.mdpi.com/2072-4292/10/10/1539/pdf

Held, I.M. and Soden, B.J., 2000. Water vapor feedback and global warming. Annual review of energy and the environment, 25(1), pp.441-475.
https://www.annualreviews.org/doi/pdf/10.1146/annurev.energy.25.1.441

Stephens, G.L., O’Brien, D., Webster, P.J., Pilewski, P., Kato, S. and Li, J.L., 2015. The albedo of Earth. Reviews of geophysics, 53(1), pp.141-163.

5 40 votes
Article Rating
539 Comments
Oldest
Newest Most Voted
Inline Feedbacks
View all comments
Tom Halla
February 24, 2023 2:12 pm

I would consider that the effects of GHGs are on a log curve, so doubling from current levels has an effect that is difficult to distinguish from other causes. Ocean currents and circulation effects have such an effect on weather it is quite noisy.

Reply to  Tom Halla
February 24, 2023 5:06 pm

The problem is that the way climate change has been defined and framed, we don’t have other causes. We cannot even explain the LIA, which occurred just a few centuries ago. This should indicate that climate science is not going the right way.

Reply to  Javier Vinós
February 24, 2023 10:41 pm

LIA little ice age you mean it was cooler before so its warmer now?

but it cant be warmer since earth has no average temperature and we can measure it or calculate the uncertainties

Reply to  Steven Mosher
February 25, 2023 7:35 am

“LIA little ice age you mean it was cooler before so its warmer now?
but it cant be warmer since earth has no average temperature and we can measure it or calculate the uncertainties”

HUH?
Are you implying the Earth went straight from the “Big” Ice Age into the LIA because Man’s CO2 wasn’t there to make it warmer in-between?
Maybe, just maybe there’s more going on than even “The Settled Science” understands?

Reply to  Gunga Din
February 25, 2023 11:10 am

His logic twisting is a shot at those people that complain ‘there is no global average temperature … so quit trying to use it as an absolute metric.’

(controlled emotional outburst based on unimportant memory connection. Early onset…)

Dave Yaussy
Reply to  Steven Mosher
February 25, 2023 7:56 am

Did you mean we can’t measure it? It’s hard to follow your cryptic drive-by mind-dumps sometimes.

Richard Greene
Reply to  Steven Mosher
February 25, 2023 9:21 am

Calling all readers:
Does anyone have a Mr. Masher gibberish decoder ring for sale?

Reply to  Steven Mosher
February 25, 2023 12:56 pm

Holy cow. For an English major, your English comes as less understandable than mine. Is your keyboard working properly?

Reply to  Javier Vinós
February 28, 2023 5:56 am

I don’t think it’s his keyboard that’s the problem 🙂

Richard Greene
Reply to  Javier Vinós
February 25, 2023 1:08 am

The IPCC was set up to “prove” manmade greenhouse gas emissions were harmful.

In 1995 the IPCC arbitrarily claimed all natural causes of climate change were “noise”

That arbitrary claim made their climate propaganda job easier.

They used the 1979 Charney Report wild guessed ECS from 1988 until a few years ago. Then arbitrarily raised the seemingly harmless lower limit from +1,5 degrees C. to +2.5 degrees C.

That was easy too.

The IPCC is a political propaganda organization that does circular reasoning:

(1) We (IPCC) assume all climate change has been manmade and dangerous

(2) Therefore we (IPCC) predict future climate change will be manmade and dangerous.

What is real climate science?
Not this claptrap from the IPCC.

Coeur de Lion
Reply to  Richard Greene
February 25, 2023 1:15 am

Dead right. Read Donna Laframboise’s two books. The second deals mainly with the corrupt and sexually compromised Rajendra Pachauri and harm to science

Richard Greene
Reply to  Javier Vinós
February 25, 2023 9:19 am

We might explain the global warming from 1975 to 2015, which could have been caused by CO2, or by 100% natural causes or, more likely, by a combination of natural and manmade causes,

1975 to 2015 may only be 40 years out of the past 4.5 billion years, but science has to focus on something !

I personally think climate science should focus on the year 1975, when natural causes of climate change were dying, and nade CO2 boss. At least that’s wht the IPCC claims.

This post is serious, not satire.

Reply to  Richard Greene
February 26, 2023 11:11 am

Javier’s final chart is very interesting. Increasing CO2 should decrease not increase OLR until Planck feedback (higher surface temp) catches up. Easily shown with a few U Chicago Modtran runs.
Probably Willis has looked at this with one or more of his ERBE analyses….story tip…but I don’t recall one.

February 24, 2023 2:18 pm

Most people don’t have a clear understanding of the greenhouse effect (GHE).

Dead right. The fundamental error is the belief that such a process can alter Earth’s energy balance. It is simplistic trivia that fails to recognise the role of ice on land, on water and in the atmosphere.

The triple point of ice at 273K is fundamentally responsible for the temperature control processes that maintain Earth’s energy balance and hence climate in a narrow range despite huge disturbances over daily, annual and throughout history.

Water, its distribution and its phases are the key to understanding Earth’s energy balance.

Reply to  RickWill
February 24, 2023 2:22 pm

That being right doesn’t mean that a change in the greenhouse effect cannot alter Earth’s energy balance. It is obvious that it can and it should. You cannot make the atmosphere more opaque to IR and not alter the energy balance. We simply don’t know by how much.

Reply to  Javier Vinós
February 24, 2023 3:29 pm

 It is obvious that it can and it should. 

It appears obvious if you have no idea how the energy balance is actually achieved. Thinking it obvious demonstrates a lack of understanding of how the energy balance is controlled. The idea that a trace gas can alter the energy balance is obvious nonsense.

The energy balance is function of transformation of water to ice at the triple point of 273K. Open ocean surface temperature is limited to 30C because the persistence of ice formation high in the troposphere prevents the water surface exceeding that temperature due to the high reflectivity of the atmospheric ice and its persistence once the surface temperature hits 30C. Trace amounts of CO2 cannot alter that temperature regulating process.

Below 15C ocean surface, convective instability shuts down and the atmosphere can reach water saturation with the saturated air forming persistent condensing cloud that lowers heat loss by lowering the radiating temperature. Trace amounts of CO2 cannot alter that process. Much of the oceans in the Southern Hemisphere exhibit this condition once the sun zenith shifts to the NH. Same in the NH but to a limited extent due to the lower proportion of ocean area relative to land.

At -1.7C, sea ice forms and dramatically lowers the heat loss from ocean surface due to its insulating properties. The air above a thin layer of sea ice can be 50C cooler than the water below the ice.

Once ice becomes persistent on land, it tends to hang around because it is hard to melt due it is high reflectivity and requires a lot of sunlight to melt. However loss of surface ice results in rapidly rising temperature in that particular location.

Water in all its forms control the energy balance. The idea that CO2 can have any influence is naive nonsense.

Your evidence proves my point. Looking for a CO2 signature in temperature is a fruitless exercise. The single contribution of climate models to understanding is proof of that reality. Surface temperatures are not warming everywhere at an accelerating rate. In fact most warming is occurring on the Greenland plateau in January. Even more than the models predicted.

Reply to  RickWill
February 24, 2023 3:42 pm

The energy balance is function of transformation of water to ice at the triple point of 273K.

You are inventing things as you go. The energy balance at the top of the atmosphere cannot depend mainly on water changes of state. Nobody says that but you. You defend a different science from everybody else. It is up to you to find evidence supporting your views. Good luck in building your own climate science, quite a task.

Reply to  Javier Vinós
February 24, 2023 4:36 pm

The energy balance at the top of the atmosphere cannot depend mainly on water changes of state. 

Have you ever seen monsoonal cloud? What is it and how does it form? What are clouds made of? What causes convective instability that can create updrafts in excess of 300kph? Why can’t open ocean temperature exceed 30C?

All explained in detail here:
https://wattsupwiththat.com/2022/07/23/ocean-atmosphere-response-to-solar-emr-at-top-of-the-atmosphere/

I have shown the process in great detail. I have not been blinded by the “common wisdom” that is plainly wrong. You have just demonstrated that CO2 does nothing. Your error is simply thinking that there is some magical process where it can alter Earth’s energy balance in the face of overwhelming power of water and its phase changes. 1kg/m^2 of ice in the atmospheric column can absorb 95% of the incident OLR. Globally the average is 23kg/m^2. Over tropical warm pools the water mass often exceeds 80kg/m^2. About 1% of that as ice will reflect most of the incident sunlight and absorb most of the outgoing OLR; releasing it to solidify more water.

Nobody says that but you. 

I am in good company when I point out a reality as opposed to the common wisdom.

You have shown CO2 is not doing what it is supposed to do and yet you still believe that some magical GHE alters Earth’s energy balance. All you have done is demonstrated you were ignorant in thinking it can. It cannot. It is a fairy tale that has somehow become common wisdom.

Reply to  RickWill
February 24, 2023 5:03 pm

and yet you still believe that some magical GHE alters Earth’s energy balance.

You don’t know what I believe or what I don’t unless you read it in my book.

The rule is: You cannot change climate without altering the radiative incoming and outgoing energy fluxes.

That rule admits very few exceptions. Unless your proposed mechanism includes an explanation for how those fluxes are changed nobody will give it a second look, and there is a very good reason for it. Nothing happens in the Universe without the energy for it to happen.

Reply to  Javier Vinós
February 24, 2023 5:35 pm

The rule is: You cannot change climate without altering the radiative incoming and outgoing energy fluxes.

You clearly did not look at my link. Monsoonal cloud turns the ground below dark when the sun is at its zenith . Only a small proportion of the incident solar power gets thermalised. A good proportion of what gets thermalised does not make it to the ground. On average only 200W/m^2 makes it to the ground and all of that goes into evaporation. There is NO OLR released from the surface of a warm pool and the temperature is steady; minor fluctuations caused by cooling rain and the occasional clear sky sunlight. .

On average, 30% of all incoming available EMR is not thermalised. The reason is ice in the atmosphere.

So 30% is gone due to ICE IN THE ATMOSPHERE before the energy does anything below that cloud. And here you are suggesting ice in the atmosphere is not involved in the radiative balance.

You have been sucked into dismissing the 30% that ice contributes to unthermalised EMR as a fixed quantity. That 30% is highly responsive to surface temperature. Unthermalsed EMR gets in excess of 50% over ocean warm pools and the remainder does nothing but drive the heat engines that circulate air throughout the globe. There is no surface warming beyond 30C in open ocean water. You can triple the amount of CO2 in the atmosphere and warm pools will never be warmer than 30C. CO2 has no impact on the processs.

Richard Greene
Reply to  Javier Vinós
February 25, 2023 1:30 am

A desire to balme everything but CO2 for global warming leads to many unusual theories. This seems more common among “Climate Realists” than in the past 25 years.

I don’t understand why CO2 can’t be recognized as a weak greenhouse gas above 400ppm and recognized as impeding Earth’s ability to cool itself by a small amount. AGW does exist. But has been harmless, obviously. What percentage of the 1975 to 2015 warming did CO2 cause? No one knows. Lab experiments suggest some of thagt warming. So what? Warming is good news on our planet.

We do know the largest 8-year period of CO2 emissions, from 2015 to 2023. caused no net warming of our planet. So much for CO2 as the climate control knob. CO2 is just one of many climate change variables,

Even with rising CO2, and falling SO2 emissions, in the past eight years, both of which do cause global warming, the net effect of all climate change variables has been no change in the global average temperature (UAH data). If the Climate Howler Global Whiners could “revise” the 2015 to 2023 UAH numbers to show warming, they would have by now.

Reply to  Richard Greene
February 25, 2023 6:36 am

Why should anyone accept what they think is incorrect?

Richard Greene
Reply to  mkelly
February 25, 2023 9:24 am

So people will take them seriously about related subjects.

Reply to  Richard Greene
February 25, 2023 10:57 am

CO2 cannot do what you claim so why should I just accept it.

Erik Magnuson
Reply to  Javier Vinós
February 25, 2023 10:40 am

I think what Rick is trying to say is that water vapor plays a very important role in convective heat transfer from the surface to the upper atmosphere. What makes water vapor especially important with convective heat transfer is that it can change phase in the process. The change in effective top of atmosphere should lead to an increase in altitude where convective heat transfer is still effective.

One other indication of the importance of water vapor is the temperature record of the last million years. During a glacial period, there appears to be a robust positive feedback driving the temperatures lower until a combination of dust and orbital changes end the glacial period. During the inter-glacials, there appears to be a very robust temperature limiting mechanism which I would think is related to the 26ºC SST required to sustain tropical cyclones.

You are correct in asserting that an increase in CO2 will have some effect.

I very much liked your going into detail about how CO2 affects radiative heat transfer – remembering how my eyes were opened by Roy Spencer’s mention of a delay between a CO2 molecule absorbing an IR photon and re-emitting that photon. This led to thinking the CO2 molecule makes for a lousy transmission antenna, with radiation being a quantum tunneling process related to gamma ray emission. After some further thought, the light dawned on me about stimulated emission and why CO2 lasers worked as well as they did.

Reply to  Erik Magnuson
February 25, 2023 1:07 pm

I am the first to defend the importance of latent heat transport in Earth’s climate system. But to understand each other we have to speak the same language, and evaporation and latent heat transport are not considered a part of convection in climate analysis. Convection is considered the ascent of warm air due to thermal buoyancy, regardless of its humidity. That is a small part of vertical energy transport, although an important part of horizontal energy transport.

Reply to  Erik Magnuson
February 25, 2023 2:24 pm

driving the temperatures lower until a combination of dust and orbital changes 

I postulate glaciation is self limiting. It is a true positive feedback in climate. Glaciation eventually gets to the point where calving and overhanging ice shelves break away and cool large areas of ocean surface. That causes a dramatic reduction in the water cycle and snowfall.

Once the water level begins to rise, the ice shelves come under increasing stress and formation of large ice bergs accelerates.

There is evidence in the Arctic Ocean and North Atlantic of glacier keels draging across ocean floor that is now 700m deep. That means there were some very large icebergs.

The big berg in Baffin Bay that grounded at Davis Strait caused Greenland to go cold again after it started to warm up.

William Howard
Reply to  RickWill
February 25, 2023 5:57 am

Not to mention that the vast majority of the CO2 in the atmosphere – something like 95-97% is naturally occurring so removing the minuscule amount related to transportation Nd industry doesn’t change the composition of the atmosphere in any meaningful way – it defies all common sense to believe a minuscule amount,of CO2 somehow magically controls the earth’s temperature or its climate

Richard Greene
Reply to  William Howard
February 25, 2023 9:26 am

About 33% of the 420ppm CO2 is from manmade CO2 emissions, not 3% to 5%

The 3% to 5% claim is claptrap and will cause people to ignore everything else you say about the climate,

That 3% to 5% claim seems to have come from two science frauds, Ed Berry and Murray Salby.

CO2 emissions added something in the range of +200ppm to _300ppm CO2 to the atmosphere since 1850. The actual CO2 level rose about +140ppm

Please explain how the CO2 level increased +140ppm if not from manmade CO2, and then explain exactly what else did cause the +140ppm rise.

You will not be able to answer these two questions, as no one else has in the past five years I have asked them, so stop claiming manmade CO2 only accounts for 3% to 5% of CO2.

You are confusing the seasonal carbon cycle with annual emissions of additional CO2.

Although manmade output of 37 gigatons of CO2 is tiny compared to the 750 gigatons (about 5%) moving through the carbon cycle each year, it adds up because the land and ocean cannot absorb all of the extra CO2. About 60% of this additional CO2 is absorbed. The rest remains in the atmosphere, and as a consequence, atmospheric CO2 is at its highest level in 15 to 20 million years. 

Human CO2 emissions upset the natural balance of the carbon cycle. Man-made CO2 in the atmosphere has increased by a third since the pre-industrial era, 

Reply to  RickWill
February 25, 2023 7:37 am

C02 controls the energy balance. The idea that water can have any influence is naive nonsense.



Reply to  Steven Mosher
February 25, 2023 1:14 pm

Where is most warming coming from in the CO2 hypothesis?

Not CO2. It has a small effect by itself of about 1ºC per doubling.

Reply to  Steven Mosher
February 25, 2023 1:18 pm

Steven Mosher February 25, 2023 7:37 am

C02 controls the energy balance. The idea that water can have any influence is naive nonsense.

The facts beg to differ …

w.

Graham
Reply to  Steven Mosher
February 26, 2023 8:19 pm

I thought that you Mosher had a working brain .
I now see that you have swallowed the climate nonsense that CO2 is the climate control knob .
The three climate optimums that have occurred since the end of the major ice age 12 thousand years ago were NOT caused by rising levels of CO2 and the Little Ice Age was NOT caused by reducing levels of CO2 .
Water vapour in the atmosphere swamps any effect that increasing CO2 levels had on the climate millions of years ago .
The effect of CO2 is logarithmic and further increases will have no measurable effect on temperature .
The sun heats the sea water in the tropics which causes convection and the typhoons, hurricanes and cyclones track north or south to dispel the heat in the polar regions .
This has been happening for thousands of years and records show that the severity of these events is not increasing .

Robert B
Reply to  Javier Vinós
February 24, 2023 4:56 pm

“You cannot make the atmosphere more opaque to IR and not alter the energy balance. We simply don’t know by how much.”
Except you can (theoretically). The atmosphere is almost completemy opaque at wavelengths strongly absorbed by CO2, until well above the tropopause, where the temperature gradient is positive. Any increase in CO2 should lead to increase in the temperature of the effective emitting surface and cooling to a lower equilibrium temperature at the surface.

I’m not saying that this is the case, it’s just not an obvious fact that our emissions caused any warming at all.

Reply to  Robert B
February 24, 2023 11:39 pm

Any increase in CO2 should lead to increase in the temperature of the effective emitting surface and cooling to a lower equilibrium temperature at the surface.

If that were the case, we would be seen an increase in temperature at the stratosphere with the increase in CO2. Just the opposite happens.

75% of the greenhouse effect is due to water vapor and clouds, and most of it does not reach the tropopause. The idea that the Earth emits from above the tropopause most of its radiation is not possible.

Richard Greene
Reply to  Javier Vinós
February 25, 2023 1:38 am

“75% of the greenhouse effect is due to water vapor and clouds”

The common estimate for water vapor is 60% to 65% of the greenhouse effect, and clouds add to that, but the percentage seems to be a guess. I assume clouds have a net cooling effect for our planet.

I do not believe much of the CHANGE in the greenhouse effect over time is due to water vapor and clouds.

Reply to  Richard Greene
February 25, 2023 2:36 am

The cooling effect of clouds does not come from their greenhouse effect but from their albedo effect. Clouds do increase the GHE and there are estimations for that.

Water is responsible for about 75% of the GHE. Most GHE changes should come from water and nearly everybody agrees on that.

Richard Greene
Reply to  Javier Vinós
February 25, 2023 3:15 am

I tried to say that clouds have a net cooling effect by blocking some sunlight, even though they also have a greenhouse effect in the other direction.
I do not think that statement is controversial.

The current consensus guess is that water vapor is responsible for 60% to 65% of the total greenhouse effect, not 75%.

I believe your statement that most of greenhouse changes should come from water vapor, which is a feedback, is a controversial statement, and is wrong.

Reply to  Richard Greene
February 25, 2023 9:44 am

Those numbers I cite come from a commonly cited source:

Schmidt, G.A., Ruedy, R.A., Miller, R.L. and Lacis, A.A., 2010. Attribution of the present‐day total greenhouse effect. Journal of Geophysical Research: Atmospheres115(D20).

You cannot be more consensus than Gavin Schmidt.
Water vapor 50% (all sky)
Clouds 25%.

Most of the warming and most of the GHE changes are supposed to come from water vapor + clouds. This might be wrong but it is not controversial in the least.

Richard Greene
Reply to  Javier Vinós
February 25, 2023 1:19 am

We know what CO2 does in a laboratory, with and without water vapor. Not that much above 400ppm. There is no reason to fear 800ppm CO2 and every reason to welcome 800ppm CO2 — such as better growth of C3 plants, which will support more human and animal life on our planet. And likely a more moderate climate in colder nations too,

There is no reason to claim CO2 works differently in the atmosphere than in a laboratory. The right answer: CO2 above 400ppm does not do very much even when you add a water vapor positive feedback.

We already know 10x as much CO2 in the past atmosphere never caused runaway warming because there is obviously a negative feedback that limits the water vapor positive feedback. Ny guess is more clouds as tropospheric water vapor increases.

We also know Antarctica is not melting from more CO2.

So the fear of CO2 as “pollution” is insane. But fear of CO2 is great propaganda for leftists to gain political power and control over the private sector, so it will continue because it is working for that devious purpose.

Reply to  Richard Greene
February 25, 2023 2:39 am

There is no reason to claim CO2 works differently in the atmosphere than in a laboratory.

What a curious statement. Are you using the “argumentum ad ignorantia” fallacy? Everything works differently in the atmosphere than in a laboratory because you cannot reproduce the atmosphere in a laboratory. That’s why they are trying computers and failing.

Richard Greene
Reply to  Javier Vinós
February 25, 2023 3:19 am

CO2 is a weak greenhouse gas in a laboratory

It is a weak greenhouse gas in the atmosphere

Any claim that is not true is claptrap.

My statement was not “curious”

It was logical

Your claim implying that CO2 has a completely different effect in the atmosphere is data-free claptrap.

Reply to  Richard Greene
February 25, 2023 6:41 am

The screen shot is from Anthony’s CO2 jar experiment. It show that in his experiment additional CO2 produces a lower temperature.

D044EB0F-431B-4030-9D88-8179A12B5C3F.png
Richard Greene
Reply to  mkelly
February 25, 2023 9:46 am

Then the experiment is obviously wrong

Reply to  Richard Greene
February 25, 2023 11:06 am

So Feynman was wrong?

Reply to  Richard Greene
February 25, 2023 9:46 am

CO2 is indeed a weaker IR absorber than others but is more abundant than most, so it is still responsible for 29% of the greenhouse effect, which is a lot.

Reply to  Javier Vinós
February 25, 2023 4:53 am

“We simply don’t know by how much.”

That’s the answer we want and the answer nobody has.

But it’s all academic anyway, because, as you say, “CO2 is innocent. Its fingerprint is not found at the crime scene. Something else is warming the planet and causing the increase in OLR.”

How sweet it is!

Reply to  RickWill
February 24, 2023 10:42 pm

wrong wrong wrong

Richard Greene
Reply to  RickWill
February 25, 2023 1:10 am

Earth’s energy balance.

There is no energy balance
Our planet is not in thermodynamic equilibrium

Reply to  Richard Greene
February 25, 2023 2:45 am

No, but matter tries to reach thermal radiative equilibrium, so the concept is valid.

If you place an object at a different temperature inside a vacuum recipient it will equilibrate its temperature with the environment through net thermal radiation exchange.

The fact that equilibrium is a moving target doesn’t mean the Earth isn’t always trying to get there.

Richard Greene
Reply to  Javier Vinós
February 25, 2023 3:20 am

Trying to get there is not the same as getting there

I’ve been trying to get to a one million dollar net worth since 1977 — so far I have $129.

Tom.1
February 24, 2023 2:33 pm

The “heat trapping” explanation may be somewhat inept with regard to physics, but it is sufficient for the average citizen. The GHE acts to make the atmosphere warmer than it would otherwise be. For most people, that is all they need to know. It is the narrative that is important, not the physics (unless you’re a physicist).

Reply to  Tom.1
February 24, 2023 4:43 pm

It should be simplistically explained as “slowing the cooling rate,” rather than “heat trapping.”

Richard Greene
Reply to  David Middleton
February 25, 2023 1:40 am

It should also be explained that global warming is a lot more pleasant than global cooling, and we have centuries of anecdotes to prove that. People tend to vacation in warmer places too.

bdgwx
Reply to  David Middleton
February 25, 2023 6:51 am

“trap” means ΔEout < ΔEin resulting in ΔE > 0.

“cooling” means dT/dt < 0.

“slowing the cooling rate” means d2T/dt > 0 when dT/dt < 0.

Since dT/dt > 0 we cannot say the planet is “cooling” (because it is actually “warming”) and thus we cannot say “the cooling rate is slowing down”…ya know…because you cannot slow something that isn’t happening in the first place.

“trap” is the appropriate terminology for what is happening since ΔE > 0.

Reply to  bdgwx
February 27, 2023 4:27 am

comment image

The planet doesn’t still cool at night? The greenhouse effect slows the rate of radiative cooling.

bdgwx
Reply to  David Middleton
February 27, 2023 7:31 am

David Middleton said: “The planet doesn’t still cool at night?”

I didn’t say that it didn’t.

David Middleton said: “The greenhouse effect slows the rate of radiative cooling.”

Yep. It certainly does that especially at night.

But that is a goal post move. In the context of the article and Tom.1’s comment the GHE does not slow the rate of cooling because the planet isn’t cooling. What it does is trap heat because  ΔEout < ΔEin resulting in ΔE > 0.

And you give me all the implied facepalms you want. It does not change the fact that in this context dT/dt < 0 is false and that ΔEout < ΔEin resulting in ΔE > 0 is true.

Reply to  bdgwx
February 27, 2023 8:13 am

Like it or not, the surface cools. CO2 will shunt some of the radiated heat into a reservoir. The surface cools!

The lower atmosphere also warms by conduction with the surface and by contact with CO2. Convection then cools the lower atmosphere. And guess what? The surface keeps on cooling.

Heat is never trapped. It is in constant movement. Its rate from here to there may change but it is never static.

Your mixed up math using Energy and Temperature is so incomplete one can’t make sense of it.

If you refuse to learn from Planck, then study this wiki page:

https://en.m.wikipedia.org/wiki/Heat

you will find entropy is an integral part of heat transfer!

Reply to  bdgwx
February 27, 2023 10:19 am

You’re the one moving the goalposts.

comment image

The heat isn’t “trapped.” It’s return to space is delayed by greenhouse gases. It takes longer to radiate away, but it still radiates away. As I noted earlier:

It should be simplistically explained as “slowing the cooling rate,” rather than “heat trapping.”

Reply to  David Middleton
February 27, 2023 11:05 am

I don’t think he understand the use of gradients. “Trapping” heat can only occur under very controlled conditions. Otherwise entropy is in control and under natural conditions it doesn’t go backwards. Think irreversible.

Reply to  Jim Gorman
February 27, 2023 2:11 pm

I don’t think English is he primary language.

bdgwx
Reply to  David Middleton
February 27, 2023 4:51 pm

English is my primary language. However, my grammar and spelling are subpar. If that creates an impediment to discussions then I am truly sorry. I am trying to do a better job at proof reading my posts.

bdgwx
Reply to  David Middleton
February 27, 2023 12:37 pm

DM said: “The heat isn’t “trapped.”

Yes, it is. ΔEout < ΔEin resulting in ΔE > 0. And I stand by that statement in exact position I originally placed it. I have no interest in moving it.

BTW…a strawman argument is one in which person A creates an argument, pins it on person B even though B never advocated for it, and then criticizes it. Your person A. You created the argument that locations don’t cool at night and pinned it on me. But I never advocated for that argument. You and you alone created that argument. That’s fine. Just don’t expect me to defend it.

The two arguments I created were 1) the ΔEout < ΔEin resulting in ΔE > 0 which means heat is trapped and 2) that dT/dt < 0 is false meaning that cooling isn’t taking place let alone a reduce in the rate of it. If you want to challenge an argument I’m making then by all means. Just make sure it is one of those 2 and not one the ones you made up.

Reply to  bdgwx
February 27, 2023 1:57 pm

The radiative equilibrium temperature of the planet Earth (based on the amount of radiation energy that the planet emits to space) is quite a bit colder than the average temperature of the Earth’s surface. The reason this is possible is because the atmosphere plays a large role in the emission of infrared radiation out to space. In effect, it slows down the net rate at which the ground surface cools by radiation. This is known as the greenhouse effect.

http://www.atmo.arizona.edu/students/courselinks/spring17/atmo336s2/lectures/sec3/energybudget2.html#:~:text=The%20reason%20this%20is%20possible,known%20as%20the%20greenhouse%20effect.

comment image

bdgwx
Reply to  David Middleton
February 27, 2023 2:33 pm

Which I think is poor wording because the ground isn’t actually cooling. It’s warming. What they actually mean and what should have been said instead is…In effect, it slows down the net rate at which the ground surface sheds energy by radiation. Reducing the energy shed rate (ΔEout < 0) does not necessarily imply that cooling is happening (dT/dt < 0) or that it has slowed down (d2T/dt > 0). That’s my beef and why I think “trapping heat” is a far better way of describing what happens.

And don’t hear what I didn’t say. I didn’t say the ground temperature does not cool with the diurnal cycle. I’m talking about the global average temperature over long periods of time. It is going up; not down.

Reply to  David Middleton
February 27, 2023 1:39 pm

If the temperature goes up doesn’t the amount of radiation go up as well? So isn’t more heat lost as the temperature goes up?

I’m not even sure that “slowing the cooling rate” is even an apt description. I would think that the main result of the temperature going up during the day would be increasing the beginning value of the decay function.

R(t) = R0 / e^(λt)

As R0 goes up so does R(t=0). So the hotter the daytime gets the more radiation the Earth dumps out initially. Then the question is what does CO2 do to λ? Not sure I have a good feel for what happens there. Certainly the amount of heat that gets dumped at night is the integral over time of R0/e^λt dt. So what does CO2 do to the integral?

Reply to  Tim Gorman
February 27, 2023 2:10 pm

Without greenhouse gases, Earth would rapidly reradiate infrared energy back into space and the average surface temperature would be significantly cooler.

The radiative equilibrium temperature of the planet Earth (based on the amount of radiation energy that the planet emits to space) is quite a bit colder than the average temperature of the Earth’s surface. The reason this is possible is because the atmosphere plays a large role in the emission of infrared radiation out to space. In effect, it slows down the net rate at which the ground surface cools by radiation. This is known as the greenhouse effect.

http://www.atmo.arizona.edu/students/courselinks/spring17/atmo336s2/lectures/sec3/energybudget2.html#:~:text=The%20reason%20this%20is%20possible,known%20as%20the%20greenhouse%20effect.

Greenhouse gases slow the radiative cooling process. They effectively convert infrared.

Certain gases in the atmosphere have the property of absorbing infrared radiation. Oxygen and nitrogen the major gases in the atmosphere do not have this property. The infrared radiation strikes a molecule such as carbon dioxide and causes the bonds to bend and vibrate – this is called the absorption of IR energy. The molecule gains kinetic energy by this absorption of IR radiation. This extra kinetic energy may then be transmitted to other molecules such as oxygen and nitrogen and causes a general heating of the atmosphere. 

http://chemistry.elmhurst.edu/vchembook/globalwarmA5.html#:~:text=The%20infrared%20radiation%20strikes%20a,this%20absorption%20of%20IR%20radiation.

The only real question is how much warming is caused each doubling of the atmospheric CO2 concentration. Observation-based (instrumental) estimates indicate a low sensitivity. Model-based estimates indicate a high sensitivity.

comment image

https://wattsupwiththat.com/2020/07/15/pat-michaels-worse-than-we-thought/

Bear in mind that those are the equilibrium climate sensitivities (ECS). The only sensitivity that matters to people is the transient climate response (TCR), which is about 1/2 to 2/3 of the ECS. The TCR occurs simultaneously with rising CO2. The difference between the ECS and TCR occurs over the next 500 years, or so, as Trenberth’s missing heat returns from the depths of the oceans.

Reply to  bdgwx
February 27, 2023 6:55 am

Your explanation is terribly unsophisticated to the point of being wrong. Radiation is captured by CO2 and shunted to a heat reservoir (N2/O2) whose loss of heat has a slower rate of heat loss. Heat is still not trapped, it is merely moved around to points with different gradients.

AGW is Not Science
Reply to  David Middleton
February 25, 2023 6:57 am

And part of the explanation should always be “all other things held equal,” since that is the foundational assumption that leads to the hypothetical conclusion of an effect on temperature.

For which there is no empirical evidence in support, and a good deal of empirical evidence against.

Reply to  AGW is Not Science
February 27, 2023 4:28 am

Yep… And Earth, as a system, pretty well never allows  “all other things held equal.”

Reply to  Tom.1
February 24, 2023 7:22 pm

My apologies to Javier in advance of declaring his normally very astute writings inadequate in this case…..But…

The “emissions height” concept is a poor way to explain GHE. It makes people believe that IR emissions are from some fixed height over the whole planet, when such is not the case at all. An IR camera on a satellite sees the ocean surface at its temperature, land at its temperature, and cloud tops at their temperature. It also sees shades of gray in certain IR bands where water vapour and CO2 have absorbed the surface emission of IR and re-emitted at their new temperature at higher altitude.

What the camera sees depends on what bandwidth pass filters are in front of the lens.
In the 8 to 14 micron band, the atmosphere is nearly transparent to IR. About 80% of IR photons in those frequencies get directly to outer space, other than that blocked by dust and O3. Also, if low and mid-cloud cover is 65%, then 65% more IR photons will be blocked from reaching outer space. This IR range represents Wein’s law temperature from +90 C to -65 C, which pretty well covers all Earthly surface temperatures. Of course I’m referring to the “atmospheric window”.
And the black body curves for those +90 to -65 temperatures shows about 40% of their total emissions are in that 8-14 micron IR band.

So Earth as viewed by an IR camera from outer space is mostly a mosaic of surface and cloud top temperatures. It is not an emission from a certain height as one might expect for a thick atmosphere like Venus.

The emission height concept is a mathematical construct no different than calculating the average depth of snow in North America…not really useful for explaiming anything about the climate.

The best treatment of this topic is Happer and van Wijngaardens 3 watt widening of the 15 micron CO2 valley in their Figure 10 of the following article.

https://co2coalition.org/wp-content/uploads/2022/03/Infrared-Forcing-by-Greenhouse-Gases-2019-Revised-3-7-2022.pdf

Reply to  DMacKenzie
February 25, 2023 12:23 am

So Earth as viewed by an IR camera from outer space is mostly a mosaic of surface and cloud top temperatures.

Not really. Only 17% of IR emissions come from the surface. When the camera is looking at a point in the Earth it is adding all the IR emissions from the surface and the atmosphere.

Earth’s IR emissions to space are coming at all sorts of temperatures that when added give a spectrum very similar to what a blackbody at 255 K would give. This is the emission temperature of a planet. It matches the energy received from the Sun minus the albedo.

The effective emission height is a concept simplification that helps understand something a lot more complex. If all the radiation from the Earth was coming from that height it would have the same emission temperature. Nothing more, nothing less.

Reply to  Javier Vinós
February 25, 2023 9:39 am

If all the radiation from the Earth was coming from that height it would have the same emission temperature.

Yes, I agree, unfortunately that means your description is an oversimplification of what is going on….to the point of being an incorrect thing to teach laypersons.

Only 17% of IR emissions come from the surface

You need a rethink on that, you are confused by the “back-radiation” watt numbers that can’t actually exist without “fore-radiation” from a warmer source.
It is a common error, even amongst physics Ph.D.s to invoke the [minus Tcold^4] term in Planck’s equation when the pertinent number is [Thot^4 minus Tcold ^4] which must be a positive number.
About 165 watts of 240 is absorbed by the surface and must be reradiated, from down here where “climate” is located. That’s 70%.
And check latest NASA radiation budget. Upwelling IR this 398 as a result of Tsurf, downwelling 340 as a result of Tsky, IR to outer space 40…..40/(398-340) equals 70%.
Circular reasoning you might be inclined to say, but it shows that watts/sw.m. due to extra surface temp will be 70% sent to outer space through the atmospheric window.

Sorry a comment here can’t adequately cover that which needs study of atmospheric thermo, and radiative heat transfer…..You obviously have the mind and dedication to pursue the topic.

Think of it this…say you sit naked in a 20 C room. Your skin temperature is say 33 C. Your skin is radiating 497 watts per square meter. The walls of the room are radiating back to you 418 watts per square meter. Your metabolism needs only generate 497-418=81 watts/sq.M. When you can reconcile this in your mind, plus colder or warmer walls, some heat escaping through a window, and so on….reconciling with Climate radiation budget diagrams and incoming sunshine, without cognitive dissonance….you’ll be good.

Reply to  DMacKenzie
February 28, 2023 6:10 am

You are still confused, DMac. “Your skin temperature is say 33 C. Your skin is radiating 497 watts per square meter” No it isn’t. That would only be true if you were in outer space. In a room temperature room, the actual power you emit is less than 100 W/m^2. The wall is not “radiating back to you 418 watts per square meter”, because you are warmer than the wall. Power doesn’t develop in that direction, entropy won’t allow it.

I am not sure how you managed to write this statement in the same post where you wrote “It is a common error, even amongst physics Ph.D.s to invoke the [minus Tcold^4]” That’s exactly the error you just committed… you left out the target temperature, you assumed it was 0. Don’t do that.

bdgwx
Reply to  stevekj
February 28, 2023 7:22 am

DMackenzie is right. sblaw(33) = 498 W/m2. sblaw(20) = 419 W/m2. The flow of heat is thus sblaw(33) – sblaw(20) = 79 W/m2 from you to the wall. If you were only radiating at 100 W/m2 you’d frozen solid and dead.

Reply to  bdgwx
February 28, 2023 8:00 am

He’s not right. The S-B law with a reference temperature of 0 is only applicable in outer space. If you were only radiating at 100 W/m^2 in outer space, then yes, that would correspond to a cold and dead temperature.

bdgwx
Reply to  stevekj
February 28, 2023 9:47 am

No one is using a reference temperature of 0 K. The temperatures are 306 K (33 C) for his body and 20 C (293 K) for the room.

If you were radiating at 100 W/m2 then his body temperature is -68 C (205 K) regardless of whether his is in the room or in space.

And we can use the 1LOT to show that his metabolism has to contribute 79 W/m2. Because F = E/(A*t) where t is time and A is area and because A and t are the same for the system and period being analyzed we can restate the 1LOT as ΔF = (Fin_room + Fin_metabolism) – Fout. At 306 K (33C) we know Fout is 498 W/m2. And because his body is in steady-state (no temperature change) we know that ΔF = 0 which means Fin_room + Fin_metabolism = Fout. Solving for Fin_metabolism we have Fin_metabolism = Fout – Fin_room. And we know Fin_room = 419 W/m2. So Fin_metabolism = 498 W/m2 – 419 W/m2 = 79 W/m2. The 1LOT does not allow it to be any other way.

Reply to  bdgwx
February 28, 2023 10:10 am

“No one is using a reference temperature of 0K.” Yes, you are absolutely using a reference temperature of 0K. If your body is at 306 K, the only way it can produce 497 W/m^2 is if you plug 0 into the “Tcold” part of the general S-B equation. That gives you a specialized S-B equation, which is only applicable in outer space, but a lot of non-physicists don’t seem to have grasped that part.

Naturally, if you are in outer space and emitting that much power to your surroundings, which are close to 0K, you will rapidly freeze to death, but that’s beside the point.

bdgwx
Reply to  stevekj
February 28, 2023 10:36 am

The Stefan-Boltmann Law says I = σT^4. For his body it would be I = 5.67e-8 W/m2.K4 * (306 K)^4 = 498 W/m2. 0 K isn’t used at all.

The radiative heat transfer equation Q/t = σ(Th^4 – Tc^4) is derived from the SB equation above and the 1LOT equation ΔE = Ein – Eout. For his body and the room it would be Q/t = 5.67e-8 W/m2.K4 * ((306 K)^4 – (293 K)^4) = 79 W/m2. 0 K isn’t used at all here either.

Note that I in the SB equation is radiant exitance while Q/t in the heat transfer equation is the rate at which heat is transferred. They are different, albeit related, concepts that happen to have the same units.

Reply to  bdgwx
February 28, 2023 11:27 am

“Radiant exitance” is an energy unit, not a power unit. It isn’t “an energy unit that happens to be measured in Watts”. Radiation is energy, not power. The first form of the S-B law that you showed is a specialized case where Tc = 0. Converting energy (Joules) to power (Watts) is done by (among other means) the general “S-B radiative heat transfer equation”, which you also showed above. It requires an entropy (temperature) gradient. So you can’t just ignore the Tc term. Setting that to 0 requires some sort of justification, such as “being in outer space”. You can’t just convert temperature (energy) into Watts (power) willy-nilly and hope that no one notices… Stefan and Boltzmann certainly knew the difference. It doesn’t sound like you do, though.

Here’s a quick quiz: how much power do you think is developed by an object at equilibrium with its surroundings? Say, at room temperature or so? And if your answer isn’t “zero”, just what exactly do you think the word “power” means?

bdgwx
Reply to  stevekj
February 28, 2023 1:19 pm

Radiant exitance is a flux. The units are W/m2. Radiation is electromagnetic energy in transit. It can be expressed in joules, watts, or W/m2 depending on how you want to analyze it. The first equation I posted is a special case of the SB law only in that it is for a blackbody with emissivity ε = 1. There is no Tc in the SB law because the SB law only relates a blackbody’s temperature to its radiant exitance. The SB law says blackbodies emit radiation and thus energy all of the time and anywhere they happen to be. There is no requirement that there be an entropy gradient between the blackbody and its surroundings for the blackbody to emit radiation. A body at temperature T has the same radiant exitance regardless of whether it is in outer space, on Earth, or even a room that is also at temperature T.

Power is the amount of energy converted/transferred per unit time. The units are joules/second or watts (W). An object X and its surroundings Y has a radiant exitance Ix = σTx^4 and Iy = σTy^4 respectively. The heat transfer between the two is given by the 1LOT and is Q = tA*(Ix – Iy) = tA*[(σTx^4) – (σTy^4)] = tA*σ(Tx^4 – Ty^4). The power of the transfer is thus P = Q/t = Aσ(Tx^4 – Ty^4). And then by letting T = Tx = Ty since they are in equilibrium we see that P = Aσ(T^4 – T^4) = Aσ*0 = 0 W. Notice for a moment that the 1LOT and SBLAW combine to produce the familiar radiant heat transfer equation. Anyway, that’s your answer…P = 0 W. Repeating the math for DMacKenzie’s scenario we see that P = 79*A W.

Reply to  bdgwx
February 28, 2023 2:25 pm

You seem to be very confused. But at least partly on the right track.

“Radiation is electromagnetic energy in transit. It can be expressed in joules, watts, or W/m2” No, energy cannot be expressed in Watts. Who told you that? It can be expressed in Joules, degrees, height (for potential energy), etc. But not Watts. Perhaps you got confused by the “in transit” part and assumed that energy that is moving must be producing power. But that is not the case.

“The SB law says blackbodies emit radiation and thus energy” This is a mis-statement. Planck’s law says that blackbodies above absolute 0 emit radiation, which is energy, and describes the spectral distribution. Energy is not the same as power, though. The S-B law, meanwhile, allows you to take objects at two temperatures and derive the power being transferred between them. It does not say anything else.

This statement is false: “An object X has a radiant exitance Ix = σTx^4” That statement is equivalent to converting temperature directly into power. That isn’t how energy and power work. I can tell that you know that. So if you stop making this particular statement, everything else you wrote will be basically correct. In particular, a correct statement would be “An object X has a radiant exitance E, or T, in joules, or degrees, based on its temperature”.

You did correctly calculate that an object in equilibrium with its surroundings produces 0 W of power, and DMac’s scenario for a human being in a room is around 79 W. These numbers are correct. Values greater than 400 W are not. Not even as intermediary terms that are going to cancel out. Those are false and imaginary. For example, you wouldn’t be able to measure them if you tried, because they don’t exist. Your understanding of the world will be much improved if you drop the imaginary intermediate values. Just plug the two temperatures you are looking at directly into the general S-B equation and you will get your answer, with a lot less faffing around, and no imaginary values.

bdgwx
Reply to  stevekj
February 28, 2023 3:40 pm

stevekj said: “No, energy cannot be expressed in Watts”

Sure it can. A watt is joules/second. It is the rate of energy conversion or transfer.

stevekj said: “Who told you that?”

Thermodynamics.

stevekj said: “Perhaps you got confused by the “in transit” part and assumed that energy that is moving must be producing power. But that is not the case.”

I don’t think so. If energy is being converted or transmitted then it is doing so with a rate. I’m not sure I’d describe that as “producing power” though. It’s really more of it just doing so with a power.

stevekj said: ““The SB law says blackbodies emit radiation and thus energy” This is a mis-statement.”

I don’t think so. Q = tAσT^4. It certainly looks like blackbodies emit radiation and thus energy according to the math.

stevekj said: “Energy is not the same as power, though.”

Yeah. I know. But they are related. P = Q*t. Just so we’re clear…when I say [it] can be expressed in joules, watts, or W/m2 I’m talking about [electromagnetic radiation in transit]. For example, radiant exitance is electromagnetic energy in transmit from a blackbody and is expressed as a flux in units of W/m2.

stevekj said: “The S-B law, meanwhile, allows you to take objects at two temperatures and derive the power being transferred between them.”

No. That’s not right. The SBLAW allows you to relate the temperature and radiant exitance of a single body. I and T in the equation are for the same one and only one body. It’s the derivation of the radiant heat transfer equation from the SBLAW and 1LOT that allows you to calculate the power of the heat being transferred between two bodies. The SBLAW itself does not do that.

stevekj said: “This statement is false: “An object X has a radiant exitance Ix = σTx^4””

It’s true. The SBLAW says Ix = σTx^4.

stevekj said: “That statement is equivalent to converting temperature directly into power.”

It’s actually equivalent to saying that radiant exitance for a blackbody is proportional to the 4th power of temperature.

stevekj said: “So if you stop making this particular statement, everything else you wrote will be basically correct.”

Sorry. I’m not going to stop saying that Ix = σTx^4. If I stopped saying that then I would be implicitly rejecting the SBLAW.

stevekj said: ““An object X has a radiant exitance E, or T, in joules, or degrees, based on its temperature”.

That’s not right. Radiant exitance is the flux exiting a surface per unit area and has units of W/m2. It obviously can’t be energy (E) or temperature (T).

stevekj said: “You did correctly calculate that an object in equilibrium with its surroundings produces 0 W of power, and DMac’s scenario for a human being in a room is around 79 W.”

Thanks.

stevekj said: “Values greater than 400 W are not.”

It’s a good thing neither I nor DMacKenzie said the transfer was greater than 400 W.

stevekj said: “Not even as intermediary terms that are going to cancel out. Those are false and imaginary. For example, you wouldn’t be able to measure them if you tried, because they don’t exist. Your understanding of the world will be much improved if you drop the imaginary intermediate values.”

I have no idea what you’re trying to say here. What intermediate value?

stevekj said: “Just plug the two temperatures you are looking at directly into the general S-B equation and you will get your answer”

The SBLAW (assuming ε = 1 anyway) only accepts one free parameter. It’s T. It does not have two T terms.

stevekj said: “with a lot less faffing around, and no imaginary values.”

Say what? Imaginary values? What are you talking about?

Reply to  bdgwx
March 1, 2023 8:13 am

There is a lot missing in this back and forth. Equilibrium is one. It means that at some point in time, an infinitely small point, these equations hold. You can not even assume they hold as an averageumpnless you make very specific assumptions about objects, which is what Planck did in his research. The exponentials involved make this a requirement.

Unless a body is a true source with a constant rate of flux and temperature, gradients are necessary to describe the heat loss/gain. The gradients must be integrated with respect to time in order to evaluate the bodies condition. The bodies involved are entirely different in measurable quantities. H2O has a higher specific heat than CO2. Land has a higher mass than air. These all cause different gradient equations.

Insolation is an example. Insolation follows a sine wave as the sun traverses the sky. Additionally, there is a cosine function based on latitude. Trying to use an average value for insolation doesn’t tell you that Miami gets much more sunshine in winter than Ottawa, Canada. That wouldn’t be a big deal if everything was linear but it isn’t. Lots of exponentials involved. Lots different gradients. Ask yourself if this variation can cause a difference in temperature growth, i.e., anomalies.

Don’t expect that simple equations tell you the whole story.

Reply to  bdgwx
March 2, 2023 5:26 pm

You sure are confused, bdgwx… let’s just take one of your incorrect statements:

“radiant exitance is electromagnetic energy in transmit from a blackbody and is expressed as a flux in units of W/m2.”

The basic error here is your usual one, namely “energy” is not the same as “W/m^2”. In one sentence you appear to know this, and in the next sentence you forget it again. Try to pay attention.

In a bit more detail, you cannot develop Watts from a single body. That’s not how Watts work. If you had a hypothetical universe consisting of a single object at room temperature floating in a vacuum, with no surrounding matter or energy of any kind, you would have no Watts, because there is nowhere for the energy to go.

Who taught you your theoretical physics? I’d like to have a chat with him, because he didn’t do a very good job. Did you pass your exams? Or are you just finding equations here and there on the Internet and trying to jam them together to get the result you want?

Remember, power is not just “energy in motion”, it is “energy in the process of doing work, i.e. rearranging matter so as to increase entropy” (at a particular rate). If entropy is not increasing, no power is being developed. A single uniform body sitting isolated by itself will not experience any change in entropy, and no power can be developed. No “Watts” anywhere. Just Joules, waiting patiently.

What you call the “S-B Law” is a specialization of the actual S-B law, which is a heat transfer calculation involving two bodies at different temperatures. The specialized one has set the second temperature to 0 K, which is not physically realistic. Even in outer space it is only an approximation, and Tc should really be around 3 K.

I know you can find lots of pages on the Internet claiming that the “S-B law” defines the amount of power produced by a blackbody at temperature T. But that only happens in outer space… You can tell that none of the pages that make this claim have studied their theoretical physics very carefully either. Most of them go on to write “energy” and “power” interchangeably, which tells you that they are not written by actual physicists. Black bodies emit energy, while power requires work, which requires two objects – a high-entropy one and a low-entropy one… it’s a tough concept, I know, but if you study hard I know you will get it.

If all of the preceding confused you, let me try another approach. In our example of an object in a room at equilibrium, you wrote:”Ix = σTx^4″, and you interpret I as a power measurement, apparently totally independently of any surroundings. So this gives you upwards of 400 W/m^2 emitted by the object at room temperature. (That’s the “fake power” that I talked about.) But next, you wrote “P = 0 W”. Now, bizarrely, you have concluded that one object is emitting both 0 W and also upwards of 400 W/m^2, at the same time. How much sense does that make in your head? Because it makes none at all in mine…

Reply to  stevekj
March 1, 2023 8:57 am

“””””No, energy cannot be expressed in Watts.”””””

It can and is. AM radio is not as popular as it used to be, but the stations were rated as to their output power expressed as Watts.

EM waves do carry energy, expressed as Joules. However Watts or Joules/second is used to designate how much energy is moved over a unit of time. It allows one to determine the amplitude of the electric and magnetic fields in a given EM wave. A 1000 Watt EM wave carries more power than a 100 Watt EM wave.

Remember power is work (Joules) divided by time (seconds). With an EM wave, it is the ability to do work divided by time. Higher amplitudes (more Joules) of the E and M fields can do more work. Thus more power is available.

Reply to  Jim Gorman
March 2, 2023 4:58 pm

You have some strange ideas, Jim… I said:

“Energy cannot be expressed in Watts.”

Then you said:

“It can and is.”

Then you said:

“EM waves do carry energy, expressed as Joules.”

You should probably make up your mind, because you made two contradictory statements. The second one is my point, and is the correct one.

Note that I never said “energy cannot develop power”. That is different, and of course it can, and that requires an entropy gradient. Every form of energy can develop power, given an entropy gradient, including EM radiation.

Reply to  DMacKenzie
February 28, 2023 9:01 am

“””””Your metabolism needs only generate 497-418=81 watts/sq.M.”””””

You have a slight misconception. SB @33° = 497 and SB@20° =418.You have two bodies. At only an instant in time you radiate 497 to the wall while at the same time the wall radiates 418 to you.The net radiation is 79 towards the wall. Your metabolism must continue supplying the energy to remain at 33° for this scenario to remain constant. That is, you must manufacture 497 W/m^2.

Far to many people fail to realize that most of these laws and equations are good for just an infinitely small period of time. You have an internal fire, so you can be a constant source. Without this, as you radiate, you cool (or warm) if you are the hot body (if you are the cold body). The rates of warming and cooling are determined by gradients which are made up of several other factors. Most of the equations are for black body’s which simplifies the overall use of radiation without gradients.

Reply to  DMacKenzie
February 25, 2023 12:03 pm

I also forgot Andy May’s write-up, his fig. 8 dispels the “emissions height of CO2” concept and his Fig 10 explains the GHE by widening of the 15 micron CO2 band. It’s at:
https://andymaypetrophysicist.files.wordpress.com/2021/09/happer_major_statement.pdf
which is very readable for the average climate rationalist.

Editor
Reply to  DMacKenzie
February 25, 2023 5:48 pm

DMacKnzie,
You are confused. The link you provide is to Professor Happer’s Major Statement. I did not write that, he did, I only preserved it. He agrees with what Javier is saying, this is from his statement:

“At the “emission altitude,” which depends on frequency ν, there are not enough greenhouse molecules left overhead to block the escape of radiation. The thermal emission cross section of CO2 molecules at band center is so large that the few molecules in the relatively warm upper stratosphere (see Fig. 5) produce the sharp spikes in the center of the bands of Fig. 8.”

There is no inconsistency between what Happer wrote and what Vinós wrote.

Reply to  Andy May
February 26, 2023 9:27 am

Andy,

I thought it was your evaluation of Happer’s papers, sorry. It is one of the best write-ups on GHE around.

Anyway, you left off Happer’s lead in statement on the same page 15 comments that you use…..

“Although the surface radiation is completely blocked in the bands of the greenhouse gases, as one would expect…”. (Italics mine)

The inconsistency is that Javier’s explanation implies that the “emissions altitude” effect IS across the whole IR spectrum when it is not, and that emissions altitude explains the GHE when it does not….It is simply a mathematical construct, like the average depth of snow in North America…not very useful…

I am not trying to denigrate anyone’s work here, only give readers with the desire to investigate more fully the GHE, and a sound basis for doing so.
Of which Professor Happer’s Major Statement is a very good synopsis.

Reply to  DMacKenzie
February 27, 2023 2:53 am

The inconsistency is that Javier’s explanation implies that the “emissions altitude” effect IS across the whole IR spectrum

It doesn’t imply that. There’s the atmospheric window that is not affected, and a change in CO2 affects only the frequencies absorbed by CO2. Since the height of emission at those frequencies is increased, the average comes higher. All averages are mathematical constructs. That doesn’t mean they are not useful.

Rud Istvan
February 24, 2023 2:39 pm

Javier, I would make two points.

First, the simplest way to think about the GHE has nothing to do with the oft misunderstood ‘backradiation’ (which is but one indicator of the physics actually involved). It is most simply an absence of sufficient radiative IR cooling.

Second, there obviously must be some CO2 GHE effect. Tyndall experimentally proved CO2 was a GHG in 1859. But its ‘fingerprint’ signal gets largely lost in the noise of all else also going on—natural variation, aerosols, land use change, contrail cirrus. And the sole alarmist claimed CO2 model ‘fingerprint’—the tropical troposphere hotspot—does not in fact exist. The only CMIP6 model without a tropical troposphere hotspot is INM CM5. And CM5 has an ECS of 1.8, meaning no alarm. (CM5 accomplished this mainly by incorporating observational ARGO ocean rainfall in its parameterization, thereby significantly reducing its water vapor feedback.)

Rick C
Reply to  Rud Istvan
February 24, 2023 3:44 pm

I think the majority of skeptics have a sound understanding of how the green house effect actually works and that “green house” is not a very good analogy. A better analogy may be the often used blanket effect. A blanket does not warm a persons body at all as it contains no source of energy. It simply slows the rate of cooling allowing the energy produced by the body to maintain a normal temperature. If a person dies in bed under a heavy blanket they will still eventually reach “room temperature”. How long it takes is dependent on how well the blanket insulates. As far as the climate goes, the effect of CO2 emissions from fossil fuel use is like adding a very thin sheet to a very heavy thick wool (or H2O) blanket.

I do like Javier’s test in that it follows the scientific method of making a prediction based on the hypothesis and then evaluating empirical evidence to see if the prediction is falsified or not. Way too little of this in Climatology.

Robert B
Reply to  Rick C
February 24, 2023 5:21 pm

The temperature, and other properties, of the outer surface of the topmost blanket will determine the loss of energy. So the insulating physics of a wool blanket is very different to that of a thin foil space blanket.

Neither is a good analogy. There is a temperature gradient in a wool blanket, so increasing the height of the TOA is like putting on another layer, but the reasons for the temperature gradients are very different. GHG are not responsible for the gradient, but for the assumption of an adiabatic movement of packets of air being incorrect (Good enough for fast flow of air over mountains).

Richard Greene
Reply to  Rick C
February 25, 2023 1:49 am

We just had a 35 hour blackout here in Michigan along with perhaps a one million other people. It was 55 degrees in the house. I slept under three blankets fully dressed with a wool cap on. The wife couldn’t sleep at all. The cat stayed on top of a floor heat vent waiting in vain for the furnace to turn on. Three cheers for blankets!

Reply to  Richard Greene
February 25, 2023 5:29 am

It sounds like you need an alternate source of energy.

Generac and propane would keep you cozy.

Dave Andrews
Reply to  Tom Abbott
February 25, 2023 7:23 am

So do cats 🙂

Richard Greene
Reply to  Tom Abbott
February 25, 2023 9:53 am

Our next door neighbor ran a 100 foot extension cord from his gasoline generator over to our house so we could turn on a small heater in the bedroom. Nice guy, After an hour of getting some heat, five DTE Energy linemen showed up in my yard and the electricity was on 30 minutes later. We were lucky this time — some people are still without power until Sunday. A transformer exploded in the back yard — the wife almost hit the ceiling — I thought a nearby home blew up — I was not expecting such fast service from DTE.

johchi7
Reply to  Richard Greene
February 25, 2023 9:00 am

Or a “Three dog night.”

johchi7
Reply to  johchi7
February 25, 2023 9:16 am

I should have said ‘large’ dogs.

Tom.1
Reply to  Rick C
February 25, 2023 8:25 am

There are a great many “skeptics” and many readers of this blog whose understanding of science is warped by their political views, which is exactly the same problem with people in the alarmist camp. You have people claiming there is no such thing as the average temperature of the atmosphere (or anything), or people claiming that hydrogen as a fuel is “suicidally dangerous”.

Reply to  Tom.1
February 25, 2023 2:05 pm

“You have people claiming there is no such thing as the average temperature of the atmosphere (or anything), or people claiming that hydrogen as a fuel is “suicidally dangerous”.”

There are people making such claims. WUWT allows such claims to be made. But WUWT usually requests that such claims be backed up by supporting facts. And then we get to sort out which claims are valid and which claims are not.

A wild claim on WUWT will be challenged. Count on it.

Tom.1
Reply to  Tom Abbott
February 25, 2023 2:58 pm

People make all kinds of bogus claims on this blog all the time and they are rarely disputed, except by me when I happen to know the claims are unsupported by facts or evidence. So, as it happens, you are wrong.

Tom.1
Reply to  Tom Abbott
February 25, 2023 3:31 pm

Further to my point, there was a very recent post here by a Mr. Eric Worall in which he referred to hydrogen as a “suicidally dangerous” substance. He never cited anything to back this up except an OSHA data sheet that carried a warning that could apply to just about any gaseous combustible. There is a weath of industrial experience to demonstrate that this is factually incorrect. Yet, not only did he author an article here, but no one took issue what his nonsense excpet me.

Reply to  Tom.1
February 25, 2023 4:22 pm

Here’s what Eric said:

The plan is, vehicles containing 10s of litres of compressed gas which is capable of leaking through the tiniest crack, has no smell, cannot be reliably odourised to warn people of leaks, burns with a flame so hot that it is invisible, ignites and explodes violently with a very low activation threshold, over a wide range of hydrogen / air mixtures, will be parked adjacent to and sometimes inside the builtin carports in people’s homes.

Also, as Eric pointed out, hydrogen

  1. Can escape through the very tiniest crack
  2. Burns with an invisible flame
  3. Has no odor and cannot be successfully odorized
  4. Burns at the widest range of concentrations of most gaseous fuels and vapors:
  • Hydrogen 4-74% flammability range
  • Propane 2.1-10.1%
  • Butane 1.9-8.5%
  • Methane (natural gas) 4.4-16.4%
  • Kerosene vapor 0.7-5%
  • Gasoline vapor 1.4-7.6%

None of those “apply to just about any gaseous combustible” as you incorrectly claim. Not one.

Me, I wouldn’t park a hydrogen car in an attached garage. Too suicidal for my taste.

You may think hydrogen is a brilliant plan.

Me, not so much.

w.

http://wattsupwiththat.com/2013/07/01/drilling-for-hydrogen/

Tom.1
Reply to  Willis Eschenbach
February 25, 2023 8:11 pm

Willis- Sorry to see you weighing in on the side on nonsense, but there you are. He also used the words “suicidally dangerous.” The fact is the world has a vast amount of experience using hydrogen in many different applications. It is used widely, and safely. Every space shuttle was launched with hydrogen, and it caused no accidents. Many, many NASA space vehicles have used hydrogen fuel cells safely. The US has hundreds of miles of hydrogen pipelines already in service. Virtually every refinery and chemical plant in the world uses hydrogen, Toyota, one of, if not the biggest auto manufacture in the world believes that fuel cell powered cars with hydrogen as fuel are going be future of automotive power and already has a commercial product in the form of the Marai. Siemans, this small technology company in Europe, is getting ready to ramp up their industrial scale hydrogen electrolyzers. You don’t have to take my word for any of this because these things are easily verified.

Reply to  Tom.1
February 25, 2023 11:47 pm

Tom, you’re right that highly trained people have “a vast amount of experience using hydrogen in many different applications”.

But your average car owner has exactly zero experience in handling hydrogen. And automobiles are often used continuously for decades by people with no knowledge of machinery, with little in the way of maintenance … you don’t find NASA or refineries or chemical plants doing that.

Yes, you are right that there are 1,600 miles of hydrogen pipelines in the US, once again maintained by professionals … but again, that’s very, very different from having tanks of pressurized hydrogen at every one of the 150,000 gas stations in the US, and pressurized hydrogen in every car in every garage.

Next, even with professionals at the helm we get things like this:

“Explosion at hydrogen fuel plant damages 60 nearby homes in Catawba County”

Finally, hydrogen is not an energy source. There are no hydrogen mines. It’s generally made from natural gas … so hey, why not just power your car on natural gas and avoid the waste and danger?

Hydrogen gas was first produced commercially two centuries ago. And it was first used to power a vehicle in 1826. There are a number of very valid reasons why in all that time our economy hasn’t adopted it as a widespread fuel source.

Best regards,

w.

Phil.
Reply to  Willis Eschenbach
February 26, 2023 7:49 pm

Actually for a long time hydrogen gas was supplied as a fuel source, known as town gas. In the UK it was supplied to houses until the late 1960s, the natural gas that replaced it turned out to be more of an explosion risk.

Reply to  Phil.
February 26, 2023 8:22 pm

Town gas is a mixture of roughly equal parts of carbon monoxide and hydrogen. It is not more explosive than natural gas, with a flammability range of 4.5 ~ 44.3%, versus 5 ~ 15% for natural gas. The use of town gas was discontinued because the carbon monoxide made it very poisonous.

w.

Phil.
Reply to  Willis Eschenbach
February 28, 2023 2:58 pm

It had become less popular because of the smell, but in the 60s they discovered Natural gas reservoirs under the North Sea and decided to make the switch. The explosion risk of Hydrogen is less than you might think because it diffuses very rapidly and becomes too lean to ignite. The problem when they introduced natural gas was that the flames on burners can ‘blow off’ and thereby produce an inflammable mixture in the house. Because hydrogen has a much higher flame speed that doesn’t happen. If hydrogen powered vehicles are introduced I think it’s most likely that the H2 will be stored in low pressure metal hydrides. About 20 years ago I was at a DoE meeting which was discussing the possibility of Hydrogen powered vehicles. The fuel manufacturers said that they were able to supply and distribute it if there was a sufficient demand and the auto companies said that they could design and make the cars if there was a adequate fuel supply network! Neither wanted to be the ones to make the first move.

Reply to  Willis Eschenbach
February 26, 2023 10:37 am

I have some experience with ignition energies. H2 in air will ignite if you drop a bolt on the concrete floor, and flash back to the leak source, which becomes a barely visible blowtorch…..All industrial facilities that use hydrogen take extreme precautions to avoid leaks, heat detectors, etc, much beyond that of the average attached garage.

DCBE8E0B-BCDB-4337-A9EC-0ED2F70E99DA.jpeg
Reply to  DMacKenzie
February 26, 2023 8:30 pm

Thanks, D. Yes, hydrogen can be handled safely.

But as you point out, that’s handled by trained professionals with proper procedures and leak and heat detectors, etc. … not by shade tree mechanics or your average humanoid.

w.

Reply to  Rud Istvan
February 24, 2023 3:48 pm

‘Second, there obviously must be some CO2 GHE effect.’

Some folks would say it already happened, meaning additional CO2 should have very little effect, and per Javier’s D&C graphic, is being swamped by something else? Just asking…

Reply to  Frank from NoVA
February 24, 2023 4:57 pm

It’s a diminishing returns function. Each doubling of the CO2 concentration yields a linear increase in temperature (ignoring feedback effects).

Example:

If going from 290 to 580 ppmv yields 1K of warming, 580 to 1160 ppmv would yield an additional 1K, 1160 to 2320 ppmv would yield an additional 1K, ad nauseum.

So the 1.5K (or C) doomsday CO2 concentration would be out around 870 ppmv.

Of course the actual sensitivity is unknown (possibly unknowable) and the net feedback effects are even more unknowable, but appear to be fairly neutral over geologic time, where CO2 effects seem to fall within the noise level.

Reply to  David Middleton
February 24, 2023 6:20 pm

Yes (to you and Rud). My statement was that we’re already pretty far along the curve given that there’s little evidence from the paleo that CO2 is the ‘control knob’.

Reply to  Frank from NoVA
February 25, 2023 12:26 am

The evidence from paleo is pretty shaky, but it does not support the CO2 control knob conjecture.

Richard Greene
Reply to  Frank from NoVA
February 25, 2023 1:52 am

There’s no evidence natural CO2 was ever a control nob — it seemed to be mainly a feedback to changes in ocean temperatures in the ice core era. per Henry’s Law. The CO2 rise from 2015 to 2023 did not control anything, nor did the CO2 rise from 1940 to 1975.

Reply to  David Middleton
February 25, 2023 5:34 am

“Of course the actual sensitivity is unknown (possibly unknowable) and the net feedback effects are even more unknowable, but appear to be fairly neutral over geologic time, where CO2 effects seem to fall within the noise level.”

Excellent!

That bears repeating.

Rud Istvan
Reply to  Frank from NoVA
February 24, 2023 5:02 pm

The GHE effect is logarithmic, Is enabled by more CO2 raising the ERL at a lapse rate ever inefficient ERL. First shown by Guy Callendar in 1927.

Reply to  Rud Istvan
February 24, 2023 4:55 pm

I agree, Rud. The back radiation idea is stoopid. For radiative-caused temperature changes the only thing that matters is the net flux.

The problem with INM CM5 is that even if it gives the correct answer it will be for the wrong reasons. Models have too many holes to be able to represent climate in any credible way even by approximation.

Richard Greene
Reply to  Javier Vinós
February 25, 2023 1:55 am

Models are programmed to scare people and support the 1979 Charney Report wild guessed ECS, revised a few years ago to the new IPCC wild guessed EPS. The Russian INM model may be the one exception. And they DO scare people.

They are actually climate confuser games — propaganda — not meaningful models of climate of this planet.

Richard Greene
Reply to  Rud Istvan
February 25, 2023 1:45 am

 The only CMIP6 model without a tropical troposphere hotspot is INM CM5. And CM5 has an ECS of 1.8, meaning no alarm

The INM ECS of +1.8 degrees C. is now below the +2.5 degree C. lower (arbitrary) limit of the IPCC’s wild guessed ECS. And no one likes Russia. So maybe the INM model will get sanctioned and thrown out of the CMIP7 models? Stranger thing have happened.

Due Diligence: My ancestors are from Russia,

Reply to  Richard Greene
February 25, 2023 5:37 am

I like Russia and Russians.

I don’t like Putin.

Mark Luhman
Reply to  Richard Greene
February 25, 2023 8:51 am

I think the Russian people have been screwed by their leadership for far to long. Yet it the Russian people who pick their leaders and their choice have been bad almost always. It also true Russia is a very dicey place to live, the climate stinks and little access to the oceans that are not frozen most of the time. In the end the Russian people are always dealt, a poor hand.

Richard Greene
Reply to  Mark Luhman
February 25, 2023 9:57 am

My family left as soon as the Communists showed up.

Reply to  Richard Greene
February 25, 2023 5:49 pm

I’m always amazed by the number of people in the US who would like to play Commissars and Kulaks, though I suppose most of them have the former role in mind for themselves.

Richard M
Reply to  Rud Istvan
February 25, 2023 12:15 pm

Nope, CO2 warms the atmosphere by absorbing energy from the surface and transferring it to other molecules just as JV stated. End of story. There is no “absence of sufficient radiative IR cooling“. That is fake science that I also believed at one time until I finally understood the process. It took me years. Sadly, JV doesn’t seem to understand the process either.

The atmosphere does not cool somewhere high in the atmosphere. It cools from all altitudes all the time. It is driven by the changes in density and not the concentration of CO2. You are not applying Kirchhoff’s Law to the process.

At equilibrium, all layers of the atmosphere exist in radiation exchange equilibrium with all other layers. No energy is exchanged. When you increase CO2, all layers of the atmosphere increase their CO2 content proportionally. Hence, radiation exchange equilibrium still holds.

What this structure implies is that higher layers can never absorb all the energy directed towards them by lower layers. There’s fewer CO2 molecules at each higher layer. Hence, some percentage of the energy radiated upwards by the lower layers has nothing to stop them from passing through and leaving the atmosphere. As a result, all altitudes radiate energy to space fairly equally.

If you disagree with this statement, then you still haven’t understood the process.

The warming in the stratosphere affects this a little, but again in a structured manner. In addition, this is an equilibrium statement. The atmosphere is almost never in equilibrium so there will always be variations due to day/night, seasons, weather, etc. However, the variations will average out and the absorption/emission of energy from CO2 molecules is always attempting to return every layer to its equilibrium state.

Most people look at the absorption of energy only and base their views on opacity without understanding how reemission affects the process. The equality of absorption and emission as required by Kirchhoff’s Law changes everything.

Richard M
Reply to  Richard M
February 25, 2023 1:15 pm

I stopped above because a base understanding is required to then understand the rest of the story. There cannot be any increase in the ERL. The only thing that occurs as CO2 levels increase is absorption of more energy in the wings of the near 15 mm frequency bands. How much warming this can produce is very complex.

Almost everyone that looks at it gets a different answer. However, from 60+ years of historic NOAA radiosonde data it appears the answer is … it doesn’t matter… the energy level is exactly what is needed to balance the cooling effect of CO2 increases.

Yes, increases in CO2 also have a cooling effect. As CO2 increases, the emissivity of the boundary layer increases. This means more low energy photons will be directed towards the surface. This does not cause any warming due to the surface and boundary layer being in thermodynamic equilibrium. However, it will lead to increases in evaporation. About 80% of the photons will strike an H2O molecule.

As many know, evaporation is a cooling effect. It takes energy from the surface and turns it into a very light weight water vapor molecule. This reduces the air density and increases convection. The water vapor is transferred by convective currents high into the atmosphere. Whenever convective currents are increased the air will be taken higher into atmosphere. Higher is colder meaning more water vapor is condensed out of the air. The end result is less water vapor is left over.

Since the water vapor greenhouse effect is saturated at low levels of the atmosphere, the only area that makes a difference is at high altitudes which is exactly where the decreases in water vapor will occur. This reduces the water vapor greenhouse effect.

The end result is that the additional energy from the wings of the CO2 absorption bands compensates for the cooling effect from CO2 increases. The overall greenhouse effect stays constant.

Reply to  Richard M
February 26, 2023 2:58 pm

What this structure implies is that higher layers can never absorb all the energy directed towards them by lower layers. There’s fewer CO2 molecules at each higher layer. Hence, some percentage of the energy radiated upwards by the lower layers has nothing to stop them from passing through and leaving the atmosphere. As a result, all altitudes radiate energy to space fairly equally.”

I would also add that all of this is a time function. The time function is sinusoidal during the day and exponential decay at night. Trying to find an “average” amount coming in and an “average” amount going out just doesn’t work very well.

More insolation from the sun (e.g. no clouds) means the exponential decay at night starts from a higher temperature than when there are clouds (or whatever) in the atmosphere. So the amount of heat radiated away is an integral of the exponential decay and sometimes it’s greater and sometimes it’s less.

The higher the temperature when the sun is no longer driving the situation the more radiation there is (T^4) headed skyward. The lower the temperature the less radiation there is. It’s really difficult to find an “average” which can be used for this process.

If you are determining the “temperature” at some point in the atmosphere based on the amount of radiation being measured it isn’t going to be some fixed amount, it isn’t even going to be some “average” amount. Parameterizing all this in a climate model just isn’t going to work very well.

February 24, 2023 2:42 pm

Something else is warming the planet and causing the increase in OLR. 

The failure here is using “averages” and “departures” or anomalies.

Is “the planet” warming when the largest increasing trend is occurring on the Greenland plateau in January?

This is NASA’s headline image for the warming from 2017 to 2021:
comment image?itok=aBc7u715

Is the Arctic really on fire as the image suggests? An increase from minus 30C to minus 20C is warming but what does it really mean?

The something else is orbital precession. It has been shifting peak solar northward for hundreds of years and the oceans are starting to respond. Arctic Ocean surface warming rapidly as the sea ice trends downward to expose more water and Southern Ocean slowly cooling as the sea ice trends upward. These are slow, long term changes in the surface condition that have a large impact on surface temperature. Particularly winter advection from warm ocean water to cool land masses.

Northern hemisphere will continue to warm until the snowfall overtakes the snow melt again.

Rud Istvan
Reply to  RickWill
February 24, 2023 2:58 pm

A comment on anomalies. They are necessary to compare global observational stuff when altitude and ocean proximity varies greatly.
But IPCC model anomalies hide the fact that model actuals (before anomalization) vary by about +/- 3C. In other words, models are awful actual predictors. Something factually illustrated in essay ‘Models all they way down’ (with footnotes) in ebook Blowing Smoke.

Reply to  Rud Istvan
February 24, 2023 8:18 pm

They are necessary to compare global observational stuff when altitude and ocean proximity varies greatly.

No – they are there to hide reality. They just amplify the noise and hide the useful information.

If you want to look at trends to get understanding then look at a location and observe the changes over periods looking at all the data. Trends are best to look at maximum and minimum months over a period of time.

The attached is part of the Greenland plateau. Produce an anomaly for this and it shows a strong warming trend. It does not inform you that the warming is predominantly occurring in January.

Comparing temperature of different things and different locations does not provide meaning. I can happily endure air temperature above 50C. But get into water at 50C and I would not last long. Same temperature but very different heat content and heat transfer capability.

The NASA image I linked above is to keep their global warming story alive. It is highly selective and based on anomalies. But it does not even show “global” warming. Some places are colder. Some places are unchanged. Even reducing this selective image to a single “global” trend removes insight.

Screen Shot 2022-11-07 at 11.10.13 am.png
Reply to  RickWill
February 25, 2023 12:30 am

Models are a way to learn about things you cannot do proper experimentation. The problem comes when people start thinking models are a good representation of reality.

As George Box said, “All models are wrong, but some are useful”.

Reply to  RickWill
February 25, 2023 4:24 am

Let me add that using averages from the very start hides so much information it is not funny. Energy is added to any point on the earth in a “pulse” consisting of something resembling a sine wave from 0 -> pi. And, that occurs day after day after day. At some point in the afternoon, as the sun sinks to the horizon, the sun’s insolation (incoming energy) falls below what the stored heat radiates and a decay function takes over. Think of a capacitor discharging.

If these are in balance over a long period of time. Then stored energy will occur, i.e., stored heat, and the earth will warm. When considering the combined mass of say the top six inches of soil, the ocean, and the atmosphere, how much heat needs to be “trapped” each day to raise the temperature of the entire earth 3 degrees over 36,500 days?

My last point is that H2O absorbs a goodly amount of the sun’s insolation at near IR frequencies. According to several sources, if a molecule absorbs at a given frequency, it also radiates at that frequency. Why do none of the Planck curve estimates of how the earth radiates show these frequencies being radiated by H2O? Is water a “special” molecule?

Reply to  RickWill
February 25, 2023 5:41 am

“The NASA image I linked above is to keep their global warming story alive.”

Yes. Temperatures are cooling globally, so NASA highlights warming in the arctic. Tricky little devils!

Reply to  RickWill
February 24, 2023 2:58 pm

This is NASA’s headline image for the warming from 2017 to 2021:

There hasn’t been any warming 2017-2021. That’s inside the new pause. What that image shows is heat being transported to the Arctic for its exit from Earth. As it happens during winter it is magnified by measuring it as anomaly. Nothing to do with orbital precession.

Reply to  Javier Vinós
February 24, 2023 3:39 pm

Nothing to do with orbital precession.

Again a lack of insight. Sure it is due to winter advection but that requires warmer oceans in lower latitudes.

The NH oceans are getting warming because the May and June solar intensity is increasing.

The warming trend in the Arctic has been occurring for at least 70 years.

The NASA image is selective to show that warming is still going on. But a 70 year trend shows the same thing.

Greenland plateau has increased almost 10C in January in those 70 years.

Reply to  RickWill
February 24, 2023 3:51 pm

This is the change in May ToA solar power flux at 40N from the Maunder Minimum to the end of this century:
 -0.400   442.409040
   -0.300   442.800035
   -0.200   443.197592
   -0.100   443.601278
    0.000   444.010670
    0.100   444.426501

Up 2W/m^2 over 500 years. Now what is CO2 supposed to do over 300 years and cause Earth to burn up.

May sunlight drives peak ocean surface temperature achieved in July.

The increasing summer solar power flux in the NH has only just started to have a noticeable impact. It will rise 20W/m^2 from current levels to its peak. There is a lot more ocean warming to come.

Reply to  RickWill
February 24, 2023 4:05 pm

The oceans are a laggard to climate change. They warm less when the planet is warming, and cool less when the planet is cooling. They are not a driver of climate change. They provide inertia to change making the planet a lot more climatically stable.

Reply to  Javier Vinós
February 24, 2023 4:46 pm

The oceans are a laggard to climate change. 

You need to take this up with Jim Hansen and the IPCC. The oceans are the poster child for global warming.
https://www.climate.gov/news-features/understanding-climate/climate-change-ocean-heat-content

Rising amounts of greenhouse gases are preventing heat radiated from Earth’s surface from escaping into space as freely as it used to. Most of the excess atmospheric heat is passed back to the ocean. As a result, upper ocean heat content has increased significantly over the past few decades.

Note the red upticks start around 1980 and it is going to hell since then.

Bob Weber
Reply to  RickWill
February 24, 2023 7:28 pm

“Most of the excess atmospheric heat is passed back to the ocean. “

Rick I suspect you realize this is what is wrong with this picture.

Reply to  Bob Weber
February 24, 2023 9:58 pm

I do but anyone who believes in the magic of the GHE altering Earth’s energy balance have demonstrated their willingness to believe in anything without the knowledge to understand it.

Reply to  Javier Vinós
February 24, 2023 5:02 pm

Yep… And this is why plate tectonics are the primary geologic control on climate change. Plate tectonics, particularly its effects on oceanic circulation patterns, drive icehouse/hothouse cycles and probably atmospheric CO2 concentrations over geologic time.

Rud Istvan
Reply to  David Middleton
February 25, 2023 1:39 pm

David, I agree. No less than famous Princeton geologist Ken Deffeyes surmised a decade ago that it was the tectonic closing of the Panama isthmus, thus altering ocean circulation, that set off the present ice ages about 2 million years ago. (The tidal rise/fall at Panama City on the Pacific side is an unbelievable 20-30 feet—I have been there and seen it.)

Bob Weber
Reply to  Javier Vinós
February 24, 2023 7:05 pm

“The oceans are a laggard to climate change….They are not a driver of climate change.”

Wrong. Your framing is IPCC framing, ie, the atmosphere is the climate.

The proper framing is (1) ocean changes are climate changes, because the ocean changes first, then atmospheric changes lag the ocean by 2 months; or (2) ocean and hence climate changes lag solar changes.

comment image

The following proof is all anyone needs to know the GHG contribution to climate change is so small as to be a negligible cause. With Stefan-Boltzmann, the sole possible GHG climate effect is via changing albedo.

After seeing this there can be no doubt that solar forcing of the ocean is 100% of the cause of climate changes since the 1880’s, as the S-B equation confirms the measured ocean change exactly, using the standard value for albedo of A=0.3 and the changes in TSI over time modeled using sunspot number.

comment image

This puts an end to several inapplicable, false but popular climate theories including ECS, TCR, effective emission height, and radiative transfer.

Reply to  Javier Vinós
February 24, 2023 6:10 pm

Agree. So many want ignore recent data, which clearly shows the pause — and very slight cooling — at least for now.

UAH.jpg
Reply to  Javier Vinós
February 25, 2023 1:35 am

Nothing to do with orbital precession

Mr. Vinós has a pet theory, and every other consideration to hell?
Can we at least agree that there are so many periodicities in this chaotic curve we call Climate, that we have not even found them all yet?
Telling each other how wrong we are is especially ignorant, seeing as we constantly find out how wrong we all were.
The image is strong supportive evidence for the precession effect, an hypothesis that will only be dis/proven once we see the Antarctic warming faster than the arctic, once the precessional angle rolls that way. Until that experiment is complete, how can you blithely say it is not due to precession?
Of course, I have as yet no reason to drop my hypothesis regarding orbital plane anomaly…

Reply to  cilo
February 25, 2023 2:59 am

Mr. Vinós has a pet theory, and every other consideration to hell?

I do not claim to have discovered anything about climate, as I am not a climate scientist and I do not carry out climate research. I just apply the scientific method and point to what the evidence supports and what it doesn’t, and look for alternative explanations to the evidence.

Precession has an effect on climate, obviously. And for some things, like the position of the ITCZ and the mid-Holocene transition it is the dominant cause. For the glacial cycle Milutin Milankovitch said no, and the evidence says no. The glacial cycle is driven by obliquity and the evidence is pretty strong. It is laid out in the second chapter of my book, and it is not my contribution. Great glacial cycle researchers like Peter Huybers see the same as I see in the evidence, and they preceded me, so no claims on my part, only my agreement.

I prefer my first name, but if you are going to use my last name preceded by treatment, that would be Dr., thanks.

Reply to  Javier Vinós
February 25, 2023 8:20 am

Agree glacial cycles driven more by obliquity not precession. Carl Wunsch has shown this to be statistically true.

Reply to  Javier Vinós
February 26, 2023 12:01 am

Once again the good doctor reads what he wants to understand? We were discussing hemispheric asymmetry, but I guess changing the subject is doctorate-level dialogue protocol…

Reply to  cilo
February 27, 2023 2:46 am

Hemispheric symmetry is a problem for the precession hypothesis, as the world goes into glaciation and comes out as a whole according to benthic cores. One hemisphere might take the lead, but one hemisphere is always doing the opposite it should be doing if precession ruled the glacial cycle.

Reply to  Javier Vinós
February 27, 2023 11:26 am

Ah, but good sir, I never said anything about glaciation. I have not actually spent any thought on the subject, and I have yet to hear a coherent, simple (not simplistic) and convincing theory on the subject.
None
But I bet obliquity COUPLED WITH precession sure explains most every instance of localised disastrous climatic orneriness.

Stephen Wilde
February 24, 2023 2:46 pm

Convection adjusts in order to neutralise the potential thermal effect from radiative molecules in the atmosphere.
Convection itself is the cause of the greenhouse effect and not radiative molecules.
An atmosphere that has no radiative materials at all will still have a lapse rate so that convection is inevitable and overall surface warming will occur.
The surface temperature of surfaces beneath atmospheres is raised by atmospheric mass motion and not by back radiation.

Reply to  Stephen Wilde
February 24, 2023 3:00 pm

Convection itself is the cause of the greenhouse effect and not radiative molecules.

The greenhouse is by definition a radiative effect. You cannot redefine it and keep the same name.

Convection is a small player in the way the energy delivered by the Sun finds its way to space.

Rud Istvan
Reply to  Javier Vinós
February 24, 2023 3:28 pm

A small caveat. The Lindzen adaptive iris is Convection driven (Tstorms) but most definitely affects radiative cooling via its cirrus impact. (More cirrus warms since ice crystal cirrus is transparent to incoming short wave solar but opaque to outgoing IR; less cirrus cools).

Philip Mulholland
Reply to  Javier Vinós
February 24, 2023 3:37 pm

The greenhouse is by definition a radiative effect. You cannot redefine it and keep the same name.

Then call it an Atmospheric Thermal Effect instead.

Convection is a small player in the way the energy delivered by the Sun finds its way to space.

The meridional advection of energy from the tropics (surplus) to the poles (deficit) is absolutely due to convection and fundamental to all of the weather processes in the atmosphere. Advection is part of the process of convection.

Reply to  Philip Mulholland
February 24, 2023 3:56 pm

The meridional advection of energy from the tropics (surplus) to the poles (deficit) is absolutely due to convection

Yes, convection is very important for meridional transport, as it is latent heat transport. Both have about the same magnitude, as far as I know. And let’s not forget potential energy transport, as air parcels rise in the tropics and sink at higher latitudes. Energy is key to climate change.

Philip Mulholland
Reply to  Javier Vinós
February 24, 2023 4:10 pm

Energy is key to climate change.

I agree, and the focus on thermal radiant opacity as the overarching mechanism that impedes energy flux thru the atmosphere is just nonsense. Adiabatic convection transmits energy and is a non-radiative process.

Philip Mulholland
Reply to  Philip Mulholland
February 25, 2023 1:39 am

Javier,
The greenhouse gas concept is a narrative based on models that are sourced in conjecture.

As with all narratives when the story fails to match the data the story adjusts, so we have the switch from “back-radiation heating” to “reduced surface cooling by insulation.” The insulation back-stop story fails to acknowledge that the primary reason for the greenhouse gas conjecture is to raise the planetary surface temperature in the first place. The Greenhouse Gas Effect (GCE) concept is based on a model that creates a low average planetary surface temperature. It is not based on primary data, it is instead based on derived data (the warm planetary surface) and so in essence the GHE concept relies on a form of inverse modelling.

Stephen Wilde and I are currently studying the atmosphere of Mars and applying our DAET modelling concept to MY29 annual atmospheric profile data.
Here is a brief synopsis of what we have found thus far:
1.     The areal weighted global annual average air temperature at the 636 Pascal average surface pressure of Mars is 211.8 Kelvin.
2.     The black body (ε =1) Vacuum Planet Equation thermal emission temperature for Mars is 209.8 Kelvin. Therefore, the Atmospheric Thermal Effect (ATE) for Mars is +2 Kelvin.
3.     The grey body (ε =0.87) Vacuum Planet Equation surface thermal emission temperature for Mars is 202.5 Kelvin. Therefore, the Greenhouse Gas Effect (GHE) for Mars is +9.3 Kelvin. (We are already deep in the weeds of moveable definitions based on parameter variations).
4.     In a comparison with the similar gas content atmosphere of Venus, we find that the thermal emission pressure level for Venus is 1,868 Pascal. This pressure is higher than the surface pressure of Mars and is in the radiatively transparent stratosphere of Venus at an elevation of 71.06 Km.
5.     Therefore the low pressure Carbon Dioxide gas  atmosphere of Mars is thermally radiatively transparent with no Greenhouse Gas Effect possible. The surface Atmospheric Widow for low pressure Mars is completely open at night.
6.     The only possible mechanisms that can explain the +2 Kelvin ATE on Mars are dust haze absorption of insolation during the day and adiabatic convection that accounts for the surface diurnal temperature change.

The model driven greenhouse gas conjecture is not based on data and therefore is not Science. It must be discarded for the moveable feast of nonsense that it so obviously is.

Reply to  Javier Vinós
February 24, 2023 4:11 pm

Convection is a small player in the way the energy delivered by the Sun finds its way to space.

The large difference between the radiating temperature and surface temperature clearly demonstrates the absurdity of that statement.

Heat transport from the surface or even the lower atmosphere to where it is released in the troposphere primarily involves mass transport. Just 1kg of ice in the entire atmospheric column will absorb 95% of incident OLR. That heat input is going to cause convection. Have you ever looked at how clouds move?

Just the 522.7E15kg of water that cycles through the atmosphere every year could hardly be claimed as “small”. And that component is only part of the convective processes.

Reply to  RickWill
February 24, 2023 4:26 pm

That statement I made is supported by an ample bibliography. For example, NASA gives for convection a value of 5% of the absorbed solar shortwave energy, or about 18 Wm-2
https://earthobservatory.nasa.gov/features/EnergyBalance

What sort of bibliography supports your statement that “Convection itself is the cause of the greenhouse effect”?

Stephen Wilde
Reply to  Javier Vinós
February 24, 2023 6:06 pm

Well, you say it is not CO2 but something else
That something else is convection.
If it isn’t CO2 then it isn’t any other radiative molecules either.
Proposing that convection is the cause accounts for your observation.
It takes time for KE to become PE during uplift and PE to become KE again in descent. Delay in the loss of solar energy to space causes warming.
The absence of a bibliography is due to the fact that it never needed to be spelled out until the radiative theory came to the fore.
You just have to consider an atmosphere that is wholly non radiative. You still have a lapse rate, convection and surface warming.
The atmosphere can never become isothermal due to the lapse rate and uneven surface temperature distribution inevitably causing convection.
Convection requires energy to drive it and that energy is locked in as PE which cannot be radiated as heat because it is not thermal energy. The process of locking that energy into convective overturning denies it to space so the temperature must rise.
Very simple really.
Also accounts for the observation that the temperature within the Venusian atmosphere at the same pressure as the pressure at the surface of Earth is much the same as that on the surface of Earth subject only to an adjustment for distance from the sun.
The radiative theory cannot account for that.

Reply to  Stephen Wilde
February 25, 2023 12:37 am

Alternative explanations for climate change are like different religions, mutually exclusive. Proposing one means one is attacked by all the others.

Convention does not solve the problem of the change in energy fluxes taking place at the top of the atmosphere needed to drive climate change, it just moves the pieces within the climate system.

Stephen Wilde
Reply to  Javier Vinós
February 25, 2023 10:41 am

When an atmosphere is forming OLR drops for as long as PE is being added to the convective overturning system. Therefore convection does solve the issue of a change in energy fluxes taking place at the top of the atmosphere.
If something were to cause an increased rate of loss of atmospheric mass to space then there would be an increase in OLR above incoming from the sun until the process stops or the atmosphere is gone.
In reality there are periods of excess OLR and periods of a deficit in OLR since convection takes a little while to neutralise destabilising influences such as a change in GHGs.
I’m not attacking your post. Just supplementing it with a mechanism that explains your observations.
Overall, convection varies in order to always match the upward pressure gradient force with the downward force of gravity.
GHGs do not change global temperature but on Earth they do have a tiny effect on the global pattern of convective overturning.
Indiscernible compared to natural variability from sun and oceans.
The same for every other planet or moon with an atmosphere.

Reply to  Javier Vinós
February 24, 2023 6:11 pm

NASA gives for convection a value of 5% of the absorbed solar shortwave energy, or about 18 Wm-2

Wow such poor understanding. You are confusing the power to drive convection with the transport of energy. It is a reasonably efficient heat engine. All that movement taking just 18W/m^2. But that has nothing to do with the heat transport that that engine achieves.

The water cycle alone corresponds to about 80W/m^2. All the atmospheric water involves mass transport of heat. All advection is mass transport of heat.

Since you are so ready to accept NASA climate phiisics, I will use their image:
comment image

They have 86.4W/m^2 as latent heat, 18.4W/m^2 as conduction 358.2W/m^2 of their surface OLR flux being reabsorbed before exiting so that component is involved in convection.

Reply to  RickWill
February 25, 2023 12:39 am

Evaporation and latent heat transport to the place where condensation takes place is a different process from convection, that’s why each one has its arrow.

Stephen Wilde
Reply to  Javier Vinós
February 25, 2023 11:07 am

Evaporation causes convection because water vapour is lighter than air and therefore involves latent heat transport (KE to PE). They have separate arrows because dry convection also creates latent heat transport (KE to PE ).
What is missing is the descent phase whereby PE gets converted back to KE.
The lapse rate for descending dry air is steeper than that for rising moist air and that compensates for moist air being lighter than dry air. It is a marvellous self balancing system.
I have long pointed out that accounting for the descent phase is a material omission.
What goes up must come down so KE from PE under descending air always matches PE from KE within rising air.
That conversion process delays energy loss to space and the surface temperature rises as a consequence.
For a non-rotating planet there would be KE to PE on the lit side and PE to KE on the dark side but for a rotating planet the Coriolis force jumbles it up across both lit and unlit sides in what we see as the Polar, Ferrel and Hadley cells which create our permanent climate zones.
In the face of that underlying balancing mechanism we would never be able to observe our contribution in the face of natural internal system variability.
I am adding to your work, not detracting from it.

Reply to  Javier Vinós
February 25, 2023 2:39 pm

Evaporation and latent heat transport to the place where condensation takes place is a different process from convection, 

This makes it clear you have no idea at all. The definition of conc=vection is:

Heat transfer in a gas or liquid by the circulation of currents from one region to another.

Convection does not distinguish between latent heat and sensible heat. It is all mass transport of heat. It excludes radiation transfer. But the heat input from any radiation is redistributed by convection.

Philip Mulholland
Reply to  Javier Vinós
February 24, 2023 6:13 pm

What sort of bibliography supports your statement that “Convection itself is the cause of the greenhouse effect”?

We suggest here that it is not a case of either / or, rather it is both opacity and convection that are involved.

Reply to  Philip Mulholland
February 25, 2023 12:45 am

The atmosphere increases its temperature mainly through four processes. In order of importance, they are evaporation, solar IR absorption, surface IR absorption, and convection.

So the vast majority of atmosphere heat gain is due to water vapor and clouds, followed by CO2, and a final contribution by convection.

Bob Weber
Reply to  Javier Vinós
February 25, 2023 3:41 am

“So the vast majority of atmosphere heat gain is due to water vapor and clouds, followed by CO2, and a final contribution by convection.”

Wrong again. Ocean heat itself drives the atmosphere temperature via convection; water vapor, clouds, & CO2 also due to ocean heat.

comment image

Stephen Wilde
Reply to  Javier Vinós
February 25, 2023 10:55 am

Evaporation causes convection because water vapour is lighter than air.
Solar IR absorption causes convection because denser lower gases absorb more than less dense higher gases.
Surface IR absorption causes most convection of all because the surface is lower and denser than any part of the atmosphere.
So, convection is the unacknowledged link underpinning all of them.
No convection, no greenhouse effect with an isothermal static atmosphere radiating to space at the temperature of the top.
In reality that cannot happen because the topmost molecules would have KE plus PE as total energy content way in excess of the downward force of gravity so the atmoshere would be incrementally lost to space.
You have to have increasing PE with height matching a decline in KE with height to keep the downward force of gravity balanced with the upward pressure gradient force.
That is all long established basic thermodynamics but it has been completely ignored.

Reply to  Javier Vinós
February 25, 2023 2:43 pm

and a final contribution by convection.

Such poor understanding. Convection moves latent heat and sensible heat.

By what process does water go from the surface to condense in the upper atmosphere if it is not convection?

You need to get a clue.

Reply to  Javier Vinós
February 25, 2023 5:33 pm

and a final contribution by convection.

How did the water get from the surface to mid troposphere to form rain?

Do you even know what convection means?

It is difficult to convey meaning on points with someone who has so little understanding of the meaning of words.

Reply to  RickWill
February 27, 2023 2:41 am

There are three types of heat transport. Latent, sensible, and potential. Some people group sensible and potential into dry-static heat transport. They are completely separable in meridional transport. For example, in the Hadley cell, latent heat transport is equatorward (this creates deserts at 30º latitude) and dry-static heat transport is poleward. The correct way of analyzing heat transport is by separating these components to see their respective contribution. By mixing them you get a poorer understanding of climatic processes.

Kevin Kilty
February 24, 2023 3:08 pm

Looking for a silver-bullet that exonerates CO2 is a thesis in Willis Eschenbach’s post “A serious question” from last September. Howard Hayden’s attempt is summarized from time to time of the weekly energy and environment roundup from SEEP.

I am not sure what Javier says here works except in the case of a significant step change in CO2. A step change large enough to not be indoubt in any way.

If the change in CO2 is more like a slow ramp, which is closer to reality than an obvious step change, which is what is discussed here, then what occurs is the following:

1) At the beginning of the ramp there is an infinitesimal drop in OLR at TOA, but OLR soon begins to rise through the entire atmospheric column as its temperature adjusts to re-establish energy balance at TOA.

2)The next increment of CO2 repeats this process over and over as CO2 rises to 2x concentration. The effect is like a delayed ramp in OLR that is a bit behind the ramp in CO2 — the process is trying to achieve an energy balance but is always out of balance.

3) I doubt one could measure the OLR to the requisite accuracy to establish its ramp-like quality, and then there would be further problems with attribution of the OLR signal.

I am not saying that the actual rise in temperature, adjustments in OLR, and measurments of CO2 concentration are immaterial to the problem, but that it is exceptionally difficult to make an iron-clad argument beyond any doubt in a situation with as much noise, contribution from other factors, and auxillary assumptions.

Reply to  Kevin Kilty
February 24, 2023 3:22 pm

Why would OLR increase if the warming is caused by an increase in greenhouse gases? An OLR increase is contrary to warming by GHGs, and indeed it is contrary to warming, as it requires an even bigger increase in absorbed solar radiation to cause warming. And an increase in GHGs does not, cannot act through an increase in shortwave absorbed radiation.

When the evidence is contrary to the theory, it is the theory that is wrong.

Kevin Kilty
Reply to  Javier Vinós
February 24, 2023 5:51 pm

Because increasing CO2 causes initially a drop in OLR, but then OLR has to rise to re-establish equilibrium through a rising atmospheric temperature. You said so yourself.

You can show this using MODTRAN.

Kevin Kilty
Reply to  Kevin Kilty
February 24, 2023 6:26 pm

Let me try to explain this another way. Here is what you have said.

So, if CO2 is responsible for the surface temperature increase, we should first expect less OLR and then the same OLR. If at any time we detect more OLR…

To go from less OLR to the same Ol’ OLR then OLR has to rise at the TOA. You will observe a rising OLR during the attempt to re-establish energy balance. Now, since you don’t know exactly what the “same OLR” is numerically, how can you possibly say that this rising OLR is inconsistent with CO2 being the cause? It might be part of the return to equilibrium…especially if equilibrium is established through a number of other processes over a long time.

Reply to  Kevin Kilty
February 25, 2023 1:06 am

CO2 goes up and down in its annual cycle, so let’s consider interannual when it only goes up. We are following here IPCC reasoning as an exercise about how things work in their view.

From one year to the next CO2 increases by about 3 ppm. This causes a minute increase in atmospheric opacity to IR. This should cause some small warming that is not noticeable but adds up over the years. That small warming comes from a reduction in OLR not an increase in ASR. As soon as the warming takes place (same year) OLR goes back to where it was because increasing the opacity of the atmosphere does not increase OLR.

Now considering many years, all the internal variability is gone, CO2 has increased, the warming has accumulated, and OLR has remained the same.

In that IPCC scenario, OLR cannot increase. If it increases that increase is from warming not produced by CO2.

Imagine an Earth without an atmosphere but with the same albedo. OLR would still have the same energy, only it would come from a half surface at scorching hot temperatures and the other half at very cold temperatures as in the Moon.

You cannot change the OLR of a planet except in the short term without changing its ASR and/or its RSR (albedo). If OLR is increasing is because ASR is increasing and/or RSR is decreasing. None of those changes can be attributed to CO2. An RSR decrease could be attributed to water vapor. Perhaps we need to increase our water vapor emissions 😉

Richard M
Reply to  Kevin Kilty
February 25, 2023 2:25 pm

MODTRAN is only part of the picture. It doesn’t show what else is happening (water vapor and convection) which just happens to adjust keep OLR constant.

Kevin Kilty
Reply to  Richard M
February 25, 2023 3:14 pm

Admittedly MODTRAN has its limitations, but it is instructive to use MODTRAN with any atmosphere model available and double CO2. One will see a reduction in OLR looking down at TOA in the neighborhood of 4Wm^{-2} and an increase looking up near the surface. The surface increase causes surface temperature to rise.

Start looking at water vapor, convection, reduction of snow and ice, changes in albedo, etc. and the return to equilibrium might take who knows how long. Javier presents a graph with a rise in OLR of a couple of watts per square meter over a couple of decades — I don’t think this observation invalidates theory.

Richard M
Reply to  Kevin Kilty
February 26, 2023 5:40 pm

Yes, you will see part of the full picture. You see what looks like warming because you don’t follow the energy flow all the way. In fact, if you keep the humidity constant, you will be eliminating the actual feedback that compensates for the warming. IOW, you will only fool yourself.

Reply to  Kevin Kilty
February 24, 2023 3:35 pm

‘Howard Hayden’s attempt is summarized from time to time of the weekly energy and environment roundup from SEEP.’

Here’s Hayden’s differentiation between the responses to surface heating by other sources vs. heating due to increased greenhouse effect:

http://www.sepp.org/science_papers/Climate%20Physics%2010.pdf

“The Planck Response, however, does have some validity. Imagine that somebody sprinkles the right kind of Pixie Dust all over the earth so that the surface warms up. It will radiate more IR and set up an imbalance so that the heat emitted to space (ca. 60% of the surface radiation) will exceed the absorbed solar heat Iout > Iin ). The imbalance will continue (and diminish) until the earth cools down to the condition before the Pixiedust was applied. This is indeed a negative feedback mechanism that tends to hold the surface temperature constant, but it most assuredly does not determine what that temperature is. In particular, it is of no use in calculating the Equilibrium Climate Sensitivity (ECS, the temperature rise due to CO2 doubling when Iout = Iin ).

If the greenhouse effect increases, such as by increasing atmospheric CO2 or H2O, then the IR emission to outerspace is decreased. That imbalance ( Iout < Iin ) warms the surface until the equality between incoming solar heat and outgoing heat radiation is re-established. (Climate modelers take note: During this time, the warming planet radiates less IR to space than when it was cooler.) In this realistic case, the increase in the greenhouse effect occurs before the temperature increase, unlike the Pixie-Dust scenario. It is important to remember that the sole source of heat to the earth is sunlight. Importantly, when the Planetary Heat Balance is restored — that is, when Iout =Iin = (Isun/4)*(1 α) the additional greenhouse effect (“radiative forcing”) must equal the additional surface radiation unless there is a change in either Isun or albedo α. 

Kevin Kilty
Reply to  Frank from NoVA
February 24, 2023 6:27 pm

If the greenhouse effect increases, such as by increasing atmospheric CO2 or H2O, then the IR emission to outerspace is decreased. That imbalance ( Iout < Iin ) warms the surface until the equality between incoming solar heat and outgoing heat radiation is re-established.

Exactly what I said above. There is a time period in which OLR at TOA is rising as energy equilibrium is being re-established. Thus, a disturbance in this case caused entirely by step CO2 leads to an observation of rising OLR (at TOA). Observation of a rising OLR does not exonerate CO2 in the process.

I refuse to discuss Pixie dust of the right kind or any other.

Reply to  Kevin Kilty
February 24, 2023 7:46 pm

‘I refuse to discuss Pixie dust of the right kind or any other.’

Fair enough.

‘Observation of a rising OLR does not exonerate CO2 in the process.’

By itself, I don’t think so. Both Vinos and Hayden indicate there has to be surface warming. And then there’s an implicit condition of whether or not the rise in OLR to ‘equilibrium’ was first preceded by a decrease in OLR. If there was an initial decrease in OLR, then I assume it would be GHE, otherwise it would be a non-GHE, e.g. a change in albedo. At least that’s my interpretation.

Kevin Kilty
Reply to  Frank from NoVA
February 25, 2023 3:18 pm

What you say here is precisely my concern. In order for an observation to invalidate theory, one has to have a better grasp on what the observation implies. It’s too bad we can’t just organize a big step increase in CO2 and then observe its effect. This puny 1% per year doesn’t produce anything convincing.

real bob boder
Reply to  Kevin Kilty
February 26, 2023 6:51 am

Except CO2 continues to rise and equilibrium should not have been achieved and per Nick and his ilk won’t be reached for 50 years after CO2 levels have stopped increasing. So you are talking pixie dust now.

observa
February 24, 2023 3:28 pm

I suspect the doomsters are working on the problem-
The First Law of Thermodynamics Has Been Rewritten (msn.com)
You just have to contextualise these matters.

Kevin Kilty
Reply to  observa
February 24, 2023 6:32 pm

From the article:

 The key is that the total amount of energy causing the balloon to expand and the gas to get hotter is the same as the amount of heat you put into the balloon.”

This is not the first law because they have left out the work that may have gone in or out. As the balloon expands it does work. I have doubts about their doubts…

Richard Greene
Reply to  Kevin Kilty
February 25, 2023 2:05 am

I took a thermodynamics course in the 1970s on my way to a BS degree. But everything has changed since then — all the laws seem to have changed, due to climate change, I suppose?

This comment is serious, not satire.

Kevin Kilty
Reply to  Richard Greene
February 25, 2023 3:21 pm

Well I had been teaching thermodynamics right up my retirement recently, and it has not changed since the 1970s I assure you. However, the climate kooks seem to think it doesn’t mean much, and the topic sure produces some heated discussion here.

Reply to  Kevin Kilty
March 8, 2023 5:28 pm

It sure does. If only there were a way to harness all this heated discussion… hmm…

Philip Mulholland
February 24, 2023 3:40 pm

Javier,
The thing that causes me endless bemusement is the total focus on polyatomic molecular gases in the atmospheric study of thermal radiant flux. As all competent physicists know the key linkage between energy of mass motion and electromagnetic energy is shear wave flexure. A solid has to be able to flex in order for the coupling that permits energy of motion to become radiant energy to occur. Thermal radiant opacity requires a gas molecule to have three or more atoms because it is impossible to flex diatomic molecules (oxygen, nitrogen, hydrogen etc.).

But all this focus on polyatomic molecular gases completely ignores the role of solid particles (dust haze and ice fog) as efficient absorbers and emitters of thermal radiant energy throughout the body of the atmosphere. The freezing point of supercooled water and the consequent formation of cirrus ice particle clouds is a process that is governed by the environmental lapse rate.  The environmental lapse rate is not fixed and so enhanced surface temperatures tend to create convection that automatically removes energy from the surface and transmits this energy to the tropopause via an adjusted environmental lapse rate. At the tropopause radiant energy loss to space is facilitated both by the presence of ice particles with their efficient flexural coupling and by the thermal radiant transparency of the overlying stratosphere.

The simple stepwise sliding model in Figure 1 with a constant adiabatic lapse rate is just a conjecture.

Philip Mulholland
Reply to  Philip Mulholland
February 24, 2023 3:51 pm

The emissivity coefficient of ice ranges from 0.96 to 0.99.1 This means that ice is highly effective in emitting energy as thermal radiation, which includes both visible radiation (light) and infrared radiation.2
Sources
1
engineeringtoolbox.com
2
en.wikipedia.org

Reply to  Philip Mulholland
February 24, 2023 4:01 pm

The simple stepwise sliding model in Figure 1 with a constant adiabatic lapse rate is just a conjecture.

Of course. The entire CO2 hypothesis is just a conjecture based on very little evidence. The way a problem is framed limits the possible solutions to the problem that can be found.

Nevertheless, the way the problem has been solved, the warming caused by increasing the greenhouse effect should not result in an increase in OLR. Changes to the lapse rate just modify the amount of warming produced, but should not increase OLR either. From space, the warming at the surface should not be “seen.” That’s what an increase in opacity does.

Reply to  Javier Vinós
February 24, 2023 4:32 pm

Javier,

I like the way you set up the issue. The main problem I see is that the Derwitte and Clerbaux 2018 chart only shows a few W/m^2 increase over ~30 years. Of course, had it been a decrease, we’d never hear the end of it from the Alarmist camp.

Reply to  Frank from NoVA
February 24, 2023 4:49 pm

chart only shows a few W/m^2 increase over ~30 years.

A few Wm-2 is a lot. The radiative forcing from a doubling of CO2 is 2-4 Wm-2, so the entire radiative forcing from CO2 (including feedbacks) since preindustrial is maybe 1.5-2.5 Wm-2, and all the warming since is supposed to come from just that.

Reply to  Javier Vinós
February 24, 2023 5:58 pm

Yes, if we were rabid Alarmists, we could talk in terms of ‘Hiroshimas’.

Dave Andrews
Reply to  Frank from NoVA
February 25, 2023 7:48 am

No lets go full rabid alarmist and talk in terms of Castle Bravos (the most powerful nuclear device ever detonated by the US at Bikini and 1000 times more powerful than Hiroshima)

That should really make people worried. 🙂

February 24, 2023 4:07 pm

Javier Vinós, thank you for the clearly stated points you have made.

“Part of it [IR radiation – dd] still takes place from the surface through the atmospheric window, but most of it takes place from higher in the atmosphere.”

It is good to have direct observations, even if not the entire planet, to assess what happens at the edge of the atmospheric window in the IR spectrum. We have such a platform: The NOAA geostationary satellite GOES East. Band 16 is centered at a wavelength of 13.3 microns. NOAA calls this the CO2 band. The full disk view provides 2 km resolution. The brightness temperature color scale used for the visualizations is such that the radiance at 30C (yellow) is 10 times the radiance at -90C (white.)

So what? The static theory of the greenhouse effect is not wrong, but the atmosphere is not static. The near-real-time animations show us that the planet is a huge array of highly variable emitter elements. The formation and dissipation of clouds has a lot to do with this, and the motion is what drives it all. The overall result is the composite of a huge range of highly variable emitter outputs. This link activates a 2-hour series of images to get the point. You can select longer time periods.

https://www.star.nesdis.noaa.gov/GOES/fulldisk_band.php?sat=G16&band=16&length=12

“CO2 is innocent. Its fingerprint is not found at the crime scene.”

I agree, and I can “see” from space that the atmosphere is performing far more powerfully as the compressible working fluid of its own heat engine operation, than as a static radiative absorbing and emitting layer. This is not a recent discovery. As I see it, the “fingerprint” of the incremental static warming effect of rising CO2 concentrations should never have been expected to be detected for reliable attribution by any means we have available to us. In other words, the claim that warming of land and oceans is being caused by non-condensing GHGs has been unsound from the beginning and remains so.

Reply to  David Dibbell
February 24, 2023 4:59 pm

the claim that warming of land and oceans is being caused by non-condensing GHGs has been unsound from the beginning and remains so.

And yet the author is trying to defend that such a process does exist. Even when he concludes it does nothing to the energy balance.

The existence of a GHE having some involvement in Eartth’s energy balance is a belief system that is well entrenched. It is the common wisdom. Even sound evidence that it does nothing has not altered the view that such a process exists. If there is a defined process that is supposed to do something but the evidence shows it does nothing, does the process exist?

Reply to  RickWill
February 25, 2023 6:15 am

 “If there is a defined process that is supposed to do something but the evidence shows it does nothing, does the process exist?”

I separate this into two different questions:

1.) Does the atmosphere absorb and emit IR energy, and are the instrument-confirmed absorption and emission properties of water vapor, CO2, other GHGs, and clouds responsible for this finding?

Sure. I see no reason to dispute this concept and I am not bothered by this author’s treatment of it. But it is incomplete, if one wishes to know what to expect to “see” from space and what to expect to happen at the surface.

2.) So as the surface looks toward space, will the incrementally stronger static radiative coupling from increased concentration of CO2 and other non-condensing GHGs cause heat energy to be accumulated on land and in the oceans?

I don’t see how such a cause-and-effect claim can be supported by observations. On the contrary, both the gridded hourly CERES outgoing LW/SW data and the near-real-time GOES data show the circulating atmosphere (including clouds) to be performing as a highly active controller of emission and reflection. And if so, one cannot reliably isolate the incremental static greenhouse effect as a cause of land or ocean warming. I conclude that the incremental energy involved is readily transferred to the working fluid of the heat engine for circulation to whatever altitudes and at whatever mass flow rates result from the physical response to absorbed and stored energy.

AGW is Not Science
Reply to  RickWill
February 25, 2023 8:30 pm

Not according to science. But clearly what they’re doing is not science, or the hypothesis would have been heavily modified or discarded long ago.

Reply to  David Dibbell
February 25, 2023 6:27 am

Let me clarify my response to the “CO2 is innocent” statement. In the U.S., the court does not declare a defendant “innocent.” Rather, a verdict of “not guilty” is returned when reasonable doubt remains after considering the evidence. In this case, the way the atmosphere operates provides reasons to doubt that CO2 is to blame for the reported warming. The verdict is Not Guilty.

Chris Hanley
February 24, 2023 4:15 pm

CO2 is innocent. Its fingerprint is not found at the crime scene. Something else is warming the planet and causing the increase in OLR

That is probably ‘tongue-in-cheek’ of course there is no crime scene, whether the increasing atmospheric CO2 concentration is partly or mostly responsible for the global warming since 1950 as per IPCC or not there is no sound evidence that any harm has resulted quite the opposite.

Nick Stokes
February 24, 2023 4:42 pm

“And the test results can be evaluated for example with Derwitte and Clerbaux 2018:”

But can that paper be relied on? It appears in a MDPI pay-to-play journal. Not much peer review:
“Received: 17 September 2018; Accepted: 21 September 2018; Published: 25 September 2018”

Here is a more recent paper in Nature, which gets plenty of re viewing.
” In particular, the increase of atmospheric CO2 and CH4 led to significant negative trends in the SOLR of −0.05 to −0.3% per year in the spectral region corresponding to the ν2 and the ν3 CO2 and in the ν4 CH4 band. Most of the trends associated with the natural variability of the OLR can be related to the El Niño/Southern Oscillation activity and its teleconnections in the studied period.”

FWIW, Dewitte et al in a more recent paper, same journal gives this plot allowing for RSR – reflected solar radiation

comment image

There is less energy outgoing than incoming, as predicted by the GHE.

Reply to  Nick Stokes
February 24, 2023 6:17 pm

Reflected? I think the criteria is LW out vs SW absorbed….

Nick Stokes
Reply to  Frank from NoVA
February 24, 2023 9:37 pm

Well, this is Dewitte, who is the sole person cited by Javier as having exonerated CO2. And that is how he adds it up.

Reply to  Nick Stokes
February 24, 2023 6:24 pm

There is less energy outgoing than incoming, as predicted by the GHE.

The thermalised solar is calibrated from ocean heat content. So the difference on your chart is not based on accurate radiative measurement but of temperature measurements in the oceans.

Oceans are retaining more heat because the water cycle is slowing down. Less evaporation means the thermocline shoals and the oceans retain more heat. It has nothing to do with the GHE. That is a fairy tale that has fitted the political agenda and it gets reinforced by the rivers of funding to government funded enterprises. .

Reply to  Nick Stokes
February 25, 2023 2:10 am

There has to be less outgoing than incoming for warming to take place. You cannot have warming if outgoing is more than incoming. That has nothing to do with GHE.

bdgwx
Reply to  Javier Vinós
February 25, 2023 7:19 am

True. But that also happens when ΔEout < ΔEin. Note that the GHE hypothesis is that ΔEout > 0 simultaneous with ΔEout < Ein. The Dewitte & Clerbaux 2018 publication is consistent with the GHE hypothesis. If you are wanting to challenge the GHE hypothesis I don’t recommend doing so by citing the Dewitte & Clerbaux 2018 publication.

Reply to  bdgwx
February 25, 2023 10:01 am

Where does the ΔEin come from in the GHE?

bdgwx
Reply to  Javier Vinós
February 25, 2023 11:27 am

Dewitte & Clerbaux 2018 say the OLR observations indicate “longwave cloud thinning” as described in [Cess et al. 1990]. In fact, they claim models are underestimating this feedback. The snow/ice albedo feedback would increase OLR as well though the declines as of yet would not be enough to explain the observed OLR trend.

Reply to  bdgwx
February 25, 2023 5:45 pm

Cloud thinning is not mentioned in Cess et al. 1990. What they say is:

A roughly threefold variation in one measure of global climate sensitivity is found among the 19 models. The important conclusion is that most of this variation is attributable to differences in the models’ depiction of cloud feedback, a result that emphasizes the need for improvements in the treatment of clouds in these models if they are ultimately to be used as reliable climate predictors

Not very encouraging that we know what happens to clouds.

You keep talking about feedbacks, yet feedbacks are not part of the GHE theory. We could be discussing feedbacks for hours and never reach an agreement because even models don’t agree about feedbacks.

bdgwx
Reply to  Javier Vinós
February 26, 2023 6:12 am

Dewitte & Clerbaux call it “longwave cloud thinning”. Cess et al. just call it “cloud feedback”.

I keep talking about feedbacks because that’s what scientists have been talking about since the 1800’s.

Editor
Reply to  Nick Stokes
February 25, 2023 5:03 am

Nick,
Your plot starts in 2001, after global warming stopped. It isn’t that different from Javiers, for the period you plotted. Looks cherry-picked to me.

bdgwx
Reply to  Andy May
February 25, 2023 6:32 am

Global warming didn’t stop in 2001. [Schuckmann et al. 2020] [Cheng et al. 2023]

Editor
Reply to  bdgwx
February 25, 2023 9:36 am

bdgwx,
Clearly a matter of opinion. Running a 5-year moving average on UAH global LT data as a poor man’s method of removing ENSO effects, shows that it certainly stopped from around 1997 to 2013 (see attached). The 2015-2016 super El Nino has raised temperatures from 2015 until 2020, but El Nino warming is temporary, since it moves heat from the ocean compartment to the atmosphere compartment where it can be more easily ejected to space. Thus, longer term, a super El Nino has a cooling effect. We will see what happens in the next few years. My prediction would flat or cooling temperatures.

5-year average LT.png
bdgwx
Reply to  Andy May
February 25, 2023 11:34 am

I don’t know how well TLT temperatures can assess global warming in this context since it only represents 1% of the system. Assuming it can for the sake of this post and using the Monckton method we find that UAH TLT shows 0.31 C of warming (+0.14 C/decade) since 2001.

Editor
Reply to  bdgwx
February 25, 2023 5:51 pm

TLT represents much more than the lower 2 meters of the atmosphere, but a linear fit is completely inappropriate for this series and quite meaningless.

bdgwx
Reply to  Andy May
February 26, 2023 6:10 am

Everybody knows TLT represents the LT layer of the atmosphere. That’s why the product is called is called TLT and not 2mT.

Perhaps you can you tell Monckton what he is doing is inappropriate and meaningless then.

Nick Stokes
Reply to  Andy May
February 25, 2023 12:12 pm

Andy
Your plot starts in 2001″
It isn’t my plot. It is a plot from Javier’s hero, Dewitte, writing just a few months later. He isn’t my hero, but I include it to show that the way he does the arithmetic, there is still a net outflow of heat. It was the claimed (by Javier) absence of outflow of heat that “exonerated CO2”.


Reply to  Nick Stokes
February 25, 2023 5:54 pm

I have no heroes, but if I had them you wouldn’t be one. Let’s start by interpreting graphs correctly. The graph you posted says “total outgoing.” That includes OLR and RSR (albedo) and does not say anything about their relative contribution.

What does that graph have to do with my post? I’m a bit lost here. For the warming to happen Ei > Eo. Everybody agrees on that and it is what the graph says.

Nick Stokes
Reply to  Javier Vinós
February 25, 2023 11:24 pm

Everybody agrees on that and it is what the graph says.”

They don’t. Here is a post from Isaac Held titled
“How can outgoing longwave flux increase as CO2 increases?“Interestingly it was prompted by the fact that GCMs predict just that. Here is a key part

comment image

The shortwave feedback can exceed the direct effect on OLR. And that is just what Dewitte’s total result is saying.

JCM
Reply to  Nick Stokes
February 26, 2023 12:10 am

Total hog wash!!

ifs, ifs, and even more ifs!

“How can outgoing longwave flux increase as CO2 increases” ???
Feedbacks on feedbacks!! audacious harebrained conjecture!!!!

It’s getting ridiculous!!

Nick Stokes
Reply to  JCM
February 26, 2023 12:14 am

But it is observed. Dewitte says so.

JCM
Reply to  Nick Stokes
February 26, 2023 12:25 am

But it is observed.

A total counter argument to heat trapping vernacular. Other hypotheses must be considered.

Nick Stokes
Reply to  JCM
February 26, 2023 12:49 am

No, Dewitte’s graph says that heat is indeed trapped. Total heat out is less than heat in.

JCM
Reply to  Nick Stokes
February 26, 2023 1:02 am

not by a diminished proportion of OLR vs surface flux!! no such effect is observed…

Decadal changes of the reflected solar radiation and the earth energy imbalance”….

It is a cognitive dissonance to dismiss alternative conjecture for such effects.

One need not limit oneself to radiative feedbacks upon feedbacks…

a diminished OLR which subsequently results in OLR exceeding the initial magnitude based on internal IR active radiative trace gas perturbation….

far fetched even for laymen!! Let’s face it.

Editor
Reply to  Nick Stokes
February 26, 2023 4:54 am

It doesn’t. Give it up Nick.

bdgwx
Reply to  Andy May
February 26, 2023 11:36 am

That’s what Dewitte’s graph shows. EEI = ASR – OLR. ASR > OLR so EEI > 0. The astute reader will notice it is the 1st law of thermodynamics ΔE = Ein – Eout. Just because you think ASR is BS doesn’t mean that the 1LOT isn’t any less true or that scientists have been predicting the albedo feedback for at least 120 years. And the very citation in this article (Dewitte & Clerbaux 2019) even says in no uncertain terms that the increase in OLR is caused by the albedo feedback. They call it the “longwave cloud thinning effect”.

Editor
Reply to  Nick Stokes
February 26, 2023 4:54 am

No, he doesn’t say so. As Javier posted:

That includes OLR and RSR (albedo) and does not say anything about their relative contribution.

And I noted, you cherry-picked your period to coincide with the Pause.

Editor
Reply to  Nick Stokes
February 25, 2023 5:54 pm

Nick,
I’m just pointing out that the plot you showed looks a lot like the one that Javier showed, it just covers a smaller period. I suspect it is the same data, you just tried to make it say something it doesn’t say.

Reply to  Nick Stokes
February 25, 2023 8:58 am

as predicted by the GHE” but there are other hypotheses equally explaining the difference.

Solar absorption in the oceans also predicts less OLR escaping than SW absorption! CO2 longwave only penetrates few microns depth into a layer named the Cool Skin Surface because it is immediately releasing heat via radiation, evaporation and contact with the air. In contrast solar radiation is absorbed at depths that delay its release requiring time to migrate to the Cool Skin Surface where only then the ocean releases absorbed solar heat. The depth and time delay allows solar heat to be transported to regions outside of the tropics, where all observations show more heat is released than absorbed.

No change in solar SW or CO2 LW is needed to have ocean heat accumulate, and then be exported. Stratification by increased salinity is all that is required to accumulate heat as seen in salt gradient solar ponds. But dynamic increases in temperature due to salinity are never considered when climate models compare CO2 radiative effects to natural radiative effects using just solar and volcanic.

Reply to  Jim Steele
February 25, 2023 10:06 am

A good depiction of what happens in the ocean. Ocean-atmosphere heat exchange depends entirely on the atmosphere, which is responsible for evaporation and for setting the net thermal radiation flux.

Nick Stokes
Reply to  Jim Steele
February 25, 2023 12:14 pm

“as predicted by the GHE” but there are other hypotheses equally explaining the difference.”

Maybe, but Javier claims that the OLR observations are inconsistent with the GHE, and so “exonerate CO2”. They are not.

Richard M
Reply to  Nick Stokes
February 25, 2023 4:57 pm

Your graph of incoming solar is not supported by CERES. I assume they missed the cloud changes.

Editor
February 24, 2023 4:44 pm

Well done Javier!

Captain Climate
February 24, 2023 5:15 pm

Great article. I’d like to see what the latent heat of evaporation does in the process. Surely those CO2 molecules banging into water molecules stores enormous amounts of energy in phase transition and that’s sent skyward. Once that vapor is in the sky, it can release enormous amounts into space with next to no temperature change just through condensation. And that’s not even to speak of cloud feedbacks which decrease incoming solar radiation.

Reply to  Captain Climate
February 25, 2023 10:10 am

Absolutely. About 50% of the absorbed solar energy goes into the ocean. Most of it comes out as latent heat through evaporation and is delivered to the troposphere through condensation and transported to other places by winds. This is a water planet and latent heat is key to its climate.

February 24, 2023 6:27 pm

Excellent discussion of the net “cause & effect” of greenhouse gas warming & cooling. Your terrific illustration of the greenhouse gas “height of emission” will cause some heads to spin — but then again sudden exposure to reality can be shocking to some.

Editor
February 24, 2023 6:33 pm

Javier, I fear your test is two simplistic—it doesn’t allow for both things happening. You say:

So, this is the test:

– Surface warming but less or same OLR: CO2 is guilty as charged

– Surface warming and more OLR: CO2 is innocent

Here’s the CERES data. The surface is warming. And the amount absorbed in the atmosphere by CO2 and other greenhouse gases is indeed increasing. This indicates that CO2 is indeed playing a part.

comment image

However, here is the TOA Outgoing Longwave Radiation (OLR):

comment image

Hmmm … and the situation is further complicated by the changes in the amount of sunlight absorbed by the surface:

comment image

To quote Shakespeare,

There are more things in heaven and Earth, Horatio, / Than are dreamt of in your philosophy [science].

My best to you and yours,

w.

Reply to  Willis Eschenbach
February 24, 2023 8:46 pm

Willis,

Is there CERES data available for LW radiation emitted by the surface? That would serve as a proxy for surface temperature that could then be compared to your TOA OLR graph in order to test Javier’s criteria.

Reply to  Frank from NoVA
February 24, 2023 8:54 pm

Indeed there is. The atmospheric absorption is calculated as the surface ULR minus the TOA OLR. The difference between what is emitted by the surface and what makes it out to space is the amount shown in the graph, the radiation absorbed in the atmosphere by greenhouse gases.

w.

Reply to  Willis Eschenbach
February 24, 2023 9:59 pm

Got it – thank you!

Philip Mulholland
Reply to  Willis Eschenbach
February 25, 2023 12:19 am

the radiation absorbed in the atmosphere by greenhouse gases.

And Dust.

JCM
Reply to  Philip Mulholland
February 25, 2023 7:04 am

Atmospheric absorbed radiation = LW down Surface – Upward Emission from Atmosphere.

This is typically depicted in the range 130 – 140 W m-2.

The radiative flux to space depends on both gases and condensate.

LW up – OLR is the Ramanathan greenhouse definition, which cannot be accounted for as the radiative gas absorption, as some is lost via the window. It requires condensate.

JCM
Reply to  JCM
February 25, 2023 7:58 am

As a rule of thumb for radiation enthusiasts:

Ramanathan style Greenhouse Effect G = LW up surface / 3

G = Surface LW up – OLR – Transmitted Flux = LW down surface – Upward Emission from Atmosphere

g = G/LW up surface =1/3 = greenhouse factor.

Surface LW up = 3 OLR / 2

From a radiation perspective it cannot be refuted that these relations appear to be fixed. They are fixed scalars of net solar.

Richard Greene
Reply to  Willis Eschenbach
February 25, 2023 2:11 am

This is an excellent comment with easy to read charts
Thank you Willie “Einstein” Eschenbach. I’m “stealing” your charts for my climate science blog — the ultimate complement. Blog editor motto: “If it’s good, I’ll steal it”.
Honest Climate Science and Energy

Reply to  Willis Eschenbach
February 25, 2023 2:15 am

This indicates that CO2 is indeed playing a part.

I did not say otherwise. That is what I think. The question is if all recent warming and more has been caused by anthropogenic GHG emissions and compensated in part by aerosol emissions, as shown by IPCC in its figures.

OLR emissions say no. There must be another cause playing an important part.

Editor
Reply to  Javier Vinós
February 25, 2023 5:11 am

Exactly! If OLR is going up, it isn’t all CO2 and other greenhouse gases. Something else is at work as well, which blows up the AR6 hypothesis. Too bad we can’t be quantitative and know how much each contributes.

Dave Andrews
Reply to  Javier Vinós
February 25, 2023 8:25 am

“There must be another cause playing an important part”.

Exactly and that is why I keep going back to the fact that the open season at the coalport in Spitsbergen went from 3 months of the year before 1920 to over 7 months of the year in the late 1930s showing considerable warming in the Arctic. Hubert Lamb reckoned the average total area of sea ice in the Arctic declined by between 10 and 20% over the period.

This was well before CO2 really began to take off after 1950. What caused it ? We don’t know but it does show that there are other causes of warming

JCM
Reply to  Javier Vinós
February 25, 2023 8:40 am

For the gaseous component of atmosphere surface net radiation = OLR / 2.

i.e. clear sky surface net radiation = net SW + net LW = flux of sensible heat + latent heat = OLR / 2

The only variable of interest for global climate change from a radiative perspective is the condensate, or more specifically atmospheric liquid and solid.

The gaseous component has fixed relations.

LW surface Up – Transmitted Flux = 1.5 Net Solar.

Reply to  Willis Eschenbach
February 25, 2023 9:06 am

Willis, How do you explain the increased surface absorption of solar energy when there is a decline in solar irradiance for the past 2 decades?

Reply to  Jim Steele
February 25, 2023 10:20 am

Jim, always good to hear from you.

The trend in TSI since 1972 has been – 0.077 W/m2 per decade, for a total of -0.3 W/m2.

Surface SW absorption is the net of downwelling SW minus reflected SW at the surface. Here’s downwelling SW:

comment image

And here’s reflected SW:

comment image

So the answer to your question is twofold – clouds, and snow/ice reflections. Less clouds and less snow/ice lead to a 0.8 W/m2 decadal increase in surface absorption. This is on the order of ten times the decrease in TSI.

w.

Reply to  Willis Eschenbach
February 25, 2023 10:43 am

Thanks. I agree. Do you have a breakdown between cloud vs snow reflections

Reply to  Jim Steele
February 25, 2023 2:53 pm

I’d have to dig for that one. Clouds are easy, the CERES data has shortwave CRE.

But on the surface, one big problem is that in the polar regions, the angle of incidence is so low that it’s hard to tell water, snow, ice, and bare tundra apart.

A second major problem is the ephemeral nature of snow. It can snow, albedo goes way up for 2 days, then the snow melts … but the CERES and most other datasets are monthly.

Average TOA SW reflections are ~ 100 W/m2, and surface reflections are ~23 W/m2.

Here are the surface albedo values for February. I’m sure you can see the problem.

comment image

One possibility would be to do a gridcell by gridcell analysis, and look for jumps. But I suspect that would run into the same problem, low sun angle = many reflections = high albedo. Hang on … OK, here’s one gridcell.

comment image

Yeah, as I thought. No clear demarcation between tundra and snow.

w.

Reply to  Willis Eschenbach
February 25, 2023 3:20 pm

Thanks. I agree it is difficult. I was just hoping that you might have some insight that could help resolve the problem.

Do you have data that shows just changes in cloud albedo driven by La Nina-like conditions?

I assume that a cooler eastern Pacific driven by upwelling of cool water reduces cloud cover in the eastern Pacific and thus enables greater solar heating of waters that get transported to increase the western Indo Pacific Warm Pool

Reply to  Jim Steele
February 25, 2023 3:34 pm

One thing about having written ~ 1,000 posts for WUWT is that I often can find something from the past … here’s something, not exact, but near to what you want.

w.
comment image

JCM
February 24, 2023 6:51 pm

Radiative equilibrium dominates the lower atmosphere under the cloud base, or in the turbulent mixing layer, and prevents heat transfer other than by convection.

The surface of interest for radiation enthusiasts is the dynamical top of mixing layer.

ferdberple
February 24, 2023 6:58 pm

The atmosphere below 5 km is opaque to IR radiation. Not translucent. Opaque. Again. Opaque. Not translucent.

Thus IR radiation has no role in surface temps below 5 km. You cannot have radiation latey wise through an opaque item. Thus, It is convection, advection and conduction that do the heavy lifting to cool the surface back radiation is physically impossible in an opaque atmosphere.

Reply to  ferdberple
February 24, 2023 7:36 pm

And just 1kg/m^2 of ice above any level is essentially opaque to OLR. Ice controls the thermal balance whether it is in the atmosphere, on land or on water. Its triple point at 273K is the reason Earth has such a stable climate despite all the disturbances it gets exposed to over days. months, years, decades centuries, millennia and throughout the 4bn or so years of its evolution.

Reply to  ferdberple
February 24, 2023 9:07 pm

back radiation is physically impossible in an opaque atmosphere.

I have a feeling you are referring to observable physics rather than climate phiisics. Anything is possible when you rewrite the laws of physics as the climate community have done and embody that phiisics in their models.

Reply to  ferdberple
February 24, 2023 9:13 pm

Ferd, a couple of points of clarification.

First, in the “atmospheric window,” something on the order of 40 W/m2 goes straight from the surface to space. So your claim that the atmosphere is “opaque” is simply not true, no matter how many times you repeat it.

Second, upwelling IR is absorbed by greenhouse gases and clouds. However, that’s not the end of the story. It’s not like light being absorbed by paint, which is never re-radiated as light.

To the contrary, once IR is absorbed, it becomes part of the energy in the atmosphere, which is constantly radiating IR both upwards and downwards.

Thus, something on the order of half of the absorbed radiation is returned to the surface, and thus it has a huge role in the surface temps.

Finally, the idea that back radiation is “physically impossible” flies in the face of repeated measurements of downwelling IR “back radiation” made for years and years by scientists and automated instruments all around the planet, 24/7/365. You are free to claim that such measurements are imaginary or incorrect or the result of scientists not understanding what the instruments do, or because the manufacturers of the instruments have no clue what they are building.

It also flies in the face of the fact that you can purchase a standoff IR thermometer and see the difference between clear-sky and cloudy downwelling IR for yourself. You can claim that that’s “physically impossible” as well.

And we are free to point and laugh at such incredible claims.

I discuss these questions in detail in my post “The Radiation Fight“.

Best regards,

w.

Reply to  Willis Eschenbach
February 24, 2023 10:04 pm

Willis,

Can the spectrum of the 40 W/m^2 radiated from the surface directly to space be used to infer an average surface temperature? Something I’ve wondered about…

Reply to  Frank from NoVA
February 24, 2023 10:15 pm

Good question. Certainly seems possible, but I have no idea if it has been done.

w.

Editor
Reply to  Frank from NoVA
February 25, 2023 5:23 am

Frank yes, it can be done and has, see here: The Greenhouse Effect, A Summary of Wijngaarden and Happer – Andy May Petrophysicist

Especially figure 1, which is also attached. Notice the temperature labeled “0 km.” Figure 3 shows surface temperatures from the spectra in Sahara, Mediterranean, and Antarctica.

Figure 87_150.jpg
JCM
Reply to  Andy May
February 25, 2023 12:31 pm

OLR spectra includes surface transmitted flux (~288K) and that emitted from atmosphere.

To understand the OLR spectra one must superimpose the bulk atmospheric emission curve which can be approximated by a 273K blackbody function. The dynamic liquid and solid condensed matter emit full spectrum IR.

Untitled.png
Reply to  Andy May
February 25, 2023 6:34 pm

Thanks, Andy! Fyi, I’ve recently had some back and forth with several folks who defend NASA’s data ‘adjustments’ as a necessary step in calculating the Earth’s GAST in order to then determine the impact of our emissions. It seems to me that if we have records of OLR in the so-called ‘atmospheric window’, changes in GAST could also be observed independently of the surface temperature record. Perhaps a good research project.

Reply to  Willis Eschenbach
February 24, 2023 11:14 pm

which is never re-radiated as light.

I am certain my house is painted white. At least that it what I observe as my eyes interact with the same EMR field that white paint is altering.

Finally, the idea that back radiation is “physically impossible” flies in the face of repeated measurements of downwelling IR “back radiation” 

The word “radiation” is short hand for ELECTRO-magnetic radiation. It is the transport of energy in the electric field coupled with the magnetic field. Electric field is a vector quantity and energy always flows from high potential to low potential. Energy can never be transferred against the potential. Temperature is just another way of measuring electric potential. The energy of an electron is often given in electron volts (eV) and that is relatable to temperature and wavelength. Energy cannot travel against the potential. Energy quanta emitted from an excited molecule will always move down potential – from hot to cold. NEVER the reverse.

The electric field in a vacuum equilibrates at the speed of light. Exactly the same speed that mass equilibrates in a gravity field. Instantly remove Earth from existence and the sun will see changes in the gravity and electric fields in 8 minutes. For example, the sun “senses” the existence of Earth through both the gravity field and electric field that we all exist in.

Whatever is being measured as back-radiation, it is not energy. It is simply using the S-B approximation for radiative transfer to infer an energy transfer. It is unphysical nonsense. It only exists in climate phiisics.

I know you will not understand this but Michael Mischenko provides the proof here:
https://www.giss.nasa.gov/staff/mmishchenko/publications/2010_OE_18_19770.pdf
If you go to the conclusions you can read:

Thus under no circumstances is the local flow of electro-magnetic energy polydirectional.

That makes it quite clear. Michael is no longer with us but his contribution to the physics of radiative energy transfer in the atmosphere is formidable.

Reply to  RickWill
February 25, 2023 8:59 am

That was tough reading. I hate matrix math so that made it worse.

Reply to  RickWill
March 2, 2023 4:52 pm

Willis has no idea what the difference between “energy” and “power” is. He also has no idea what “radiation” is. Trying to explain these concepts to him is about as effective as trying to explain them to a brick wall. He is a fisherman, remember, not a physicist. All he can do is say “but the climate scientists say so! It must be true! They wouldn’t all be lying, would they? No of course not, that’s just common sense! Bovine excrement! Pass! Pass! Pass!”

Reply to  stevekj
March 2, 2023 7:01 pm

stevekj  March 2, 2023 4:52 pm

Willis has no idea what the difference between “energy” and “power” is. He also has no idea what “radiation” is. Trying to explain these concepts to him is about as effective as trying to explain them to a brick wall. He is a fisherman, remember, not a physicist. All he can do is say “but the climate scientists say so! It must be true! They wouldn’t all be lying, would they? No of course not, that’s just common sense! Bovine excrement! Pass! Pass! Pass!”

This is why I ask people to quote the exact words that they are discussing … and even when I ask, some jerkwagon like stevekj will still come along, make up some nonsense, and claim I said it.

Here’s my Google scholar record for those interested:

comment image

199 citations to my work, a peer-reviewed “Brief Communications Arising” in Nature magazine, a short piece in the Proceedings of the National Academy of Sciences … not bad for a “fisherman”. Nature’s peer reviewers sure didn’t whine about my history like you are … because as scientists, the peer reviewers were judging my ideas, not my persona.

And you? Oh, wait, you don’t even have the stones to sign your own name, so we can’t tell what you’ve published.

Finally, yes, I’m totally self-educated. However, my education and lack of credentials (I have none) are meaningless.

Here’s why

w.

Reply to  Willis Eschenbach
March 3, 2023 8:22 am

I would sign my correct name if I could, but WordPress seems to ignore my attempts to do so since the change to registered-users-only. It’s “Steve Keppel-Jones”.

Anyway it’s fascinating that you have so many published articles and still have no idea how physics works. That’s not a compliment to you, it’s a condemnation of the lack of rigorous review of published articles. But we knew that already…

I have no problem with self-educated people, of course, more power to them. But your self-education has missed the fundamentals of theoretical physics. ALL of them. Whenever anyone (such as me) asks you to defend anything related to physics, your only retort is “but the experts said so”, followed by a lot of name-calling, like “pond scum” and “bovine excrement”, and then, when pressed further, “Pass!” “Pass!” “Pass!”. That’s not “self-education”, that’s “a fisherman pretending to be a scientist, and doing a terrible job of it”.

Not only do you not know the physics, you don’t know that you don’t know. You pretend to know, like you’re some sort of expert. Real scientists are humble in their ignorance. You are arrogant and obstinate in yours.

You have also, in your so-called self-education, missed everything related to logic and philosophy. So you cannot recognize a logical fallacy when you see one, and you insist on repeating the same ones over and over. Your favourite one is the Argument from Authority, but you use lots of others regularly too, such as Argument from Incredulity and Ad Hominem Fallacy. And of course the brand-new fallacy that I had to invent and name after you, the “Willis Eschenbach Fallacy”, or the Argument from Common Sense.

In your favour, I haven’t seen anything wrong with your self-educated use of statistics. Yet. But you have no idea what the concepts behind the statistics mean, so they aren’t really helping anyone. Props for your astounding volume of ignorant output, though…

Reply to  stevekj
March 3, 2023 11:58 am

Steve, thanks for identifying yourself. However, I note that once again you’ve provided paragraph after paragraph of name-calling and ad-hominem personal attacks against me … but you haven’t had the balls or perhaps the scientific knowledge to quote the exact words of one scientific claim that I’ve made and show us, not claim but demonstrate to us, that it is wrong.

I also can’t find a single peer-reviewed article about climate or any other subject under your name in Google scholar … but you’re more than willing to accuse me and every scientist who peer-reviewed my articles of not doing proper scientific work. Cute.

Here’s a post on how to conduct a scientific debate. And here’s the graphic from that post.

comment image

Protip—you’re at the bottom of the pyramid, the one titled “Name-Calling”, with occasional forays up one step to “Ad Hominem”.

You desperately need to move way, way up the pyramid to be of interest to readers. Until you do so, you’re just bitching, whining, calling me ugly names, and standing on tiptoes in a failed attempt to bite my ankles … and more than anything, it’s boooring …

w.

Reply to  Willis Eschenbach
March 4, 2023 1:17 pm

No, Willis, I am not just calling you names. I have pointed out every time you have made a false physics statement that I’ve seen, and so have a lot of other people. Your response to every one of these rebuttals is “but the government scientists said so”, and when we point out that the government scientists are wrong, your second line of defense is invariably “Pass”. Is that what you call “thinking”, or “defending the words you wrote”? Because the rest of us call that “one logical fallacy after another”. No, you are not at the top of the pyramid, despite your delusion to the contrary.

You have no idea what any of the following physics words means: “power”, “work”, “energy”, “radiation”, “temperature”, “heat”, or “entropy”. Go ahead and prove me wrong: just provide a one-sentence definition of each, in your own words, without looking anything up on Wikipedia. I’ll wait.

Reply to  stevekj
March 4, 2023 6:21 pm

Once again, as if to truly hammer home my point, all you’ve done is call me names without quoting one scientific claim I’ve made and refuting it. Just as I said in my previous posts, that’s your métier. Booooring.

Regarding proving myself to you, I have no need to do any such thing. That’s your fantasy. I have my life. I have my journal publications and my online publications. The peer reviewers didn’t have your strange ideas and demands. No surprise, they’re scientists. And unlike you, I guess, most of the folks who read what I write here on WUWT have no problem understanding exactly what I’m saying, and they are willing to both discuss my ideas rationally and quote and point out any errors they think I’m making.

If you’re going to “wait” as you claim for me to accede to your bizarre demands, I hope you’ve got lots of survival food and a bottle to piss in—it’s gonna be a very long and unproductive stakeout.

w.

Reply to  Willis Eschenbach
March 5, 2023 12:45 pm

No, Willis, I am not going to trawl through your myriad of publications to find even more of the same physics nonsense you post here. I assume it’s the same nonsense, but tell me if I’m wrong.

And no, publishing papers does not make you smart, wise, or correct. Those two phenomena may be correlated, mostly by accident as far as I can tell; but it is equally likely that you are just a widely published ignoramus, like Michael Mann, or David Viner.

In case you have forgotten, the Central Point of the argument at hand is as follows:

“There are no positive downwelling longwave infrared power measurements, at ambient temperature, at the surface of the Earth, at night.”

I (and quite a few others) tell you this every time you state the (false) opposite. Both statements are on Level 1 of your pyramid, and my rebuttal is an objectively verifiable fact. It does not depend on my opinion, or yours – just the first principles of physics.

Your responses so far have consisted of the following extremely lame attempts to duck the issue, since you can’t argue the physics directly, because you obviously don’t understand any of it in the slightest, and (the unforgivable sin for a wannabe-scientist) you have no interest in learning (“Booooooring!” “Pig wrestling!” “Pass! Pass! Pass!”):

1) “Government scientists told me so.” (Argument from Authority, i.e. a Logical Fallacy, a form of debate so weak that it didn’t even make it onto your pyramid, so let’s add that level and call it Level 8)
2) “I can’t believe all those scientists would lie to me, that’s just common sense.” (Argument from Incredulity, also Level 8)
3) “I have published a lot of papers, so I must be right.” (Argument from Authority again, and this time the authority is yourself – still on Level 8, but an even weaker fallacy than before, if that were possible)
4) “You have published no papers, so you must be wrong.” (Reverse Argument from Authority, still a Logical Fallacy, Level 8, or possibly, more charitably, simply an Ad Hominem, level 6)
5) “You are pond scum.” (Name Calling, pyramid level 7)
6) “I refuse to engage in pig wrestling.” (Implying that your opponent is a pig, i.e. Ad Hominem, level 6 again)
7) “You can lead a horse to water, but you can’t make him think.” (Once again implying that your opponent is an animal, in this case a horse, i.e. another Ad Hominem, level 6 again)
8) “Pass.” (You don’t have a level in the pyramid for this one either, so we’ll have to add Level 9, or the “Willis Eschenbach” level, also known as “Refusal to engage brain on the subject at all“. I’m tempted to call this the Curley Howard level – “I’m tryin’ ta think, but nuthin’ happens!”)

I suppose we could be generous and consider all those logical fallacies to be simple Contradiction, i.e. Level 4, but that may be giving them more credit than they are due. We should probably distinguish between regular Contradiction (simply disagreeing, with no substantive backing evidence) and Logical Fallacies, which are a more egregious case of a failure to think properly, despite giving the appearance of trying.

Is it any wonder that the rest of your interlocutors who know more about physics than you do, faced with this level of absurdly arrogant ignorance and hypocrisy, simply give up and call you an idiot? I won’t stoop to that level; it is beneath me. I’ll stick with “Fisherman pretending to be a climate scientist” instead.

Reply to  stevekj
March 5, 2023 5:14 pm

stevekj March 5, 2023 12:45 pm

No, Willis, I am not going to trawl through your myriad of publications to find even more of the same physics nonsense you post here.

Of course not. That would require that you actually do some work.

In case you have forgotten, the Central Point of the argument at hand is as follows:

“There are no positive downwelling longwave infrared power measurements, at ambient temperature, at the surface of the Earth, at night.”

I cannot find that quotation anywhere in this thread, so I must assume that you just made it up.

I (and quite a few others) tell you this every time you state the (false) opposite.

No, you don’t. If you did, I’d find it as a quotation somewhere in this thread. But you just made it up.

Both statements are on Level 1 of your pyramid, and my rebuttal is an objectively verifiable fact. It does not depend on my opinion, or yours – just the first principles of physics.

Re-read the description of the pyramid. To be at Level 1, you need to actually quote my scientific claim, and then provide the scientific evidence that it’s false.

You’ve done neither. Instead, you just invented a quote.

On the other hand, I’ve provided links to places where they routinely measure “positive downwelling longwave infrared power measurements, at ambient temperature, at the surface of the Earth, at night.”

These include the TAO buoys, the SURFRAD station network, and the ARM network.

And I’m sorry, but claiming that all of the measurements that those large groups of scientists are constantly taking are incorrect because of “the first principles of physics” is a sick joke.

w.

Reply to  Willis Eschenbach
March 8, 2023 6:18 pm

“you just invented a quote”

Sure, Willis, you go with that, if it makes you feel better.

To placate you, here is an actual quote of the statement you followed up with, that I apparently “invented”, and which I have tirelessly been refuting (with direct quotes) about once a month, every time you make it here on WUWT:
“places where they routinely measure “positive downwelling longwave infrared power measurements, at ambient temperature, at the surface of the Earth, at night.””

I also rebutted the same statement downthread, when you said, quoting another poster’s comment:

“‘But longwave radiation warms the surface as well’. That’s correct, it does, at a rate of about 320 W/m2”. And I said “no it doesn’t”.

(If it’s too complicated for you to follow all these interwoven threads, I apologize – there sure are a lot of comments here.)

Anyway, let’s work with the quote you helpfully repeated above, so that you can’t claim I “invented” it this time (anything to avoid discussing the actual physics, right?) Here is my rebuttal:

No, they did not measure positive DWLWIR at the surface at ambient temperature at night. They measured negative DWLWIR, in the range of 0 to -100 W/m^2 depending on humidity. This corresponds to the CERES average value for “net power” of -53 W/m^2 (they made up the “net” part, because there is no such thing as “net power”, there is only “actual physics power” and then “fake climate science power, which disappears when you look at it closely”), so those all hang together and make sense. These measurements also respect the Second Law of Thermodynamics, which is a good clue that they’re correct, or at least in the right ballpark.

So the direct measurement is negative, and then they adjust the measurement (by a lot) to produce an adjusted positive reported value. That’s not the same thing as a measurement!

This claim is Level 1 on the debate pyramid. A statement of fact, verifiable by reference to first principles, or direct experiment. You know, science.

I predict that your response to this, as always, will not be to look up the way they make adjustments and try to defend them on the facts, because that would involve actual work, not to mention learning physics. Nor will you perform the experiment yourself, also too much work. Instead, as before, you will rely on an Argument from Authority logical fallacy. That is level 8 in my amended version of your debate pyramid, in case you forgot. You may, equally likely, also choose to “Pass” (Level 9), because it’s easier than thinking. But I could be wrong in my prediction. Surprise me…

Richard Greene
Reply to  Willis Eschenbach
February 25, 2023 2:51 am

The Radiation Fight was a great article, worth re-reading. It set off the First World War of Climate Science at this website, with over 839 comments.

“And we are free to point and laugh at such incredible claims.”

That point and laugh behavior is outlawed by Al Gore’s Internet Rule 1b, which warns of the potential dangers of pointing and laughing. Al recommends merely refuting the comment … and I suggest adding a character attack on the commenter’s ancestors, comparing them to farm animals, as the finishing touch.

This comment is serious, not satire.

Reply to  Willis Eschenbach
February 25, 2023 9:22 am

What needs to be quantified is the time lag between LW emitted from the surface then absorbed in the atmosphere and then radiated back to the surface?

The loss of LW through the transparent atmospheric window is not a static phenomenon.

For example, the surface absorbs 160 W/m2 and let’s assume it then immediately radiates it all back towards space. 40 W/m2 (25%) escapes thru the window, while GHGs absorb specific wavelengths and re-emit half back to the surface. Thus those wavelengths of LW adds 60 W/m2 (half of the remaining 120 W/m2) back to the surface, which acts like a black body. But now the surface radiates the entire spectrum back towards space and again another 25% escapes undeterred through the window. If this happens at the speed of light it wouldn’t take long to cool the surface and minimize any GHE.

Camping in Death Valley I went from getting badly sunburned to having my water bottle freeze in less that 12 hours.

Reply to  ferdberple
February 25, 2023 1:06 am

And yet all sorts of things, just about absolutely everything, can be seen in IR on and above the surface.

Reply to  AndyHce
February 25, 2023 9:08 am

Andy, the spectral response of IR thermometers is carefully chosen to avoid absorption by the atmosphere or other intervening things. See here for a good discussion. From the article:

Whereas the early IRT’s required a broad spectral band of IR to obtain a workable detector output, modern IRT’s routinely have spectral responses of only 1 micron. The need to have selected and narrow spectral responses arises because it is often necessary to either see through some form of atmospheric or other interference in the sight path, or in fact to obtain a measurement of a gas or other substance which is transparent to a broad band of IR energy.

comment image

MODERN INFRARED THERMOMETER

Some common examples of selective spectral responses are 8-14 microns, which avoids interference from atmospheric moisture over long path measurements; 7.9 microns which is used for the measurement of some thin film plastics; and 3.86 microns which avoids interference from CO2 and H2O vapor in flames and combustion gases.

My best to you,

w.

Reply to  ferdberple
February 25, 2023 2:18 am

It is convection, advection and conduction that do the heavy lifting to cool the surface

The surface cools primarily by evaporation. This is a water planet.

Editor
Reply to  Javier Vinós
February 25, 2023 5:37 am

Javier, the main control on evaporation is wind speed, so convection modulates evaporation.

Reply to  Javier Vinós
February 25, 2023 9:14 am

Javier Vinós February 25, 2023 2:18 am

The surface cools primarily by evaporation. This is a water planet.

Not true. The surface cools primarily by radiation, with a radiative loss of ~390 W/m2. Evaporation is on the order of 75 W/m2, and sensible heat loss is about 20 W/m2. See below.

comment image

Please don’t say “But longwave radiation warms the surface as well”. That’s correct, it does, at a rate of about 320 W/m2 … but we’re not talking about what warms the planet.

w.

Reply to  Willis Eschenbach
February 25, 2023 10:24 am

Not true. The surface cools primarily by radiation

To change the temperature what matters is the net flux. This was cleverly disguised in that figure to make all others appear irrelevant.

The truth is this:

comment image

Latent heat loss is about double the net thermal flux loss for the ocean surface. If you calculate the net flux for the entire surface the image is similar.

from:
Schmitt, R.W., 2018. The ocean’s role in climateOceanography31(2), pp.32-40.

Reply to  Javier Vinós
February 25, 2023 12:21 pm

You are changing the goalposts. You said “The surface cools primarily by evaporation.” You said nothing, zero, about how it warms.

But now, you want to include how the surface WARMS. If that was what you meant, then you need to include that in your statement.

w.

Reply to  Willis Eschenbach
February 25, 2023 5:31 pm

I didn’t change anything. The graphic I posted shows very clearly that the surface cools primarily by evaporation, as I said. The biggest arrow going up is labeled latent heat loss, not longwave loss. It does not cool primarily by radiation. 100 Wm-2 by latent heat, 53 Wm-2 by longwave loss, in case you missed it.

The only thing that warms the surface is the Sun, the rest is exchanged with a net negative result.

JCM
Reply to  Willis Eschenbach
February 25, 2023 10:59 am

In the diagram we see once again the imagined discontinuity in LW flux.

339 Absorbed – 321 LW down = 18 W m-2 discontinuity.

Correcting such diagrams:

The total column transmittance can be derived from absorptivity.

Atmospheric absorbance = LW down surface / LW up surface

Absorbance here = 321 / 392

Absorbance = 0.82

By fixed relation, transmittance = 0.15

https://www.sigmaaldrich.com/CA/en/support/calculators-and-apps/absorbance-transmittance-conversion

Transmitted flux (total column window) = 0.15 * 392

Transmitted flux = 59

The diagram illustrates the imagined 40 W m-2 total column window.

Here the discontinuity is accounted for by 59 – 40 = (19)

OLR = Transmitted Flux + Atmospheric Emission

Corrected atmospheric emission = 237 – 59 = 178.

The discontinuity of atmospheric emission reflects the discontinuity of surface flux.

178 = 147 + 50 – (19)

There can be no discontinuity of atmospheric absorbed radiation and back radiation.

The boundary layer is constrained by radiative exchange equilibrium due to turbulent mixing. No LW discontinuity.

The error stems from double accounting of about 20 W m-2 in such budgets. This represents a double accounting of sensible heat and radiation.

The imaginary discontinuity is illustrated in the zone indicated with red arrow below.

The convective adjustment is equivalent to Net Solar (F) / 2 = OLR / 2 = Surface Net Flux = LH + SH

Sensible heat is retained in the turbulent mixing layer, while latent flux represents net flux aloft.

Net cooling of the surface is by LW transmittance + Latent Flux.

Untitled.png
JCM
Reply to  JCM
February 25, 2023 11:27 am

“Surface Net Flux = LH + SH” should read Surface Net Radiation equivalent = LH + SH.

A lazy mistake by me!!

Reply to  Willis Eschenbach
March 3, 2023 9:29 am

“‘But longwave radiation warms the surface as well’. That’s correct, it does, at a rate of about 320 W/m2”

No it doesn’t.

gyan1
February 24, 2023 8:51 pm

“If at any time we detect more OLR that would indicate another cause for the warming.”

At least a dozen studies have documented a reduction in clouds during the modern warm period. The average increase in solar radiation reaching the surface found from these is about 5W/m-2.

CO2 has been exonerated from being the dominant forcing by these measurements.

Reply to  gyan1
February 25, 2023 10:25 am

Clouds are complicated so we don’t really know how they have changed. What we do know is that albedo has changed very little.

gyan1
Reply to  Javier Vinós
February 25, 2023 11:21 am

“Clouds are complicated so we don’t really know how they have changed.”

We do know how they have changed during the satellite era from measurements.

https://file.scirp.org/Html/22-4700327_50837.htm

https://journals.eco-vector.com/0205-9614/article/view/11444

http://www.bbso.njit.edu/~pgoode/txt/txt/Goode_Palle_2007_JASTP.pdf

https://acp.copernicus.org/articles/13/8505/2013/

Reply to  Javier Vinós
February 25, 2023 11:49 am

Javier, it’s not clear what you are calling “very little”.

Per CERES, in the 22 years since March 2000, the amount of solar radiation reflected by the earth has dropped by -1.6 W/m2.

During the same period, theoretical CO2 forcing has increased by 0.7 W/m2, less than half the change in albedo.

Best regards,

w.

Reply to  Willis Eschenbach
February 25, 2023 5:23 pm

I call very little what Stephens et al. 2015 call very little:

The albedo of Earth appears to be highly buffered on hemispheric and global scales as highlighted by both the hemispheric symmetry and a remarkably small interannual variability of reflected solar flux (~0.2% of the annual mean flux).

Stephens, G.L., O’Brien, D., Webster, P.J., Pilewski, P., Kato, S. and Li, J.L., 2015. The albedo of EarthReviews of geophysics53(1), pp.141-163.

Dewitte et al. say:

The trend of the original CERES RSR relative to the ERA5 RSR is −0.5 ± 0.03 (1 σ) W/m2 /dec. The trend of our ageing corrected CERES RSR relative to the ERA5 RSR is −0.03 ± 0.03 (1 σ) W/m-2/dec. Thus after our CERES RSR ageing corection there is no more signifcant trend compared to ERA5, while before our ageing corrrection there is a significant trend.

Dewitte, S., Clerbaux, N. and Cornelis, J., 2019. Decadal changes of the reflected solar radiation and the earth energy imbalance. Remote Sensing11(6), p.663.

I wouldn’t put much faith in the evidence that the albedo is decreasing significantly.

Reply to  Javier Vinós
February 25, 2023 6:15 pm

Thanks, Javier, interesting.

w.

Editor
Reply to  Willis Eschenbach
February 25, 2023 6:06 pm

Willis,
Albedo is decreasing, but not as fast as the CERES data suggests. In the attached plot blue is the CERES Albedo and black is the Earthshine albedo. I think the CERES data has some problems.

Albedo.png
bdgwx
Reply to  gyan1
February 25, 2023 2:40 pm

What caused the reduction in clouds?

gyan1
Reply to  bdgwx
February 25, 2023 3:50 pm

“What caused the reduction in clouds?”

Nobody knows with certainty.

bdgwx
Reply to  gyan1
February 25, 2023 5:17 pm

Then how can you eliminate CO2?

gyan1
Reply to  bdgwx
February 25, 2023 5:56 pm

CO2 is supposed to increase water vapor which would increase clouds.

bdgwx
Reply to  gyan1
February 26, 2023 11:20 am

How do you know it isn’t CO2 that catalyzed changes in water and clouds?

gyan1
Reply to  bdgwx
February 28, 2023 1:42 pm

Physicists who have done the line by line calculations of CO2’s radiative flux show that it doesn’t have a strong enough forcing to produce the effects you are suggesting.

bdgwx
Reply to  gyan1
February 28, 2023 3:10 pm

I think you have me confused with someone else. I’ve not discussed CO2 effects in this subthread. I’m just asking how you know CO2 cannot cause a reduction in clouds if you don’t know what the cause is.

real bob boder
Reply to  bdgwx
February 26, 2023 7:16 am

The theory

bdgwx
Reply to  real bob boder
February 26, 2023 11:29 am

The theory is that CO2 will put a positive radiative force on the planet, warming the planet, reducing albedo, and increasing OLR. That’s what is being observed so…

Reply to  bdgwx
February 27, 2023 2:25 am

Anything that warms the planet should increase OLR and affect albedo.

What distinguishes the no-feedbacks GHE is that the warming comes from a reduction in OLR. A reduction in OLR has never been observed.

The increase in OLR should come from H2O effects, not from CO2 effects.

bdgwx
Reply to  Javier Vinós
February 27, 2023 9:49 am

We missed our opportunity to observe the no-feedback OLR reduction. It would have need to have been at least prior to 1900 and probably well before that even.

Dodgy Geezer
February 24, 2023 9:30 pm

Interesting.

Now try to convince the businessman making billions from worthless carbon credits, the politicians that he has bought, and the mass of scientists who depend on this money for grants and income.

That is where the problem is. Not in the technical understanding of radiative energy interchange….

February 24, 2023 9:51 pm

Shoot. I did a lot more calculations. But came to the same conclusion. You forgot to dwell on the incoming SW which is also affected by the CO2.
https://breadonthewater.co.za/2022/12/15/an-evaluation-of-the-greenhouse-effect-by-carbon-dioxide/

Editor
Reply to  Henry Pool
February 25, 2023 5:45 am

Thanks for the link Henry. Good post. The link in your post to the spreadsheet is busted though.

Reply to  Andy May
February 25, 2023 6:13 am

Thanks Andy. let me check on that.

Reply to  Henry Pool
February 25, 2023 7:08 am

The link works, but for some reason you end up in the middle of the Xcel document which has many entries. To see the end result of all my calculations you have to scroll up to the first three rows, tune to columns K,L and M. It shows that the cooling effect by the CO2 due to the sun (by deflecting SW) is just as big as warming effect by the CO2 due to earth (by deflecting LW)

FYI (all), I just watched the first presentation at the Heartland Institute

Patrick Moore, mede-oprichter van Greenpeace, over het klimaatnarratief versus feiten – Climategate Klimaat

Amazing. Amazing. This story is so simple to understand. Too bad so many people can’t understand it. That the lie has become so enormous that it has led all of humanity – or at least a very large part of it – astray. We have to consider that this idea of gas purity is probably just as diabolical as the idea of racial purity back in the late thirties. (of the previous century)
You wonder who is behind it, and who wants the confusion to continue. You wonder if people like SM and FE perhaps get paid by someone to try and suppress the truth.
I wonder.

Editor
Reply to  Henry Pool
February 25, 2023 10:08 am

Still doesn’t work for me, must be a permissions problem. Don’t worry about it, have a good time at the conference.

Reply to  Andy May
February 25, 2023 10:17 am

Thanks. Let me work on that.

February 24, 2023 10:39 pm

wrong wrong wrong

you cant critique the AGW argument until you demonstrate that you understand it

Reply to  Steven Mosher
February 25, 2023 2:54 pm

Funny! I have never been convinced that Mosher ever understood natural climate change never mind AGW. Instead of demonstrating his implied superior “understanding” he just engages in juvenile personal attacks such as his simple minded “wrong, wrong wrong” to deflect science he disagrees with.

February 24, 2023 10:45 pm

energy balance is why it warms
https://youtu.be/slPMD5i5Phg

Reply to  Steven Mosher
February 25, 2023 10:37 am

I perfectly understand what Pierre & Humbert say in that short video and I agree with it. Do I need to get some sort of certification to demonstrate I understand AGW? Is it required to agree with the conclusions of AGW, which are not based on how the GHE works, to be able to speak?

I know Earth’s energy imbalance is why it warms, what I don’t know is if you read my article before posting such “profound” arguments.

Editor
Reply to  Steven Mosher
February 25, 2023 1:23 pm

Steven,
Pierrehumbert’s video is quite good. I agree with all of it, and it is perfectly consistent with Javier’s post as far as I can tell. I suspect that Pierrehumbert and I might disagree on the magnitude of the greenhouse gas effect on climate, but on basic principles, we agree.

Reply to  Steven Mosher
February 25, 2023 3:12 pm

Of course Mosher, as all skeptics understand, and Humbert states, the only way the earth can lose the radiative heat energy it absorbed and prevent runaway heat accumulation is by radiating that heat back to space. Duh!

But you and Humbert somehow ignore how oceans accumulate heat so that it cannot be radiated back to space unless it migrates to the Cool Skin Surface Layer. Instead of discussing ocean dynamics that store heat, you insist on a narrative that only CO2 can slow the release of heat back to space. How stooopid.

John Power
Reply to  Steven Mosher
February 26, 2023 4:44 pm

energy balance is why it warms’
 
Not necessarily. The Global Mean Temperature doesn’t just depend on the amount of heat-energy in the system; it also depends on the way in which that heat is distributed in the system – i.e. the system’s entropy, in other words.
 
The more evenly the system’s heat is distributed, the higher is its entropy and the higher is its overall mean temperature for a given amount of heat-energy. And conversely, the more unevenly the system’s heat-energy is distributed, the lower is its entropy and the lower is its overall mean temperature for the same amount of heat-energy. So, if the total entropy of the Earth’s heat-energy system increases but its total heat-energy remains the same (perhaps because the TOA energy-balance remains the same), that must automatically raise the Global Mean Temperature.
 
In plainer language, the planet doesn’t necessarily need to absorb more energy in order to undergo global warming. It will also undergo global warming if the existing total heat-energy content is redistributed more evenly over the globe.
 
Perhaps I should add that this is not ‘new science’ which I’ve just invented. It’s standard thermodynamics applied to the global climate system.

February 25, 2023 12:00 am

Two fundamental errors in this story:

While the timing of collisions of an excited CO2 molecule is smaller than the time needed to emit the IR radiation, that effect is bidirectional: high energy N2 and O2 molecules can excite a CO2 molecule and that may emit its energy as a photon before the next collision…

Second problem: the temperature of the CO2 molecule when emitting its IR photon is of not the slightest influence on the wavelength or energy content of that photon, thus:
“the planet should emit less OLR as the new altitude is colder and a reduced OLR is the warming mechanism.” is completely wrong for the effect of any GHG.

Reply to  Ferdinand Engelbeen
February 25, 2023 12:53 am

There are no ‘high energy’ N2 or O2 molecules, as they are permeable to all radiation.
As far as your second point is concerned, I addressed this in a reaction earlier, which has not come up yet.

Reply to  Henry Pool
February 25, 2023 4:41 am

Henry, in a mix of gases, there are always individual molecules with higher and lower vibration energy, “temperature” is only showing the average energy of all molecules…
In the case of collisions: an excited CO2 molecule can give its extra energy to a N2 or O2 molecule and further collisions can distribute that to the whole mass, but the opposite is also possible for collisions of “hot” N2 and O2 molecules with CO2 or water molecules…

Reply to  Ferdinand Engelbeen
February 25, 2023 7:54 am

FE,
In my post (see comments earlier up this thread) I make the case that together, we (by burning fossil fuels) and nature (warmer oceans due to more heat from sun/earth), have added only 0.01% extra CO2 to the atmosphere since 50 years ago. That works out to 1 extra CO2 molecule which is surrounded by 10000 other molecules, on average. Now you may argue that there are a billion or so collisions between molecules, but that is not the issue with the GHE. You have to have mass (density) as well to catch the heat from the effect. In the case of water, usually the molecules stick together and there is always some mass. The CO2 however divides itself by diffusion and the molecules do not stick together. So the moment the CO2 molecule receives light of the absorbing wavelength, it will start re-emitting it, IMHO 62,5% in the direction of the source, the rest goes around. The O2 and N2 has no absorptions, so it lets all emitted radiation from the CO2 through.
On a previous thread on sunspots in the atmosphere I shared an important fact about the absorption of water (the cause of sunburn, the cause of CO2 in the air and cause of clouds) in my discussion with Bob Weber.

Reply to  Henry Pool
February 25, 2023 8:03 am

PS
the 62.5% comes from the assumption that the molecule is a perfect sphere.

Reply to  Henry Pool
February 25, 2023 8:45 am

There are no ‘high energy’ N2 or O2 molecules, as they are permeable to all radiation.”

if this were true then the attached from a thermodynamics book is wrong.

AA484370-2875-4488-891C-C7322738CB41.jpeg
Reply to  mkelly
February 25, 2023 10:35 am

Eishh. A book is what you got going. Have you measured, anywhere, anything?
Read my post quoted earlier and leave a comment to tell me where I am wrong

Reply to  Ferdinand Engelbeen
February 25, 2023 3:14 am

For N2 and O2 molecules to excite a CO2 molecule they have to have higher energy. Due to energy sharing through collision, all neighboring molecules in the atmosphere have the same temperature. The energy within the lower troposphere is constantly being exchanged through collisions and IR exchange, but due to IR opacity, most heat has to rise before it can be emitted to space.

The temperature of the molecule affects the chances of emission. That’s why warmer bodies emit more radiation than colder ones. Emitting from a colder layer reduces emissions.

Reply to  Javier Vinós
February 25, 2023 4:35 am

Javier,

I do disagree:

Measured temperature is only the result of the average vibration energy of all molecules at a certain point in the atmosphere. Individual N2 and O2 molecules can have (much) higher and (much) lower vibration levels (individual “temperature”)…

That warmer bodies emit more radiation than colder bodies is the case for solid and liquid materials. where adjacent molecules can form dipoles and therefore can radiate in many wavelengths according to the Planck curve.
That is not the case for gases, where molecules most of the time are apart of each other and only GHGs can receive and emit IR waves in very specific wavelengths.

It is possible that the temperature vibration of an individual CO2 molecule affects its relaxation period (but have not seen anything in that direction), but the emitted photon does have exactly the same energy at the same wavelength, no matter if that CO2 molecule is at ground level at 293 K or high in the stratosphere at 233 K…

Editor
Reply to  Ferdinand Engelbeen
February 25, 2023 6:04 am

Ferdinand,
Wijngaarden and Happer disagree with you.

“Radiation transfer in the cloud-free atmosphere is controlled by the temperature”

[2103.16465] Relative Potency of Greenhouse Molecules (arxiv.org)

“The cross section can depend strongly on altitude because temperature and pressure are functions of altitude. Temperature controls the distribution of the molecules between translational, rotational and vibrational states. Pressure, together with temperature, determines the width of the molecular resonance lines.”

Reply to  Andy May
February 25, 2023 6:28 am

Thanks Andy, I think that has to do with saturation of the main emission/absorption bands, while the emissions/absorptions still increase at the side bands (thus at a slightly different wavelength). I will check it with William Happer what that really means when the pressure / temperature of the atmosphere decreases with height…

Reply to  Andy May
February 25, 2023 8:09 am

According to this graph of CO2 emissivity at low pressure and low temperature CO2 has zero emissions.

4505BA63-D451-499F-AFCA-3D9D42912EF6.jpeg
Reply to  Ferdinand Engelbeen
February 25, 2023 9:40 am

I agree with the point being made by Ferdinand that energy transfer between CO2, N2 and O2 is a two way street.

Temperature is a measure of kinetic energy. When the sun heats the earth’s surface, N2 and O2 absorb that heat via collisions with the surface and increase their kinetic energy. That causes air volume and thus buoyancy to increase, causing observed convection. If N2 and O2 do not pass their acquired energy to a GHG with less kinetic energy, then they can never lose that energy. They can cool adiabatically, but if those molecules start to sink they warm adiabatically and rise. Thats why air parcels are observed to “bob” up in down in the atmosphere. If N2 and O2 do not pass their kinetic energy to a GHG that can then radiate that heat away there would like be runaway heating

Stephen Wilde
Reply to  Jim Steele
February 25, 2023 11:22 am

They only bob up and down locally.
The overall pattern is within the Polar, Ferrel and Hadley cells in the Troposphere and the Brewer Dobson circulation in the Stratosphere.
There is a constant well defined convective overturning circulation that adjusts as necessary to neutralise all imbalances so as to keep the upward prssure gradient force balanced with the downward force of gravity.

Reply to  Stephen Wilde
February 25, 2023 12:13 pm

I think you missed my point that energy transfer between CO2, N2 and O2 is a two way street.

Phil.
Reply to  Jim Steele
February 26, 2023 8:32 pm

But predominantly in one direction, CO2 to N2 and O2. An IR excited CO2 molecule will collide over a million times with surrounding molecules during the average emission lifetime. A small fraction of N2/O2 molecules have sufficient energy to excite the CO2 energy level (Maxwell Boltzmann), however to excite that particular vibration they’d have to strike the CO2 molecule from a particular direction (very rare event).

Reply to  Jim Steele
February 25, 2023 5:02 pm

One of the best simplest explanations yet. Conduction finally gets mentioned.

Reply to  Jim Steele
February 25, 2023 5:05 pm

 If N2 and O2 do not pass their acquired energy to a GHG with less kinetic energy, then they can never lose that energy.

I believe that is not correct. Everything is capable of emitting IR radiation and everything does. A sphere of pure nitrogen at 300 K in space would emit IR and cool down. It would not keep its temperature forever. GHGs are just good absorbers and emitters of IR radiation. Apparently, N2 and O2 could have a bigger GHG effect than methane because they are so abundant.

Höpfner, M., Milz, M., Buehler, S., Orphal, J. and Stiller, G., 2012. The natural greenhouse effect of atmospheric oxygen (O2) and nitrogen (N2)Geophysical research letters39(10).

Reply to  Javier Vinós
February 25, 2023 5:15 pm

Javier Vinós February 25, 2023 5:05 pm

I believe that is not correct. Everything is capable of emitting IR radiation and everything does. 

While this is widely believed, it is not true of the monatomic gases (helium, neon, argon, krypton, xenon, and radon).

In addition, most diatomic gases are generally non-emitting, although under some conditions (collision-induced absorption) they can do so. From your link:

We have found that on global average under clear-sky conditions the OLR is reduced due to O2 by 0.11 Wm−2 and due to N2 by 0.17 Wm−2. 

GHGs absorb ~ 160 W/m2 of the upwelling surface radiation. Of that, the total effect of O2 + N2 is under 1/3 of a W/m2, about 0.2% of the total. In other words, pretty much lost in the noise.

Regards,

w.

real bob boder
Reply to  Jim Steele
February 26, 2023 7:25 am

A co2 molecule emitting a photon “cools”, another more exited molecule hitting it would “warm” it. Doesn’t seem complicated.

Phil.
Reply to  real bob boder
February 26, 2023 8:36 pm

But the colliding molecule would have to hit it with exactly the right energy from exactly the right direction to make the CO2 bonds vibrate in exactly the right frequency, not that easy.

Reply to  Ferdinand Engelbeen
February 25, 2023 10:47 am

Individual N2 and O2 molecules can have (much) higher and (much) lower vibration levels (individual “temperature”)

Sure but given the amazingly short time between collisions in the lower troposphere and the amazingly high number of collisions, the temperature is a good statistic estimation, and much higher or much lower vibration molecules are very rare.

We know that is true because as a parcel of air moves in the atmosphere its potential temperature is a conserved physical characteristic. So yes, I think I can say “all neighboring molecules in the atmosphere have the same temperature” and there is only a small error in what I say which does not affect the conclusion.

Pierrehumbert explains the same as I say in the short video (6 min) Steven Mosher shared above. A lower temperature means less emission.
https://youtu.be/slPMD5i5Phg

Reply to  Javier Vinós
February 25, 2023 3:04 pm

Javier Vinós February 25, 2023 10:47 am

Individual N2 and O2 molecules can have (much) higher and (much) lower vibration levels (individual “temperature”)

Sure but given the amazingly short time between collisions in the lower troposphere and the amazingly high number of collisions, the temperature is a good statistic estimation, and much higher or much lower vibration molecules are very rare.

Not really. At say 300K the average velocity is ~400 m/sec. But velocities of half (200 m/sec) and twice (800 m/sec) the average are about half as likely as 400 m/sec … see the Maxwell-Boltzmann distribution.

My best to you,

w.

Phil.
Reply to  Willis Eschenbach
February 26, 2023 8:42 pm

Here’s a plot of the M-B distribution Willis is referring to.
maxwelldistr1.gif

February 25, 2023 12:53 am

If the height of emission Ze rises from let’s say 10.000 km to 10.150 Km, then the surface of the sphere emitting (proportional to r^3) increases by (10.15/10.00)^3 = (1.015)^3 = 1.0456, an almost 5 percent increase in emission surface. Was this taken into account in the calculations? The absorption increase by doubling of CO2 from 400 to 800 ppm would only be maybe ca. 2%, so that could compensate it.

Reply to  Eric Vieira
February 25, 2023 9:01 am

Eric, I fear your math is wrong because you’ve left out the radius of the earth, which is ~ 6,400 km.

So your two spheres are actually 6,410 km and 6410.15 km in diameter. As you correctly point out, surface area goes up by the cube of the radius.

This gives (6410.15/6410)^3 = 1.00007, a 0.007% increase in area.

As a result, that’s generally ignored in calculations because it’s well down in the noise.

Best regards, and don’t worry, I’ve made many math mistakes worse than that one …

w.

Richard Greene
February 25, 2023 1:02 am

“The truth is that after adjusting to a change in GHG levels, the planet still returns all the energy it receives from the Sun.”

If that was true, the average temperature would never increase or decline. But the average temperature is ALWAYS increasing or declining. So that statement must be false. If I misinterpreted the statement, you ought to write more clearly.

Ian_e
Reply to  Richard Greene
February 25, 2023 1:59 am

Yep: the statement assumes that we are always at a steady state. Obviously wrong.

Reply to  Ian_e
February 25, 2023 2:29 am

I wouldn’t assume such a thing even if steady-state assumptions are useful for calculations. Ask a chemist.

Reply to  Richard Greene
February 25, 2023 2:27 am

Reaching a radiative equilibrium is a moving target that the Earth always aspires to but never really accomplishes. However, it is so close to it that the temperature of the Earth barely changes over a decade, a mere 0.15ºC perhaps. That, in radiative terms, means the Earth’s energy imbalance is tiny. Saying that the planet returns all the energy it receives from the Sun is just a small imprecision.

One of the reasons why the Earth could never be in radiative equilibrium is that its distance to the Sun is constantly changing, and so is the energy it receives.

Richard Greene
Reply to  Javier Vinós
February 25, 2023 3:28 am

” that the Earth always aspires to but never really accomplishes. However, it is so close to it that the temperature of the Earth barely changes over a decade, a mere 0.15ºC”

You are rounding 0.15 C. to zero and I am not.

Beta Blocker
Reply to  Richard Greene
February 25, 2023 6:34 am

Javier: ” that the Earth always aspires to but never really accomplishes. However, it is so close to it that the temperature of the Earth barely changes over a decade, a mere 0.15ºC”

Richard: You are rounding 0.15 C. to zero and I am not.”

If the earth’s global mean temperature continues to rise at a mean rate over time of 0.15C per decade for the next two centuries (twenty decades) — from whatever combination of causes, natural and man-made — then the earth’s GMT in the year 2223 will be roughly 3C higher than it is today.

For someone like me who is more interested in the public policy debate over climate change than in the physical mechanisms of climate change, watching the debate over the physics of GHG-driven warming is like watching a karate match from the bleachers where the participants on the gym floor beat each other silly, but without any one of them ever gaining anything resembling a clear victory over their opponents.

Richard Greene
Reply to  Beta Blocker
February 25, 2023 10:12 am

Scientists say another +0.15 degrees C. and the world will end. The big TIPPING POINT.
News at 11pm.

The +50% rise of CO2 since 1850, roughly estimated, was accompanied by about a +1 degree C. global warming since 1850, also roughly estimated.

CO2 obviously could not have caused all that warming since there were small CO2 emissions before the 1950s.

And CO2 emissions, faster than ever from 2015 to 2023, caused no warming at all (UAH data).

How those basic facts and rough estimates lead to a panic over CO2 emissions is hard to believe.

I personally worry about more important things: A meteorite hitting my home while I am sleeping, and invasions of aliens from Mexico.

This comment is serious, not satire.

Reply to  Beta Blocker
February 25, 2023 10:52 am

If the earth’s global mean temperature continues to rise at a mean rate over time…

You don’t know what is going to happen in the future. Assuming it will continue to rise at that rate for two centuries is very likely to constitute a mistake.

You think the goal of scientific exchanges is victory. You are also wrong about that. The goal is to increase knowledge, so everybody can come up as a victor, at least those with an open mind.

Reply to  Javier Vinós
February 25, 2023 3:08 pm

What Javier said.

w.

real bob boder
Reply to  Willis Eschenbach
February 26, 2023 7:40 am

The goal should be thus, however it’s been polluted by the politics of collectivism which is only concerned with winning.

nobodysknowledge
February 25, 2023 2:24 am

I think it is a disappointing ignorance in much of the discussion on OLR.
The first analysis came out over 20 years ago, and saw the role of cloud cover (ERBE data). And it has been repeated the last 20 years CERES data). Martin Wild has been repeating the importance of cloud cover (global brightening) from about 1983 over the last 20 years.
How can it be possible to attribute the change in IR from top of atmosphere to CO2 with the knowledge we have had for a long time. The part of greenhouse gases plays a minor role, perhaps about 1/3 of the warming (CERES), and then about 1/3 of OLR change.
Let us listen to what was said in 2004:
On the decadal increase in the tropical mean outgoing longwave
radiation for the period 1984–2000.
D. Hatzidimitriou, I. Vardavas, K. G. Pavlakis, N. Hatzianastassiou, C. Matsoukas and E. Drakakis.
“Conclusions. To summarize, our model calculations, which are based on ISCCP-D2 cloud climatologies, and temperature and humidity profile information from NCEP/NCAR reanalysis show that there has been an increase of OLR at TOA of 1.9±0.2 Wm−2 /decade between 1984–2000. This decrease is lower than the decrease displayed by the ERBE S-10N (WFOV NF edition 2) non-scanner OLR time-series, or by the results of Wielicki et al. (2002a, b). Analysis of the interannual and long-term variability of the various parameters determining the OLR at TOA, showed that the most important contribution to the observed trend comes from a decrease in high-level cloud cover over the period 1984–2000, followed by an apparent drying of the upper troposphere and a decrease of low-level cloudiness. Opposite but small trends are introduced by a decrease in low-level cloud top pressure, an apparent cooling of the lower stratosphere (at the 50 mbar level) and a small decadal increase in mid-level cloud cover.”

nobodysknowledge
February 25, 2023 2:51 am

And then we have Steven Dewitte and Nicolas Clerbaux, 2018.
Decadal Changes of Earth’s Outgoing Longwave Radiation
“The Earth Radiation Budget (ERB) at the top of the atmosphere quantifies how the earth gains energy from the sun and loses energy to space. Its monitoring is of fundamental importance for understanding ongoing climate change. In this paper, decadal changes of the Outgoing Longwave
Radiation (OLR) as measured by the Clouds and Earth’s Radiant Energy System from 2000 to 2018, the Earth Radiation Budget Experiment from 1985 to 1998, and the High-resolution Infrared Radiation Sounder from 1985 to 2018 are analysed. The OLR has been rising since 1985, and correlates well with the rising global temperature.”
“The spatial analysis of the regional OLR change from 1985–2000 to 2001–2017 shows a ‘clear-sky effect’ mixed a ‘tropical cloud’ effect.
Concerning the clear sky effect, we see regional changes of the OLR which are correlated with surface temperature changes. In the Arctic—where the strongest temperature increase occurs—we also see a strong increase in the OLR. In general, in the Northern Hemisphere—where the surface temperature increase is stronger than in the Southern Hemisphere—we also see an OLR increase.
Concerning the tropical cloud effect, we see regional patterns in the changes of the OLR, which are suggesting a relative strengthening of La Niña conditions compared to El Niño conditions. These changes imply societally important regional changes in precipitation. The relative La Niña strengthening can also be seen in the ‘cumulative MEI index’ that we have introduced.”

Reply to  nobodysknowledge
February 25, 2023 10:07 am

I agree with the conclusions that “The regional patterns of the observed OLR change from 1985–2000 to 2001–2017 show a warming pattern in the Northern Hemisphere in particular in the Arctic, as well as tropical cloudiness changes related to a strengthening of La Niña.” The small radiative imbalance is due to increasing energy being stored in the oceans caused by a more La Nina-like ocean.

February 25, 2023 5:22 am

will the defendant please rise.
foreman, has the jury reached a verdict?
we have your honor.
on the charge of committing catastrophic climate change, we the jury find the defendant…not guilty.

i here by order the intimidate release of mr. co2 from custody.
you are free to go.
this court is adjourned.

get  out of jail free.JPG
jshotsky
February 25, 2023 6:40 am

Radiation is not ‘reemitted’. When a molecule absorbs radiation (a photon) it is gone. The energy level of the molecule is increased by one photon. And radiative molecules don’t wait to absorb a photon before they emit one. If you could actually see radiative particles, such as a CO2 molecule, it would look like a microscopic flashlight, except the radiation goes in all directions – it is a ‘flash’. Everything that is above absolute zero K radiates, including the so-called non-radiative gases. That is why you can make argon and nitrogen lasers, as well as Co2 lasers. It is all a matter of energizing those molecules to the point at which they radiate freely. Ever see lightning? That isn’t just Co2, it is ALL of the ‘air’ molecules through which the lightning passes. The electricity ionizes the gases which heats them to the point of emitting visible light.

Next, there is one Co2 molecule for every 2500 ‘air’ molecules. That Co2 molecule doesn’t ‘heat’ those air molecules, (heat is the wrong term, but it’s what people understand.) All air molecules are in constant collisions with each other, and in a collision, any of three things can happen: They can ‘bounce’ off each other, they can ‘glance’ off each other or they can transfer energy from one to the other. It ONLY passes energy from the more energetic (warmer) molecule to the less energetic (cooler) molecule, regardless of whether it is an inert molecule or a radiative one. But one Co2 molecule is not going to ‘heat’ 2500 non-radiative gas molecules, THEY are going to control the ‘temperature’ of that Co2 molecule.

And for good measure, Co2 is not ‘well mixed’ as is always claimed. First of all, it is heavier than ‘air’ and will sink to the floor of a self-contained room. It is used to extinguish fires. It ‘hugs’ the ground, and it kills people in nature if there is enough of it. See Limnic Eruption.

Not to mention that, other than aircraft, all Co2 is emitted at the earth’s surface and is also absorbed at the earth’s surface. What magical ‘force’ could ‘well mix’ this mysterious Co2 molecule? None. And of course, all atmosphere is densest at the surface, meaning that is where the greatest concentration of all gas molecules exists, including Co2, of course.

What’s with this ‘doubling of Co2’ anyway? 95% of all emitted Co2 is natural. What mysterious thing would make EARTH double its Co2 output? Humans emit only 5% of the total, annually. If we stopped ENTIRELY, earth would not notice.

Lastly, all gas molecules follow the ideal gas laws. Laws, not hypotheses. The temperature of any gas is a function of its volume and pressure. Period. Co2 has no special ‘gas law’ that pertains only to Co2. A container of Co2 next to a container of nitrogen in the same room will be the same temperature. If you don’t believe that, take a non-contact thermometer to a gas supply company and take the temperatures of all the tanks of various gases. They will all be the same temperature, after filling and stabilizing.

Co2 doesn’t CONTROL anything. And wasting trillions of dollars, a high percentage of all of earth’s wealth, attempting to control Co2, won’t change anything. It CAN’T. Nor is it ‘responsible’ for earth’s temperature being above the so-called ‘black body’ temperature. It has nothing whatsoever to do with earth’s temperature. (But I can explain it.)

Reply to  jshotsky
February 25, 2023 10:59 am

I agree with the first two paragraphs but disagree with the third. CO2 vertical profiles show it is quite well mixed up to the stratopause, unlike the other two main GHGs, H2Ov and O3.

Reply to  jshotsky
February 25, 2023 5:17 pm

Nice post. The specific heat and mass of CO2 preclude it from being the control knob.

Reply to  jshotsky
February 25, 2023 10:50 pm

”Lastly, all gas molecules follow the ideal gas laws. Laws, not hypotheses. The temperature of any gas is a function of its volume and pressure. Period. Co2 has no special ‘gas law’ that pertains only to Co2. A container of Co2 next to a container of nitrogen in the same room will be the same temperature. ”

Yes agreed. Question from a layman. ….. 1. You have a given, uncontained amount of co2. 2. You heat it. 3. It expands. 4. It’s volume is increased but it’s concentration is not. Pressure remains the same.
The question… Is it the concentration or the volume which is said impede this OLR?
If it is the concentration, how the hell does that work? In this instance, concentration is the distance between molecules not the number of them. We are always told it is the concentration of co2 which is the problem.
If it is the increased volume (or number of molecules) which ”traps” the ”heat”, how does the increased volume affect emission to space? Does that not also increase?
Sorry if this is a stupid question.

jshotsky
Reply to  Mike
February 26, 2023 5:21 am

Questions are not stupid, they indicate that someone has interest in something they do not understand.
Co2 does not trap heat. It isn’t even heat, it is energy and it is transient. There is no trapping of anything. Radiation is photons, which are units of energy with no mass. Any molecule that can accept a photon can just as easily emit one. And that is exactly what happens, unless energy is transferred via collision first.

bdgwx
February 25, 2023 6:43 am

In this context “trap” means ΔEout < ΔEin resulting in ΔE > 0. It’s not unlike saying the door on the oven in your kitchen “traps heat” when it is closed resulting in the inside being warmer than it would be otherwise. Given its definition “trap” is appropriate terminology.

Reply to  bdgwx
February 25, 2023 11:01 am

That’s a matter of opinion. I just don’t consider that augmenting the kinetic energy of a molecule traps anything.

bdgwx
Reply to  Javier Vinós
February 25, 2023 11:16 am

Fill in the blank. When you close the door on the over in your kitchen it _____ heat causing the inside to be warmer than it would be otherwise.

Reply to  bdgwx
February 26, 2023 10:29 am

You are full of crap. If the oven is a constant source heating heating an enclosure, it will not get hotter than the source. Short the thermostat and the heating elements will only get to a certain temperature. The gases inside, will heat to that temperature and become in equilibrium with it. If it is considered a leakproof container, the pressure inside will rise as the temperature rises, until equilibrium. You’ve never had any real science education have you? Something like calculus based upper class physics, chemistry, or engineering. Back radiation actually heating a hot source to an even higher temperature is a joke!

You want to refute accepted physics, show the math for how two objects at equilibrium heated each other to a higher temperature than the hottest one.

bdgwx
Reply to  Jim Gorman
February 27, 2023 9:48 am

I stand by what I said. When you close the door on the oven in your kitchen its interior temperature will increase. You can call that crap and challenge the 1LOT, SB Law, Planck’s Law, etc. all you want. It doesn’t make it any less true.

Reply to  bdgwx
February 27, 2023 10:55 am

You are resorting to argumentative fallacies now. Specifically, Appeal to Authority. And even then, you have shown nothing that proves your point.

Do us a favor and past your math that allows reflected energy to warm the source to a higher temperature.

bdgwx
Reply to  Jim Gorman
February 27, 2023 1:11 pm

JG said: “You are resorting to argumentative fallacies now. Specifically, Appeal to Authority.”

Nope. I’m appealing to the 1st law of thermodynamics.

JG said: “Do us a favor and past your math that allows reflected energy to warm the source to a higher temperature.”

Sure. The 1LOT says ΔE = Σ[Ein_i, 1, i] – Σ[Eout_o, 1, o]. The heat capacity formula says ΔT = ΔE/mc when there is no phase change. We substitute the 1LOT into the heat capacity formula and we get ΔT = (Σ[Ein_i, 1, i] – Σ[Eout_o, 1, o])/mc. Then setting ΔEout_o = 0 for all o and ΔEin_i = 0 for all i except DWIR which we set to ΔEin_dwir > 0 we get ΔT = ΔEin_dwir/mc. And since ΔEin_dwir > 0 then it necessarily follows that ΔT > 0 because m and c do not significantly change and are finite.

I know…you don’t like the 1LOT and I suspect you don’t the ΔT ~ ΔE relationship either which means you’ll reject the fact that when I system traps energy (ΔE > 0) it warms. I get it. You don’t like the laws of physics. I’ve accepted that I’ll never be able to convince you. I do this for the lurkers.

You can have the last word. Deflect. Divert. Make up as many strawman arguments that I never advocated for. Have fun!

Reply to  bdgwx
February 27, 2023 1:24 pm

No, your math is sloppy. “m” can’t just be set to a constant. You have mass_surface, mass_ CO2, mass_N2, mass_O2, mass_H2O.

Then you have c_surface, c_CO2, … , c_H2O.

Then you have the E_convection, E_conduction, E_ IR.

Then you must deal with all the gradients involved between each pair of variables.

Why do you think modelers can’t get it right? If it was simple they would follow your lead with a simple equation. You haven’t even shown how entropy works in your little old equation. It applies too you know!

JCM
Reply to  Jim Gorman
February 27, 2023 2:35 pm

yes, once the system fires up, which it did a few billion years ago, turbulent flux becomes the dominant transport mode near the surface. This includes oceanic and atmospheric boundary layer mixing. Parameters include flux of mass and latent transport of heat. Reduced net transport of radiant equivalent energy aloft can arise from changing any of these parameters. Radiative flux mechanisms only dominate in very low pressure situations or in rudimentary 1D conceptual models.

bdgwx
February 25, 2023 7:11 am

Javier Vinos said: “And the test results can be evaluated for example with Derwitte and Clerbaux 2018:”

The GHE hypothesis is consistent with the [Dewitte & Clerbaux 2018]. Remember, the GHE hypothesis is not that ΔEout < 0. It is that ΔEout < ΔEin. We expect feedbacks like “longwave cloud thinning” to increase Ein as DC18 and others like [Loeb et al. 2021] observe.

Here is the OLR prediction from CMIP5 from 1985 to 2100. Notice that CMIP5 predicts OLR to increase.

comment image

I encourage you to download the CMIP5 data from the KNMI Climate Explorer and see for yourself. Note that TOA OLR is variable “rlut”.

Editor
Reply to  bdgwx
February 25, 2023 10:20 am

bdgwx,
CMIP5 predicts global surface warming, so it should predict an increase in OLR. Javier is just saying that if all the warming is due to greenhouse gases, OLR should not go up. I think he is correct in this.

This is just another, in a long line of fallacies built into the CMIP models because they are based on 60-year-old radiation logic that ignores the impact of energy transfer from one compartment to another (e.g., ocean to atmosphere). Some of Earth’s surface warming has to be coming from somewhere else.

bdgwx
Reply to  Andy May
February 25, 2023 11:55 am

Andy May said: “Javier is just saying that if all the warming is due to greenhouse gases, OLR should not go up.”

Which implies a misunderstanding regarding how OLR is expected to behave under a GHG dominant warming scenario. Remember, the expectation since at least the late 1800’s [Arrhenius 1896] has been that feedbacks will reduce albedo and bump up ASR and thus OLR at the new balance point. [Cess et al. 1990] which is cited by Dewitte & Clerbaux 2019 discuss how modeling pre-CMIP handles the shortwave feedbacks including water-vapor and cloud thinning that bump ASR. [Donohoe et al. 2014] does a pretty good job of explaining the expectation that OLR will increase.

Editor
Reply to  bdgwx
February 25, 2023 1:37 pm

bdgwx,
You guys are hilarious. Every time you get caught in an obvious model error you come up with some ad hoc “feedback.” I was highly amused when AR6 introduced feedbacks to feedbacks to get around the CMIP6 model’s obvious problems with SSTs in the eastern Pacific.

Paraphrasing AR6.

Well, you see, the feedbacks don’t stay the same. The feedbacks cause changes that change the feedbacks. That means what happened in the past can’t be projected into the future, we must change all the feedbacks. Oh, and by the way, computing the ECS from historical records, well that doesn’t work because the feedbacks change. You have to trust our models, because we can compute the changes in the feedbacks.

Now you create BS ASRs! Sorry, man, I’m not convinced.

Pardon me while I put on my hip waders and gasmask. Getting pretty deep and smelly around here.

For other readers, I’m referring to Donohoe’s 2014 paper.

bdgwx
Reply to  Andy May
February 25, 2023 2:06 pm

ASR = Absorbed Solar Radiation. There is only one of them.

Editor
Reply to  bdgwx
February 25, 2023 6:12 pm

Sorry, BS ASR

bdgwx
Reply to  Andy May
February 26, 2023 6:05 am

Absorded Solar Radiation is BS?

Editor
Reply to  bdgwx
February 28, 2023 4:49 am

This is the BS (from Donohoe):

However, climate models forced with CO2 reveal that global energy accumulation is, instead, primarily caused by an increase in absorbed solar radiation (ASR). This study resolves this apparent paradox. The solution is in the climate feedbacks that increase ASR with warming—the moistening of the atmosphere and the reduction of snow and sea ice cover. Observations and model simulations suggest that even though global warming is set into motion by greenhouse gases that reduce OLR, it is ultimately sustained by the climate feedbacks that enhance ASR.”

His imagined, and untestable, feedback to increase ASR.

bdgwx
Reply to  Andy May
February 28, 2023 7:11 am

That is a goal post move. The goal post set in the article is that the GHE hypothesizes a decrease in OLR. That’s not correct. The GHE actually hypothesizes an increase. Donohoe et al. explain the hypothesis. It doesn’t matter if you think the hypothesis is BS or not. It’s still the hypothesis.

BTW…an observation of an increase in OLR with a positive EEI necessarily means ASR is increasing too. That is the 1LOT. And as Javier Vinos’ points out in the article OLR is observed to increase so clearly the hypothesis is testable.

Reply to  bdgwx
February 25, 2023 4:37 pm

[Donohoe et al. 2014] does a pretty good job of explaining the expectation that OLR will increase.

Not really. They show OLR increasing after 80 years of increasing CO2 by 1% every year. What they say is that the warming comes from increased ASR, not decreased OLR. This means the warming is not coming from the GHE but from feedbacks responding to warming. All hypothetical, of course. I don’t see how something hypothetical can settle a scientific matter.

bdgwx
Reply to  Javier Vinós
February 26, 2023 11:17 am

That is not what they are saying. They are saying that additional warming comes from the albedo change and that the albedo change is catalyzed by the planetary energy imbalance caused by a pulse of CO2. No CO2 pulse no albedo change and no warming. And this albedo feedback was predicted at least 120 years ago so it’s not a new concept.

Reply to  bdgwx
February 27, 2023 2:15 am

Except we are not sure the albedo feedback is there.

The interannual variability of global reflected flux is 0.2 W m−2 which is only 1.4% of the annual cycle of this flux and 0.2% of the total global mean reflected flux.

Stephens, G.L., O’Brien, D., Webster, P.J., Pilewski, P., Kato, S. and Li, J.L., 2015. The albedo of Earth. Reviews of geophysics53(1), pp.141-163.

The trend of our ageing corrected CERES RSR relative to the ERA5 RSR is −0.03 ± 0.03 (1 σ) W/m2/dec

Dewitte, S., Clerbaux, N. and Cornelis, J., 2019. Decadal changes of the reflected solar radiation and the earth energy imbalance. Remote Sensing11(6), p.663.

bdgwx
Reply to  Javier Vinós
February 27, 2023 9:45 am

The observation of increasing OLR and the 1st law of thermodynamics says it is there. That’s the smoking gun. But there are attribution studies like [Loeb et al. 2021] that provide corroborating evidence as well.

Reply to  bdgwx
February 25, 2023 11:25 am

Feedbacks are not part of the CO2 greenhouse effect but due to the warming. You can fit anything you want in the feedbacks, as they can’t be measured. You can’t trust what models say about clouds because models don’t understand clouds. Cloud microphysics is not understood.

The GHE is not a hypothesis. It is a well-supported theory. The unsupported hypothesis is that changes in the GHE are responsible for nearly all recent warming.

That graph there means models don’t consider the warming is coming from a reduction in OLR, as the GHE says. Since the Sun is not increasing its emissions we must conclude models say the warming is coming from a reduction in albedo. Don’t you think? That is an interesting conclusion that is not publicized in the least.

bdgwx
Reply to  Javier Vinós
February 25, 2023 2:28 pm

I think part of confusion is that you are focused on the response of an idealized no-feedback instant pulse of GHGs. In the real world the GHE plays out with feedbacks over a long period of time with a drawn out pulse of GHGs. [Donohoe et al. 2014] describes this pretty well. Figure 1A is the idealized no-feedback instant pulse response. 1C is like 1A except feedbacks are considered. 1D is like 1C except the pulse is drawn out over several years.

Anyway, my point isn’t dependent on the details of how the GHE plays out in the real world. My point is that climate science theory says that we should expect an increase in OLR in the real world. And that finding an observation that shows OLR is increasing does not falsify that theory. If anything it provides corroborating evidence in support of it.

Reply to  bdgwx
February 25, 2023 4:28 pm

I think the confusion is coming from considering the feedbacks as part of the GHE. The GHE theory does not say anything about feedbacks. Feedbacks are the planet’s adjustment to a change in temperature by any means, greenhouse gas mediated or not.

Feedbacks are essentially unknown. By definition, they cannot be measured because they are the effect of a variable on another variable.

GHE theory says we should not expect an OLR increase coming from the GHE. There is not a single climate science theory. Some models say we should expect an OLR increase from feedbacks and some models say we shouldn’t.

What Donohoe et al. 2014 say is:

under the Special Report on Emission Scenarios A1B emissions scenario, wherein increasing radiative forcing is driven principally by increasing GHG concentrations, OLR changes little over the 21st century and global energy accumulation is caused nearly entirely by enhanced ASR—seemingly at odds with the canonical view of global warming by reduced LW emission to space

Doesn’t say much about increasing OLR, does it?

So that’s the problem. What you call climate science theory is just a computer game about unknown feedbacks that admit anything. I don’t call that science because it is not subject to the scientific method; I don’t call it a theory because there is no theory behind the feedback response to GHE-induced warming. You have chosen to believe in that. I have chosen not to believe in anything that can’t be demonstrated.

Editor
Reply to  Javier Vinós
February 25, 2023 6:17 pm

Well said Javier. A theory must make predictions that later come true. The GHG “theory” has not done this.

bdgwx
Reply to  Andy May
February 26, 2023 6:04 am

CMIP5 predicted OLR would increase. OLR increased.

real bob boder
Reply to  bdgwx
February 26, 2023 7:50 am

Because temperature would increase.

bdgwx
Reply to  real bob boder
February 26, 2023 11:13 am

Because the GHE played a significant role in that temperature increase. But, that is irrelevant here because my point is that climate scientists made a testable prediction that turned out to be right contrary the thesis of the article.

Reply to  bdgwx
February 26, 2023 11:33 am

The greenhouse effect predicts a decrease in OLR. Some climate models, not all, predict an increase in OLR from feedbacks, not from GHE. Problem is that is indistinguishable from warming from any other cause. The GHE prediction has turned not to be correct, and it was a prediction that would have distinguished warming coming from the increase in CO2 from other causes.

bdgwx
Reply to  Javier Vinós
February 26, 2023 12:41 pm

It is an expectation that the GHE causes warming. It is an expectation that warming causes an increase in ASR and OLR. Therefore it is an expectation that the GHE causes an increase in ASR and OLR. You can’t say the GHE prediction is incorrect because of an observation that is consistent with the expectation.

If you’re wanting a fingerprint observation that closest you are going to get is the TLT minus TLS trend simultaneous with the TLT trend. If want to falsify the GHE the best way is to show that either the TLT trend or the TLT-TLS trend are not positive over long periods of time and after controlling for volcanic activity when GHG concentrations are increasing.

Reply to  bdgwx
February 27, 2023 2:06 am

I have no problem with the GHE, I think it is a pretty solid theory. I have a problem with the CO2 hypothesis that says 2 times the warming produced by the direct CO2 increased GHE is produced by feedbacks that nobody can measure. It requires faith in a human abstraction (climate models), and as a scientist, I am very short on faith in scientific matters.

So every time we find a contradiction with the solid GHE theory, like the increase in OLR, the answer is “yes, but the feedbacks… because models…” That is not an acceptable answer unless we have lost contact with reality and living within a computer model.

The Earth is warming, the GHE theory says part of the warming should come from the increase in CO2, but we really don’t know where the rest (most of the warming) is coming from because we don’t have the evidence to say it. Where is the increase in water vapor? If there is an increase in water vapor, where is the increase in clouds? Where is the change in albedo? Why is there an increase in solar shortwave absorption?

You believe we are going to get the answers by asking the models. That requires faith.

Reply to  Javier Vinós
February 27, 2023 7:17 am

“You believe we are going to get the answers by asking the models. That requires faith.”

faith in a religious dogma, not in science.

Exactly like: have faith – the earth is flat.

bdgwx
Reply to  Javier Vinós
February 27, 2023 9:28 am

JV said: “Where is the increase in water vapor?”

[Allan et al. 2022] [Dressler & Yang 2008] [Soden et al. 2002] [Soden et al. 2005] [Borger et al. 2022] [Wang et al. 2016] [Santer et al. 2007]

JV said: “If there is an increase in water vapor, where is the increase in clouds?”

Who says clouds should increase?

BTW…I don’t think the cloud topic is as straight forward as your question implies. If clouds by night increase more than clouds by day decreases then you’ll have more clouds overall, but ASR and thus OLR will still increase. There are similar considerations with the height and microphysical composition as well.

JV said: “Where is the change in albedo? Why is there an increase in solar shortwave absorption?”

The change in albedo is the ASR increase. ASR can increase due to increased water vapor, decreased snow/ice extent, cloud pattern changes, land use changes, etc.

Reply to  bdgwx
February 27, 2023 7:23 am

‘It is an expectation that the GHE causes warming. It is an expectation that warming causes an increase in ASR and OLR. Therefore it is an expectation that the GHE causes an increase in ASR and OLR.’

Your first statement is incomplete, because it fails to note that GHE causes warming because it decreases OLR. Your second statement is therefore misleading because it implicitly includes GHE, which causes warming because it decreases OLR. Your third statement is therefore a logical fallacy.

As my late high school football coach once said, “Bad snap, bad hold, bad kick – the whole thing stinks”.

bdgwx
Reply to  Frank from NoVA
February 27, 2023 9:42 am

Frank from NoVA said: “Your first statement is incomplete, because it fails to note that GHE causes warming because it decreases OLR.”

No. I’m sorry. That is not correct. The GHE only decreases OLR under an instant pulse of GHGs scenario and only at the moment of the pulse. OLR begins increasing immediately. And after only 20 years it will even exceed the point at where it started. Under a 1%/yr pulse scenario OLR decreases for about 20 years and then begins increasing. And after about 80 years it will exceed the point at where it started. [Donohoe et al. 2014].

Frank from NoVA said: “ Your second statement is therefore misleading because it implicitly includes GHE, which causes warming because it decreases OLR.”

Of course it includes the GHE. The GHE causes warming. And again, the GHE does not cause a decrease and only a decrease in OLR.

Frank from NoVA said: “Your third statement is therefore a logical fallacy.”

I stand by my statement. The confusion here is that there is a misunderstanding regarding how the GHE works. And I don’t mean some hypothetical idealization of it. I’m talking about the real GHE and how it plays out on Earth including albedo changing feedbacks.

Reply to  bdgwx
February 27, 2023 10:30 am

‘The GHE only decreases OLR under an instant pulse of GHGs scenario and only at the moment of the pulse.’

Sorry, man. A while back I asked Nick Stokes for a reference to any long dated GCM runs that might have investigated the long-term effects of CO2 doubling. Here’s the SI from Paynter et al (2018). Take a look at Figure S2, panels C&D. Does that look like OLR is increasing?

https://agupubs.onlinelibrary.wiley.com/action/downloadSupplement?doi=10.1002%2F2017JD027885&file=jgrd54423-sup-0001-data_si.pdf

bdgwx
Reply to  Frank from NoVA
February 27, 2023 12:34 pm

Frank from NoVA said: “Take a look at Figure S2, panels C&D”

Figure S2 does not have a C or D so I’m assuming you meant figure S3. C is LWCLR which is the longwave clear sky effect and D is the LWCRE which is the longwave cloud radiative effect.

Frank from NoVA said: “Does that look like OLR is increasing?”

Yes.

Let me explain the graph. The concept being discussed in the publication is based N = F – αT where N is the net downward radiation flux, T is the surface air temperature and α is the feedback parameter. When N is positive ASR > OLR. N is being plotted in figure S2 for the 4x experiment. What you are seeing in figure S3 is the component breakdown of N depicted in S2.

Consider only CM3 for simplicity. At T = 0 N = 6 W/m2. At the moment of the pulse we have a 6 W/m2 imbalance. Of that N = 6 W/m2 imbalance SWCLR accounts for about 0 W/m2 and LWCLR accounts for about 6 W/m2. In other words SWCLR remained unchanged and LWCLR declined by 6 W/m2. Remember, positive N values are downward where N = F – αT and F = SW – LW so we reverse the sign on LW components. Now notice that as T increases (which you can also think of as a progression of time) the LWCLR declines which means the upward push at TOA is increasing. At about 3.5 K of warming we cross the threshold where LWCLR is higher than when it started.

Your own source [Paynter et al. 2018] confirms what Nick and I are saying. That is it is an expectation that GHG pulses and the GHE that OLR will increase.

Reply to  bdgwx
February 27, 2023 10:33 pm

‘Figure S2 does not have a C or D so I’m assuming you meant figure S3.’

My bad. Actually I meant Figure 2 in the main paper. Per the paper CM3 surface temperature increases by 4.84K at T=4700. And per the charts, LW and SW are decreasing and increasing with temperature, respectively.

bdgwx
Reply to  Frank from NoVA
February 28, 2023 7:28 am

Figure 2 in the publication is the same as figure S3 in the supplemental materials except that it is for the 2X experiment.

Remember, LWCLR is the graph is a component of N. N is the planetary energy imbalance N = ASR – OLR. When you see LWCLR declining in the graph that means N is declining and thus OLR is increasing.

Reply to  bdgwx
February 28, 2023 5:12 pm

‘Figure 2 in the publication is the same as figure S3 in the supplemental materials except that it is for the 2X experiment.’

Yes, let’s stick with the 2x ‘experiment’, please. Again we’re discussing GFDL’s model CM3 that doubles CO2 from 280 ppm to 560 ppm at a rate of ~1% / annum for a period of 100 years and then runs out for another 4,700 years until N stabilizes at ~ 0.00 W/m^2, yielding an ECS of 4.84K.

Paynter et al don’t provide a lot of detail re. their forcings, but I think we can glean what we need from the paper’s graphs:

Starting with Figure 1B, N (the net downward TOA radiative flux) starts at ~1.5 W/m^2 at Time = 0 (after 100 years of increasing CO2) and then decreases to ~0 over the next 4,700 years. Similarly, Figure 1C shows the evolution of N with Temperature, T, starting at ~1.5 W/m^2 at T ~ 2.5 K and then decreasing to ~0 W/m^2 at T 4.84.

I estimate the CM3 LW and SW components of N from Figure 2 A-D as follows (all numbers in W/m^2):

SW_CLR: 2.00 ==> 3.25 (+1.25)
SW_CRE: 1.00 ==> 3.25 (+2.25)

Total change for SW = 3.50

LW_CLR: -1.00 ==> -5.75 (-4.75)
LW_CRE: -0.50 ==> -0.75 (-0.25)

Total change for LW = -5.00

Total change for N = -1.50

So here is a question for you: If, per your sign convention, OLR (LW) is increasing by 5.00 W/m^2 and ASR is increasing by 3.50 W/m^2, why is T increasing by 4.8 K – 2.5 K= 2.3 K over the 4,700 year modeled period?

I think the answer is that you’ve missed the implications that N is defined as the ‘net downward TOA radiative flux’ with respect to both LW and SW radiation. Specifically, since green house gases emit LW radiation in all directions, a decrease in LW down from TOA also means a decrease in LW up from TOA, hence your OLR is actually decreasing. Note, this does not apply to SW because all atmospheric gases are transparent to SW.

PS – One other question for you. Given that Paynter et al’s ECS estimate of 4.84K implies an increase of about 27 W/m^2 LW from the surface, where do you suppose they come up with the additional 18.5 W/m^2 (27 – (5.0 +3.5)) needed to ‘balance’?

bdgwx
Reply to  Frank from NoVA
February 28, 2023 6:40 pm

Frank said: “N is defined as the ‘net downward TOA radiative flux’ with respect to both LW and SW radiation.”

Right. Positive N means means a net flow inward to the planet.

To get a positive N you can either increase ASR (SW) or decrease OLR (LW).

CM3 says the N = +2.5 W/m2 initial imbalance is composed of -1 W/m2 of SW and +3.5 W/m2 of LW. In other words SW decreased by 1 W/m2 and LW decreased by 3.5 W/m2. That is a net of +2.5 W/m2 of imbalance.

Remember, N = SW – LW. So when ΔLW < 0 then ΔN > 0.

Frank said: “Specifically, since green house gases emit LW radiation in all directions, a decrease in LW down from TOA also means a decrease in LW up from TOA,”

The only LW down at TOA is solar. A decrease in LW down occurs only when solar output decreases. Solar output only changes by like maybe 0.25%. That means LW down only changes by like maybe 0.01 W/m2. Of course in this experiment solar output is held constant so its moot.

Frank said: “hence your OLR is actually decreasing.”

No. Remember LWCLR and LWCRE are expressed as contributions to N where N = ASR – OLR. They are NOT contributions to OLR.

Frank said: “Note, this does not apply to SW because all atmospheric gases are transparent to SW.”

The surface absorbs about 47% of incoming solar radiation. The atmosphere absorbs about 23% of it. The remaining 30% is reflected.

Frank said: “ One other question for you. Given that Paynter et al’s ECS estimate of 4.84K implies an increase of about 27 W/m^2 LW from the surface, where do you suppose they come up with the additional 18.5 W/m^2 (27 – (5.0 +3.5)) needed to ‘balance’?”

I’m not sure where you got the 5.0 and 3.5 figures. I see 6.5 W/m2 increase in ASR and OLR at the end of the CM3 experiment. Anyway, the Planck surface radiative response is balanced by the GHE radiative response. From 255 K to 288 K the GHE is about (395 W/m2 – 240 W/m2) = 155 W/m2. That is a response of 155 W/m2 / 33 K = 4.7 W/m2.K. With an extra 6.5 W/m2 of ASR in the experiment we only need to account for (27 – 6.5) = 20.5 W/m2 of extra GHE which would be a response of 20.5 W/m2 / 4.8 K = 4.3 W/m2.K and on par with the 4.7 W/m2.K from the previous 33 K.

Reply to  bdgwx
March 1, 2023 1:16 pm

‘CM3 says the N = +2.5 W/m2 initial imbalance is composed of -1 W/m2 of SW and +3.5 W/m2 of LW. In other words SW decreased by 1 W/m2 and LW decreased by 3.5 W/m2. That is a net of +2.5 W/m2 of imbalance.’

So, here you acknowledge that the initial imbalance of 2.5 W/m^2 was due primarily to a decrease in OLR. Doesn’t this mean that you also agree with Vinos that OLR initially declines with the introduction of additional CO2? In other words, for an increase in surface temperature, we would need to see a decrease in OLR to confirm that the warming resulted from enhanced green house effect? Note, upon further consideration, I would also agree that to re-establish planetary ‘equilibrium’, OLR must eventually rise to the point where it equals ASR and reduces N to zero.

‘From 255 K to 288 K the GHE is about (395 W/m2 – 240 W/m2) = 155 W/m2. That is a response of 155 W/m2 / 33 K = 4.7 W/m2.K. With an extra 6.5 W/m2 of ASR in the experiment we only need to account for (27 – 6.5) = 20.5 W/m2 of extra GHE which would be a response of 20.5 W/m2 / 4.8 K = 4.3 W/m2.K and on par with the 4.7 W/m2.K from the previous 33 K.’

Nice arithmetic. I’ve seen estimates from 3.7 W/m^2 for 2xCO2 alone, to 8.0 W/m^2 for 2xCO2 +water vapor feedback + ‘other’ feedbacks, so 20.5 W/m^2 seems like a very heavy lift. Also, isn’t the impact of increased CO2 on forcing supposed to be logarithmic?

JCM
Reply to  Frank from NoVA
March 1, 2023 2:32 pm

one additional triviality, as a result of SB, when considering the virtual greenhouse effect with virtual effective radiating temperature of 255K using blackbody assumption – Is that for each additional K more power is required.

For example,

for 255K to 257K requires 7.6 W m-2

for 288K to 290K requires 11.4 W m-2

a 3.7 W m-2 virtual no adjustment response at 288K would result in a 0.7K virtual effect.

a 3.7 Wm-2 virtual no adjustment response at 255K would result in a 1K virtual effect.

bdgwx
Reply to  Frank from NoVA
March 1, 2023 2:33 pm

Frank said: “So, here you acknowledge that the initial imbalance of 2.5 W/m^2 was due primarily to a decrease in OLR. Doesn’t this mean that you also agree with Vinos that OLR initially declines with the introduction of additional CO2?”

Of course I acknowledge it. I was the one trying to explain it Javier Vinos. I even linked to the Donohoe et al. 2014 publication showing how it works. That is…OLR drops quickly and then begins rising as the GHE plays out.

Frank said: “In other words, for an increase in surface temperature, we would need to see a decrease in OLR to confirm that the warming resulted from enhanced green house effect?”

No. In fact, observing a continual decrease would not only falsify the positive feedback hypothesis that has been around since the 1800’s it would mean there is a negative feedback playing out which would have all kinds of bizarre implications.

Frank said: “Nice arithmetic. I’ve seen estimates from 3.7 W/m^2 for 2xCO2 alone, to 8.0 W/m^2 for 2xCO2 +water vapor feedback + ‘other’ feedbacks, so 20.5 W/m^2 seems like a very heavy lift.”

The canonical 3.7 W/m2 figure is for 2xCO2 is the radiative force. The 20.5 W/m2 figure is for the radiative response. The force and the response and different concepts. Do not conflate the two.

Frank said: “Also, isn’t the impact of increased CO2 on forcing supposed to be logarithmic?”

Yes. Myhre et al. 1998 found that the logarithmic relationship holds at least to 1000 ppm. The first 100 ppm from 300 ppm is 1.5 W/m2. The next is 1.2 W/m2. The next is 1.0 W/m2 and so on.

Reply to  bdgwx
March 2, 2023 9:20 am

Glad to see you’re still checking in on me…

‘Of course I acknowledge it [decrease in OLR]. I was the one trying to explain it Javier Vinos. I even linked to the Donohoe et al. 2014 publication showing how it works. That is…OLR drops quickly and then begins rising as the GHE plays out.’

JV’s point was that if we don’t first observe a drop in OLR, i.e., if it remains unchanged or increases, then any warming must be caused by something other than an increase in CO2 or other GHG. Do you concur?

‘In fact, observing a continual decrease would not only falsify the positive feedback hypothesis that has been around since the 1800’s it would mean there is a negative feedback playing out which would have all kinds of bizarre implications.’

I’m pretty certain that most folks would hold that the system will move towards re-establishing radiative balance between ASR and OLR. I would call that a negative feedback. Where do you see positive feedbacks prevailing, and why wouldn’t these be destabilizing?

‘The canonical 3.7 W/m2 figure is for 2xCO2 is the radiative force. The 20.5 W/m2 figure is for the radiative response. The force and the response and different concepts. Do not conflate the two.’

Yes, the canonical figure is 3.7 W/m^2, but I recall the IPCC had SSP scenarios where the worse case forcings, including the impact of an increase in albedo, were pushing 8.0 W/m^2. That would support an increase in surface temperature from 288.15K to 289.61K, which is 1.46K. You’ll need to explain to me how this forcing feeds back to a ~27 W/m^2 ‘response’ absent a change in orbital mechanics or continental position.

bdgwx
Reply to  Frank from NoVA
March 3, 2023 12:49 pm

Frank said: “JV’s point was that if we don’t first observe a drop in OLR, i.e., if it remains unchanged or increases, then any warming must be caused by something other than an increase in CO2 or other GHG. Do you concur?”

Like I told JV our opportunity to observe the initial OLR drop is long gone. We would have needed to have done it 100 or more years ago.

Frank said: “I’m pretty certain that most folks would hold that the system will move towards re-establishing radiative balance between ASR and OLR. I would call that a negative feedback. Where do you see positive feedbacks prevailing, and why wouldn’t these be destabilizing?”

I’m talking about the albedo feedback here; not the Planck feedback.

Frank said: “Yes, the canonical figure is 3.7 W/m^2, but I recall the IPCC had SSP scenarios where the worse case forcings, including the impact of an increase in albedo, were pushing 8.0 W/m^2. That would support an increase in surface temperature from 288.15K to 289.61K, which is 1.46K.”

It’s the same concept either way regardless if 3.7 or 8.0 W/m2 is cited. Those are radiative forces. The force is best described as what the planetary energy imbalance would be if the force were applied instantly. To rebalance OLR must increase to match ASR. But to get OLR to go up by the amount of the original force the surface must respond by an even larger amount since each incremental increase in temperature results in an incremental increase in W/m2 which is then partially returned back to the surface due to an incremental increase in DWIR from the GHE. This effect is the primary reason why the lapse rate increases under the GHE. In other words, the bottom gets warmer and the top gets colder. And when you add on top of all of this the albedo feedback which increases ASR the surface has to respond even more to close the ASR-OLR gap. I think the statement that best helps understand why the response is always larger than the force is that when the surface warms due to the uptake of excess energy it increases UWIR which in turn increases DWIR.

Reply to  bdgwx
March 3, 2023 4:27 pm

‘Like I told JV our opportunity to observe the initial OLR drop is long gone. We would have needed to have done it 100 or more years ago.’

Like prior to the Roaring Twenties? Somehow, that seems like an extraordinary example of special pleading given this:

Annual CO₂ emissions by world region

‘The force is best described as what the planetary energy imbalance would be if the force were applied instantly. To rebalance OLR must increase to match ASR. But to get OLR to go up by the amount of the original force the surface must respond by an even larger amount since each incremental increase in temperature results in an incremental increase in W/m2 which is then partially returned back to the surface due to an incremental increase in DWIR from the GHE. This effect is the primary reason why the lapse rate increases under the GHE. In other words, the bottom gets warmer and the top gets colder. And when you add on top of all of this the albedo feedback which increases ASR the surface has to respond even more to close the ASR-OLR gap. I think the statement that best helps understand why the response is always larger than the force is that when the surface warms due to the uptake of excess energy it increases UWIR which in turn increases DWIR.’

The IPCC itself defines the GHE (~159 W/m^2) as the difference between LW emitted by the surface and LW emitted out to space, so I find this force / response multiplier narrative to be rather amazing. Can you provide any independent sources as to how 3.7 W/m^2, or even 8 W /m^2, becomes 27 W/m^2? I mean, how have serious people from Linzden to Happer missed this?

Neo Conscious
February 25, 2023 7:19 am

So CO2 does not trap heat like a greenhouse at all, and thus the term GHE is just another complete fraud intended to mislead the public. The power of a greenhouse to transform temperatures is so dramatic it can be downright scary to a layman considering the horror of being trapped in one on a hot day.

If CO2 acts by increasing opacity to infrared radiation, it will also block more incoming IR radiation from the sun and thus decrease surface heating with a proportionate increase in lower tropospheric heating. That heat then escapes back out to space more quickly and thus may in fact unexpectedly result in lower surface temperatures, not higher.

bdgwx
Reply to  Neo Conscious
February 25, 2023 8:10 am

I get an outgoing 14-16 um band radiance of about 7 W/m2.sr.

I get an incoming 14-16 um band radiance of about 0.01 W/m2.sr.

CO2 blocks a lot more outgoing terrestrial radiation than incoming solar radiation.

real bob boder
Reply to  bdgwx
February 26, 2023 7:52 am

Increased surface temperatures increase convection, the exact opposite of how a green house works.

bdgwx
Reply to  real bob boder
February 26, 2023 11:27 am

Convection occurs within a greenhouse too. But that is irrelevant to the fact that CO2 does NOT block more IR radiation from the Sun.

Richard Greene
Reply to  Neo Conscious
February 25, 2023 10:17 am

How about the “greenhouse effect” of CO2 enrichment inside most greenhouses. used to accelerate plant growth, just like CO2 enrichment in the troposphere does for outdoor plants?

The wife got a part time job making flower arrangements in a greenhouse but lasted about two days because of the summer heat — she’s Greek and like heat, too. I practically had to carry her home after the second day.

Reply to  Neo Conscious
February 25, 2023 11:28 am

Yes. It means increased GHGs in the atmosphere will increase atmospheric temperature also by increasing solar radiation absorption. But that also means a reduction in the net flux from the surface, and so surface warming.

Reply to  Javier Vinós
February 25, 2023 3:20 pm

Absorption of sunlight in the atmosphere decreases surface temperature.

CO2 absorbs very little, CH4 absorbs more, and water vapor is responsible for about 70% of the atmospheric absorption of sunlight by GHGs.

Regards,

w.

February 25, 2023 7:22 am

Javier, I agree that something other than CO2 is warming the earth. I believe it is the ocean and the way solar heat is stored and released in the oceans as I described in my video Science of Solar Ponds Challenges the Climate Crisis
https://www.youtube.com/watch?v=wl3_YQ_Vufo&t=314s or transcript https://perhapsallnatural.blogspot.com/2023/01/the-science-of-solar-ponds-challenges.html

but in your comments you argue the oceans lag warming. So what is the “something else” that is warming the earth and why not oceans?

Editor
Reply to  Jim Steele
February 25, 2023 10:27 am

I think (and Javier may add to this) he is referring to changes in meridional transport in the troposphere and stratosphere. Variations in the meridional transport (from the tropics to the poles) cause climate change at all time scales. What causes these changes is not 100% known, but high latitude planetary waves (affecting the polar vortex), ENSO (varying the release of heat from the Pacific), and long-term changes in the Sun and Earth’s orbit (affecting ozone) play a significant role. See Javier’s book for the details.
(11) (PDF) Climate Change: Past, Present, and Future (researchgate.net)

Stephen Wilde
Reply to  Andy May
February 25, 2023 11:13 am

I suggest that it is known, or should be.
Variations in meridional transport anywhere within the vertical column of an atmosphere (or oceans) are simply the convective overturning system operating as it should to neutralise any imbalances that seek to alter the balance between the upward pressure gradient force and the downward force of gravity.
The only legitimate question is whether our emissions would produce a measurable response within the system compared to natural variability.
The evidence and logic would indicate not.

Reply to  Andy May
February 25, 2023 12:32 pm

Andy, here’s some data to consider while making your argument. First, the 22-year average of advection.

comment image

Positive values show areas that are exporting power, and negative values show areas that are importing power.

And next, the monthly changes in the amount advected from the tropics after the removal of seasonal variations.

comment image

Hope this helps,

w.

Editor
Reply to  Willis Eschenbach
February 25, 2023 1:43 pm

Thanks Willis, great illustrations. The winter months are actually the most important. Averages for DJF and the change over the years in DJF transport would be nice to see, it might make a good post for you. Same with JJA in the SH, although it is probably more boring.

The poles are huge net emitters of OLR in the wintertime, since nothing is coming in except through meridional transport.

Reply to  Jim Steele
February 25, 2023 11:45 am

Hi Jim, I did not want the article to be about my hypothesis but about the GHE and its meaning.

My hypothesis is not really that different from yours. It relies on differences in heat transport to high latitudes during winter. Still, mine is primarily an atmospheric theory because nearly all heat delivered to high latitudes during winter is carried there by the atmosphere. All meridional transport papers agree on that.

The winter high latitudes are like a different planet. They have almost no water in the atmosphere, the driest places on the planet. They are dominated by radiative cooling and often the GHE works in reverse because temperature inversions make the surface colder than the atmosphere. They are like a bleeding wound in the planet hemorrhaging heat into space. The amount of heat transported there is the main cause of climate change. It has always been. Even Milankovitch forcing works through changes in meridional transport.

The ocean is important because the atmosphere transports heat above the ocean basins and the ocean is the source of most heat to the atmosphere. The opening of the Arctic gateway is the ultimate cause of the Cenozoic descent into an ice age. The isolation of Antarctica actually stopped the process for a time by reducing the loss at the South polar cap. But the rising of the Himalayas and closening of the Panama gateway just helped and accelerated heat loss by enhancing meridional transport making the Arctic the source of most climate change on the planet.

Winter Arctic sea ice loss is reducing the rate of global warming and is being helped by polar vortex weakening due to low solar activity that allows the atmosphere to bring more heat to the Arctic. When the multidecadal oceanic oscillation kicks in driving more heat to the Arctic the whole climate science community is going to be very surprised.

That is my view of climate change.

Responding to your question, my opinion is that what has warmed the planet is a reduction in meridional transport due to the grand solar maximum plus the contribution from the increase in CO2.

higley7
February 25, 2023 7:24 am

No test is needed to exonerate ALL atmospheric gases. The climate models are sunlight 24/7 and the surface is always hotter than the atmosphere.

Very simply, the IR energy levels of the surface (15 degC) are full to a higher temperature than the IR energy levels of the gases in the atmosphere, particularly at the -17 deg C of the upper tropical. Thus, any IR directed downward by the atmosphere will be reflected (rejected) by the surface.

When you, at 98.6 deg F start in a room at 30 deg F, do you warm the room or does the room warm you? Clearly you warm the room. It’s that simple.

Because this junk science does not work, the climate nonscientists ignore this simple thermodynamic fact and pretend that the IR in the atmosphere warms the atmosphere directly. However, an object cannot warm itself. It does not matter as the IR energy is redistributed constantly and IR lost to space. The atmosphere is constantly trying to cool itself.

Yes, the surface is hotter than the atmosphere and will warm the atmosphere. But, again, the nonscientists ignore the fact that this is not solely a radiative situation.

85% of the energy carried away from the surface is from conduction and subsequent convection, producing a huge heat engine that actually ramps up when heated. The rising moist warm air then cools as it rises, by adiabatic cooling, the water vapor condenses, releasing latent heat and returning cooler back to earth while the heat as IR is lost to space.

No tests are required. It’s not rocket science, either, although such studies are much too complicated, particularly when you ignore the convection, global heat engine.

Reply to  higley7
February 25, 2023 9:39 am

higley7 February 25, 2023 7:24 am

No test is needed to exonerate ALL atmospheric gases. The climate models are sunlight 24/7 and the surface is always hotter than the atmosphere.

I’m sorry, but that’s simply not true in the slightest. The models simulate both day and night, and the surface is sometimes cooler than the atmosphere.

There are huge problems with the climate models, but those two are not among them. See my post “Meandering Through A Climate Muddle” for some of the problems.

w.

bdgwx
Reply to  higley7
February 25, 2023 12:09 pm

higley7 said: “Thus, any IR directed downward by the atmosphere will be reflected (rejected) by the surface.”

That’s not how radiation works though. Photons don’t have a “I was emitted by a source at temperature T” property. Bodies (like Earth’s surface) don’t absorb photons based on the temperature of the source that emitted them because they don’t know the temperature. Bodies (like Earth’s surface) absorb photons based on their frequency and only their frequency.

Reply to  bdgwx
February 25, 2023 2:59 pm

It is how radiation works. From Planck’s Theory of Heat Radiation:

“””””For example, if we let the rays emitted by the body fall back on it, say by suitable reflection, the body, while again absorbing these rays, will necessarily be at the same time emitting new rays, and this is the compensation required by the second principle.”””””

This is the only way equilibrium can be reached. Otherwise, the hot body gets hotter and hotter each time the “radiation + back radiation” is reflected back. It is an ever increasing series!

Lay it out in a series using even a 50% factor.

1 => 100 W
2 150 W
4 225 W
6 on and on

Even 1W will result in an infinite series if the hot body gets hotter from “back radiation” and equilibrium will never occur!

In terms of photons, if the hot body is radiating 10 photons at temp T, it keeps right on radiating 10 photons at temp T. If it receives 1, the net radiation between the two bodies is 9.

That is the only way two bodies can achieve equilibrium.

Again, from Planck;

“”””Since a quantity of energy emitted causes a decrease of the heat of the body, and a quantity of energy absorbed an increase of the heat of the body, it is evident that, when thermodynamic equilibrium exists, any two bodies or elements of bodies selected at random exchange by radiation equal amounts of heat with each other.”””””

bdgwx
Reply to  Jim Gorman
February 26, 2023 6:20 am

higley7 said: “Very simply, the IR energy levels of the surface (15 degC) are full to a higher temperature than the IR energy levels of the gases in the atmosphere, particularly at the -17 deg C of the upper tropical. Thus, any IR directed downward by the atmosphere will be reflected (rejected) by the surface.”

bdgwx said: “That’s not how radiation works though. Photons don’t have a “I was emitted by a source at temperature T” property.”

Jim Gorman said: “It is how radiation works.”

No it isn’t. Photons do NOT carry with them the temperature of the body that emitted them. I know…there’s no way I’m going to convince you of that. I can’t even convince you that the 1LOT and Stefan-Boltzmann Law work. I’m just pointing out for the lurkers that this insistence that a receiving body knows the temperature of the source body is absurd and that the receiving body does NOT reflect photons based on the temperature of the source.

Reply to  bdgwx
February 26, 2023 9:38 am

You didn’t read my post nor what Max Planck found through experiment did you.

You must believe that frequency is the only part of radiation that counts. It is not. Look at the Stefan-Boltzmann equation in more detail. The end result is POWER, not just frequency. Looking at it as “a photon” simply doesn’t include the power portion which controls how many photons are emitted or absorbed.

The net result is that a cold body CAN NOT heat a hot source. The source will continue to radiate at its temperature and area regardless of how much power a cold body can direct its way. The powers DO NOT ADD at the hot body. As I said, you could never achieve equilibrium if that occurred. Read Planck’s theory if you don’t believe me. That is compensation.

If you want to disagree, show the math of how a cold body causes the temperature of a hot body to rise. You’ll have to deal with entropy to do so, so get busy!

bdgwx
Reply to  Jim Gorman
February 26, 2023 11:25 am

Stop deflecting and diverting. Do you or do you not support highley7’s belief that body’s decide whether or not to absorb radiation based on the temperature of the body that emitted them? Yes or No?

Reply to  bdgwx
February 27, 2023 4:50 am

You didn’t even read what Jim Gorman posted did you?

What does the phrase “the compensation required by the second principle” from Planck mean?



bdgwx
Reply to  Tim Gorman
February 27, 2023 6:52 am

Stop deflecting and diverting. We are not discussing Planck’s Law and the fact that blackbodies emit more photons at higher temperatures. We are discussing the statement Very simply, the IR energy levels of the surface (15 degC) are full to a higher temperature than the IR energy levels of the gases in the atmosphere, particularly at the -17 deg C of the upper tropical. Thus, any IR directed downward by the atmosphere will be reflected (rejected) by the surface. which thermodynamics including but not limited to Kirchoff’s Law says is false, but which Jim defends.

Reply to  bdgwx
February 27, 2023 7:39 am

1) “energy levels” traditionally refer to electron levels. Those are immaterial to IR.

2) Vibrational and rotational movements of atoms/molecules are what is involved with IR.

Read this pdf. Vibrational Spectroscopy (Infrared, IR-Spect.) – Tanta

I am not saying that IR is reflected, although it can be, again read Planck. It can also be scattered.

You have shown your ignorance of Planck. His treatise does not only deal with “black body” radiation. Much of his theories deal with “monochromatic” radiation. That is, single frequency” radiation and not black body radiation. In addition he deals with entropy in a very detailed fashion.

Your dismissal of Planck’s experiments and mathematical derivations of radiative heat transfer is just plain ignorant. You need to reflect on what you might learn by studying his texts.

Lastly, you have failed to show how equilibrium is ever achieved without compensation as described by Planck. Until you can do that your ruminations are worthless.

Phil.
Reply to  Jim Gorman
February 28, 2023 6:30 am

“1) “energy levels” traditionally refer to electron levels. Those are immaterial to IR.
2) Vibrational and rotational movements of atoms/molecules are what is involved with IR.”

And anyone familiar with Physical Chemistry knows that there are electronic, vibrational and rotational energy levels. In the case of IR emission by CO2 we’re talking about vibrational levels and the rotational fine structure.

Reply to  bdgwx
February 27, 2023 5:55 am

Here is what you said:

“”””””Photons don’t have a “I was emitted by a source at temperature T” property. Bodies (like Earth’s surface) don’t absorb photons based on the temperature of the source that emitted them because they don’t know the temperature”””””

But, they do carry with them, a signature that says “I was emitted at xxx° temperature”. It is commonly known as wavelength!

Molecules and atoms do absorb IR at unique wavelengths, ala CO2. So, ultimately, yes bodies do “decide” in a sense, whether or not they will absorb heat emanating at a given temperature or not! Why do you find that controversial?

Planck discusses this using oscillatory springs that only vibrate at given frequencies. You really should study his treatise carefully and in depth. As far as I know his work on heat radiation and entropy have never been refuted, only extended.

bdgwx
Reply to  Jim Gorman
February 27, 2023 7:09 am

JG said: “Here is what you said:”

Yep. :That’s what I said and I stand by it.

JG said: “But, they do carry with them, a signature that says “I was emitted at xxx° temperature”. It is commonly known as wavelength!”

No they don’t. Planck’s Law does NOT say that photons carry the signature of the temperature of the emitting body. What Planck’s Law says is that for a blackbody it 1) emits more photons at all frequencies for higher temperatures and 2) that the frequency of peak emission is higher for higher temperatures.

I will repeat this again. Photons do NOT contain information about the temperature of the body that emitted them. The only information they contain is a frequency. And because they do NOT contain temperature information about the source that means receiving bodies cannot decide to absorb them based on that temperature.

JG said: ” So, ultimately, yes bodies do “decide” in a sense, whether or not they will absorb heat emanating at a given temperature or not!”

Are you deflecting and diverting here. We are not talking “absorbing heat”. We are talking about higley7’s statement Very simply, the IR energy levels of the surface (15 degC) are full to a higher temperature than the IR energy levels of the gases in the atmosphere, particularly at the -17 deg C of the upper tropical. Thus, any IR directed downward by the atmosphere will be reflected (rejected) by the surface.”

I will repeat this again. The surface does NOT reflect DWIR based on the temperature of the atmosphere.

And I’ll you again. Do you or do you not support highley7’s belief that body’s decide whether or not to absorb radiation based on the temperature of the body that emitted them? Yes or No?

Reply to  bdgwx
February 27, 2023 7:59 am

No they don’t. Planck’s Law does NOT say that photons carry the signature of the temperature of the emitting body.”

You are just plain wrong.

from wikpedia:
The energy and momentum of a photon depend only on its frequency  or inversely, its wavelength (λ):

E = ĥω where ĥ is the reduced Planck constant, ω is the frequency

Frequency is the inverse of wavelength, λ.

The frequency at which a photon is emitted is based on the temperature of the emitting body.

Wein’s Displacement Law: λ_peak = b/T
where T is the absolute temperature and b is Wien’s displacement constant.

You may get a distribution of λ around the peak but there *will* be a peak that is dependent on Temperature. Thus the receiving body *will* know the temperature at which the photon was emitted +/- whatever the distribution is.

bdgwx
Reply to  Tim Gorman
February 27, 2023 8:35 am

TG said: “You are just plain wrong.”

I gave you the chance to clarify your position. I now have no other choice but to accept that you reject the fact that 1) bodies do not decide whether to absorb photons based on the temperature of body that emitted them and 2) that Planck’s Law says bodies emit more photons at all frequencies with the peak emission occurring at higher frequencies as temperature of the increases. That’s what I’m saying and you are saying I’m wrong about that. Fine. I’ll just log this as yet another absurd argument along with the challenges of the 1LOT and SB Law that you and Jim are making and move along.

Reply to  bdgwx
February 27, 2023 8:59 am

In other words, you can’t refute anything, so you’ll take your ball and go home.

Your argument attempts to do away with IR spectroscopy. In your words, Planck says everything radiates like a black body, which also means everything absorbs like a black body.

Funny how CO2 absorbs/emits most at 15μm. I wonder what frequency and temperature that is.

Reply to  bdgwx
February 27, 2023 9:26 am

You can’t even quote what is being said correctly. Quit paraphrasing and actually address what is being said.

t 1) bodies do not decide whether to absorb photons based on the temperature of body that emitted them and”

No, that is *NOT* what compensation is all about. Go read Planck. Absorption of lower energy (i.e. lower temperature) photons just get absorbed and then emitted back again. They do not heat the body. In order to heat the body they would have to be at a higher energy level than the receiving body.

” that Planck’s Law says bodies emit more photons at all frequencies with the peak emission occurring at higher frequencies as temperature of the increases.”

So what? That does *NOT* say that lower frequency photons with less energy can heat a body emitting at a higher frequency!

Colder bodies simply can’t warm hotter bodies. It’s just that simple. If they could nothing would ever wind up in equilibrium. If they could then putting an ice cube next to your glass would warm the soda in the glass!

Wein’s Displacement Law should be a perfect clue as to what actually happens. But you refuse to even address it.

Take your ball and go home. It’s a waste of time trying to educate you.

Phil.
Reply to  Tim Gorman
February 27, 2023 12:04 pm

You should learn something about emission of light yourself before purporting to ‘educate someone’ about it. A blackbody at 300K will emit a wide range of photons varying in wavelength from ~3 to over 30 microns. It will emit 15 micron photons at ~6.6 W/m2/sr/µm. Reduce the temperature to 200K and it will emit 15 micron photons at 1.3 W/m2/sr/µm. A body absorbing 15 micron photons can’t distinguish between those emitted at 200 and 300K. ‘Lower energy photons’ are not ‘lower temperature photons’. Take some low energy photons (10 micron) and focus them on steel and you’ll melt the steel.

Reply to  Phil.
February 27, 2023 12:40 pm

Is a CO2 molecule a blackbody or does it emit at specific frequencies?

Look at this chart and tell me why CO2 is not shown transmitting or absorbing as a blackbody!

Show rme a reference that shows CO2 radiating as a blackbody!

You do realize if everything on earth transmitted as a blackbody, it would be hard to distinguish between substances.

Reply to  Jim Gorman
February 27, 2023 1:11 pm

Forgot the chart!

OIP.jpeg
Phil.
Reply to  Jim Gorman
February 27, 2023 2:50 pm

Well I was responding to a comment about a ‘body’ emitting. Your statement about CO2 radiating is even more misinformed. An excited CO2 molecule emits at 526 cm-1 regardless of the temperature of its location. However they all emit from a vibrational temperature of 757K which is hotter than most of the Earth’s surface.

Reply to  Phil.
February 27, 2023 2:42 pm

Wein’s Displacement Law says the spectral radiance of black body radiation per unit wavelength peaks at the wavelength λ_peak

λ_peak = b/T

What does T stand for? Lower energy photons just get turned around and re-emitted. They simply don’t increase the temperature of the hotter body. If they didn’t then you could never reach equilibrium and an ice cube next to your coffee cup would warm your coffee!

Phil.
Reply to  Tim Gorman
February 27, 2023 5:45 pm

You and your ‘twin’ need to make up your mind about whether you’re talking about black bodies or molecules! Lower energy photons do not get ‘turned around’. You don’t appear to understand Wein’s Law.

Reply to  Phil.
February 28, 2023 5:43 am

And you seem to think that an ice cube can warm the coffee in your cup while it’s sitting beside the cup.

Ain’t gonna happen.

Phil.
Reply to  Tim Gorman
February 28, 2023 12:48 pm

Quite, it wouldn’t happen. I’d take a bet on CO2 at 1000K though but I’m not sure what material the cup would need to be made from to be transparent to the IR.

Reply to  Phil.
February 28, 2023 8:00 am

Suffice it to, a CO2 molecule or a conglomeration of CO2 molecules are NOT a black body, and do not absorb nor emit like a black body.

https://www.tec-science.com/thermodynamics/temperature/plancks-law-of-blackbody-radiation/

This site has a good derivations of the different derivations.

If molecules don’t follow Wein’s law then all bets are off.

Wein’s law is:

T = 2897.8 μm K / λ_max or λ_max = 2897.8 μm K / T

Reply to  Tim Gorman
February 27, 2023 8:50 am

A photon is part and parcel of an EM wave and has a given frequency depending on the temperature.

Anybody that dismisses Planck also dismisses the experimental discovery of “quanta” or in other terms, photons.

Phil.
Reply to  Jim Gorman
February 28, 2023 6:19 am

No the frequency does not depend on the temperature of the emitter. The frequency of the photon emitted by an excited CO2 molecule is the same whether the CO2 is at 200K or 300K.

Reply to  Phil.
February 28, 2023 10:19 am

Then praytell, how do we know that CO2 is radiating at -80°C? Your explanation has no way to determine CO2’s actual temperature.

Phil.
Reply to  Jim Gorman
February 28, 2023 12:49 pm

CO2 is not radiating at -80ºC that’s a misunderstanding of Wien’s Law which refers to the position of the peak in the blackbody radiation curve and is irrelevant to CO2 emission. CO2 emission is from the first vibrationally excited level of the O-C-O bending mode at 667cm-1 above the ground state (with some rotational fine structure). That corresponds to a vibrational temperature of ~1000K.

Reply to  Phil.
February 28, 2023 1:16 pm

So this illustration is all bonkers? CO2 doesn’t REALLY radiate at ~215K?

R (1).jpeg
bdgwx
Reply to  Jim Gorman
February 28, 2023 1:44 pm

There is nothing wrong with the illustration. Yes. CO2 radiates at 215 K. And although CO2 does radiate at 215 K that’s not what that graph is saying. The dashed lines are the blackbody curves at 220 to 320 K in increments of 20 K . Notice that the peak spectral radiance shifts to higher wavenumbers at higher temperatures.

Phil.
Reply to  Jim Gorman
February 28, 2023 2:20 pm

The illustration isn’t bonkers! What it shows is that in equilibrium a gas can’t have a higher radiance than a blackbody at the same temperature. Consequently CO2 molecules that are radiating to space have to be high enough in the atmosphere to not be absorbed by the intervening CO2 molecules. This graph shows that those molecules are at a temperature of 215K (the ones in the sidebands where the absorption isn’t as efficient are at about 260K). In other words the CO2 emissions seen from space originate near the tropopause. The emissions still originate from CO2 molecules which are excited to a vibrational temperature of ~1000K.

Reply to  Phil.
March 1, 2023 8:28 am

Word salad!

How do you know the molecules are at 215K?

Will anything absorbing that energy see the radiation as if it came from an object at 215K or 1000K?

Phil.
Reply to  Jim Gorman
March 1, 2023 11:01 am

You know the CO2 molecules are at a temperature of 215K because of their radiance. If you reran the Modtran calculation with the sensor at 2km you’d see the CO2 radiance corresponding to ~300K.
It will see a photon of wavenumber 667cm-1 i.e. a vibrational temperature of ~1000K, regardless of the temperature of local atmosphere.

Reply to  Phil.
March 1, 2023 12:05 pm

You are dancing.

Here is what I said.

“””””I was emitted at xxx° temperature”. It is commonly known as wavelength!”””””

See that term wavelength I used? That is part and parcel of the EM wave. It is the signature that allows absorption.

The power contained in the EM wave is determined by the temperature of the molecule.

Give a reference that shows the photons within that wave can be at 1000°K

I learned that a photons energy is E = hc / λ. I don’t see an energy in that equation.

Phil.
Reply to  Jim Gorman
March 1, 2023 12:51 pm

About time you went and took a freshman Physical Chemistry course!
“Here is what I said.
“””””I was emitted at xxx° temperature”. It is commonly known as wavelength!”””””
See that term wavelength I used? That is part and parcel of the EM wave. It is the signature that allows absorption.”

And it is not determined by the molecule’s temperature. 

“Give a reference that shows the photons within that wave can be at 1000°K”

The photons emitted by an excited CO2 molecule are the result of the transition from the excited vibrational energy level to the ground state. The frequency of the transition is 667cm-1, in other words the molecule is excited by about 8kJ/mole above the ground state. The vibrational temperature of that level is 959K, that’s the temperature you’d have to raise CO2 to for that frequency to arise from the surrounding heat.

Phil.
Reply to  Jim Gorman
March 1, 2023 1:26 pm

I learned that a photons energy is E = hc / λ. I don’t see an energy in that equation.”

h=6.626×10-34 J.sec
Checkout what the SI unit of energy is!

Beta Blocker
February 25, 2023 8:14 am

Steven Mosher: “wrong wrong wrong ….. You cant critique the AGW argument until you demonstrate that you understand it.”

Exactly so.

No one here on this forum, nor on any other skeptic forum that I am aware of, has ever presented a clear explanation of the physical theories and theoretical physical mechanisms which the mainstream climate scientists claim are the cause of global warming.

Speaking as someone who looks at the climate change issue from a public policy viewpoint, the most important facet of the science debate over physics and physical mechanisms is the mainstream climate science explanation for why the basic 1C to 1.2C of CO2-induced warming from a doubling of CO2 concentration is amplified into a +3C warming, or a plus +4C warming, or whatever.

At its most simplistic level, the Held and Soden amplification theory is that the basic 1C to 1.2C of CO2-induced warming at the surface allows the atmosphere to hold more water vapor than it otherwise would, which then in turn induces a feedback mechanism which amplifies the original base level warming. 

The physics and the physical mechanisms being postulated are a good deal more complex than that. But for most of those reading this forum, the process of gaining an in-depth understanding of the postulated physics and physical mechanisms the mainstream climate scientists are pushing requires that we crawl before we can walk.

The best thing those of you who do have a thorough understanding of the physics and the physical processes mainstream climate science is now pushing is to start with the basics and to then move on from there in expanding our understanding of their scientific arguments.

Once that process is done, the critiques being made of mainstream climate science will carry a lot more weight with those who are inclined to be open-minded about the science.    

Richard Greene
Reply to  Beta Blocker
February 25, 2023 10:24 am

You had six paragraphs of text to “teach” AGW, and just focusing on CO2 emissions would have been fine, but you failed miserably.

AGW also includes air pollution, land use changes, Urban heat island effect, and faulty measurements and adjustments of the temperature data, either deliberate or unintentional.

gyan1
Reply to  Beta Blocker
February 25, 2023 10:58 am

The strong water vapor feedback assumed in climate models has never been observed in the real world.

Reply to  gyan1
February 25, 2023 12:13 pm

Exactly, the CO2 hypothesis of global warming remains unsupported by observations. It is amazing it is being used as an excuse to change many nations’ energy infrastructure under the motto “save the planet.” Are we out of our minds?

Richard Greene
Reply to  Javier Vinós
February 26, 2023 8:10 am

Exactly, the CO2 hypothesis of global warming remains unsupported by observations. 

Irrelevant statement: You implying we don’t know everything about climate change, so we know nothing. That is claptrap.

The exact causes of global warming can not be determined because there are too many possible causes of warming without complete detailed knowledge about any of them.

But it can be stated with high confidence that more CO2 in the atmosphere impedes Earth’s ability to cool itself. That means more CO2 is very likely one contributor of the 1975 to 2015 warming trend.

Reply to  Richard Greene
February 27, 2023 1:50 am

You implying…

I am not implying that. The hypothesis that feedbacks multiply the direct warming from CO2 by a factor of 3 (from 1 to 3 ºC per doubling) is only supported by climate models. Climate models are an abstraction of the human mind, so they do not constitute scientific evidence.

There’s really not much more to say. The hypothesis is based on faith in a human abstraction. The first duty of a scientist is to be skeptical, and there’s a lot to be skeptical of.

bdgwx
Reply to  Javier Vinós
February 27, 2023 12:47 pm

The hypothesis is not based on faith. It is based on the Clausius-Clapeyron relationship for water vapor, the albedo difference between snow/ice and land/water, and more speculatively the microphysics of cloud formation and evolution. The first two have been known since the 1800’s. The 3rd has only gain prominence in the last few decades.

Reply to  bdgwx
February 27, 2023 11:52 pm

It is based on the faith that the Clausius-Clapeyron relationship and a few other physical principles can explain the response of the entire climate system to a change in temperature. We don’t even know how many things we are missing. Evaporation is controlled mainly by wind speed, not the Clausius-Clapeyron relationship, and we know wind speed changes, but we don’t know why or in which direction should change with warming. And hundreds or thousands of other things not correctly represented in models.

The truth is we don’t know how much warming the increase in CO2 should have caused, and we have made little progress in answering that question in 45 years. Yet we are told that we must reduce our emissions by X% in Y years to avoid reaching +1.5ºC. If we don’t know how much warming the increase in CO2 causes how can we know that? It is obvious we are being lied to.

The entire thing is a house of cards based on the inadequate output of the human mind we call models.

Reply to  Javier Vinós
February 28, 2023 5:54 am

Nicely put!

Reply to  Beta Blocker
February 25, 2023 11:55 am

No one here on this forum, nor on any other skeptic forum that I am aware of, has ever presented a clear explanation of the physical theories and theoretical physical mechanisms which the mainstream climate scientists claim are the cause of global warming.

Then please do so. After over 40 years I would think someone somewhere must have been able to explain it so people can understand it. I’ve got over 50 books on climate science and have done extensive reading of the scientific bibliography. I am also a scientist with a Ph.D. in a scientific discipline. If I can’t understand it (and you think I don’t understand it), who can? Do you understand it?

Richard Greene
Reply to  Javier Vinós
February 26, 2023 8:13 am

Ph.D. sometimes stands for Piled High and Deep !

Here is all that everyone needs to know:

(1) The climate will get warmer, unless it gets colder

(2) Summers will be warm, and winters will be cold.

(3) Most correct answers to climate change questions are “we don’t know that yest”

($) No one will admit to (3)

viejecita
February 25, 2023 10:08 am

WoW ! Even I an old ignorant grandmother understands this.
Great. Thank You.

Reply to  viejecita
February 25, 2023 12:09 pm

Muchas gracias, viejecita. Poner la ciencia al alcance de los que la financian con sus impuestos es un deber.

viejecita
Reply to  Javier Vinós
February 26, 2023 3:46 am

Mil Gracias a Usted :
Veo que a otros,( a los que leo siempre, y aprecio, aunque frecuentemente sus escritos me rebasen ) , les parece simplista. Pero a las viejecitas curiosas, nos encanta poder comprender por lo menos algo. Aunque luego haya más facetas que tener en cuenta. Hay que estar muy preparado para admitir, como Judith Curry, que , de algunas cosas, una sólo sabe que tiene dudas.
Pues eso ; Que muchas Gracias.

Reply to  viejecita
February 26, 2023 9:14 am

Bien dicho, jovencita. Te llamo asi porque cuando ya estas aprendiendo, ya estas joven.

Como se dice, te deseo “Salud, y pesetas, y tiempo pa’ gozarlas …”

w.

Tom.1
February 25, 2023 3:04 pm

There is no proof, evidence, or scientific argument no matter how elegant or profound that is going to alter that state of the debate on global warming/climate change and what causes it (if there even is a debate at this point). The other side has already carried the day on this matter, so save your breath.

Reply to  Tom.1
February 25, 2023 4:03 pm

Getting the science right matters to me.

Tom.1
Reply to  Javier Vinós
February 25, 2023 8:14 pm

And it should matter to people of science and technology, but this is largely a political problem now, and I don’t think anything science has to offer can change that dynamic.

Editor
Reply to  Tom.1
February 26, 2023 5:35 am

Politicians never change. The only way to get change is to vote them out.

real bob boder
Reply to  Andy May
February 26, 2023 8:00 am

Lol, all sides buy into anything they can use to manipulate the ignorant.

Richard Greene
Reply to  Andy May
February 26, 2023 8:32 am

WHAT IF THE NEW POLITICIANS ARE WORSE?

Richard Greene
Reply to  Javier Vinós
February 26, 2023 8:31 am

Separating actual science from always wrong wild guess predictions of climate doom is my priority.

Richard Greene
Reply to  Tom.1
February 26, 2023 8:30 am

What debate?

JoeG
February 25, 2023 3:17 pm

Basic physics exonerates CO2. CO2 infrared absorption – Climate Auditor

Thus the only absorption and re-emission by atmospheric CO2 of any consequence must be the photons in the 15 micron band.

CO2 will poison the air before it causes climate issues that adversely affect us.

esalil
February 25, 2023 3:25 pm

A humble question: if the effective emission height is about 5 km what harm do the CO2 exhaustions of airplanes flying at 10 km?

Reply to  esalil
February 25, 2023 4:04 pm

Nothing, as far as I can tell. CO2 moves freely up through the stratosphere, much higher than planes.

esalil
Reply to  Javier Vinós
February 25, 2023 11:03 pm

but does it cool at 10 km?

Reply to  esalil
February 25, 2023 11:48 pm

Yes. Higher than the average height of emission any GHG acts as a coolant agent for the atmosphere, facilitating IR outgoing radiation.

JCM
February 25, 2023 6:02 pm

OLR is greatest during the nino, when atmospheric specific humidity is highest.

Planetary heat uptake is the highest during the nina, when surface temperature is lowest.

It all seems backwards.

In observation there is no such thing that the OLR diminishes and the surface warms.

In observation there is no such thing that the higher water vapor content increases heat uptake.

It’s all backwards….

OLR increases when the surface warms….

Heat uptake is least when water vapor is highest.

No amount of ad-hoc feedback adjustments to IR radiative reductionist theory will make any of it work.

It’s not unlike the ability using our eyes to observe the yellow tint of jaundice on the skin of a loved one….

a spectral observation by remote sensing. However, we would be beside ourselves if we limited the investigation of cause to spectral phenomena. The wrong color paints on the walls? Incorrect exposure to UV light?

Such a notion is just as ludicrous as limiting the investigation of climate drivers to radiative phenomena, due to our fascination with noticing symptoms by radiometer….

the physics is much more fascinating, engaging, and elegant than what the radiation enthusiasts propose, I assure you.

Editor
February 25, 2023 6:42 pm

Javier, thought you might enjoy this if you haven’t seen it.

Best to you and yours,

w.

Reply to  Willis Eschenbach
February 25, 2023 11:46 pm

Thank you Willis

Richard M
February 25, 2023 7:16 pm

If the warming is due to an increase in CO2 when the increase takes place and the altitude of emission increases

The altitude of CO2 average emissions is in the stratosphere. If the altitude increased the emissions would increase. I know the alarmists like to average all the gases together to bring the altitude down but that is wrong. Every gas needs to be looked at separately.

It doesn’t really matter though. The emission altitude of well mixed GHGs is fixed. It is determined by the gravitational force, the mass of the atmosphere and the available energy.

JCM
Reply to  Richard M
February 25, 2023 8:10 pm

Does anyone have evidence the optical depth is changing?

tau = -ln(Transmitted Flux / Surface LW Up)

I think, not really.

Richard M
Reply to  JCM
February 26, 2023 8:11 am

The fact no one is looking at it since Miskolczi showed it was flat tells us all we need to know.

JCM
Reply to  Richard M
February 26, 2023 9:58 am

Yes – the required constraints are still supported in CERES.

Atmospheric Greenhouse Gas factor in CERES EBAF Ed4.1 Version 3, March 2000 – Feb 2022:

Clear Sky values:

TOA LW UP 266.01
Surface LW UP 398.51 

G = 398.51 – 266.01 = 132.5
g = 132.5/398.51 = 0.33

Atmospheric greenhouse gas factor is stable and stationary. The 0.33 is not a coincidence, but required!

Furthermore, G = OLR / 2 = [H + LE] = surface net radiation is supported to within 0.5 W m-2 in CERES.

266.01 / 2 = G

Incredible stability, and a testament to the incredible work being done by the CERES team!!

It leaves only non gaseous and non radiative flux parameters to explain climate changes, no doubt.

as a PS Surface LW Up = 3 OLR / 2 is also supported to within 0.5 W m-2. These are non coincidental relations!! Pure geometry at work.

398.51 = 3/2 (266.01)

Richard M
Reply to  JCM
February 26, 2023 11:47 am

Thanks for showing the numbers. This should be required reading for all skeptics (and climate scientists).

I think one of the big problems skeptics have is they trying to account for warming with negative feedbacks to warming. The feedbacks occur immediately. There is no warming.

I refer to these as boundary layer feedback. The primary one is increased evaporation (driven by increased CO2 downwelling IR) which enhances the water cycle and counterintuitively, reduces the water vapor greenhouse effect.

“Global rainfall increases typically cause an overall reduction of specific humidity (q) and relative humidity (RH) in the upper tropospheric levels of the broader scale surrounding convection subsidence regions. This leads to a net enhancement of radiation energy to space over the rainy areas and over broad areas of the globe. ” Dr. William Gray

http://tropical.atmos.colostate.edu/Includes/Documents/Publications/gray2012.pdf

JCM
Reply to  Richard M
February 26, 2023 12:09 pm

thanks for the paper! looks good and I plan to check it out.

what’s for certain is that the greenhouse gas factor is not a free variable… totally stable on climate scales, and set unequivocally from first principles without reference to gaseous composition or radiative transfer codes. unbelievable!!

Schwarzschild 1906 – “Thus our considerations are neither complete nor compelling, but by explaining a simple idea in its simplest form, they may form the basis for further speculations”.

Beyond natural drivers, from my perspective there is possibility for humanity to meddle in the partitioning of surface LE + H. This is where things get interesting.

Climatology’s reductionism to radiation transfer and atmospheric optics is wrong wrong wrong. splitting hairs on trivialities, and missing the nature of things.

Richard M
Reply to  JCM
February 26, 2023 12:27 pm

Once again I agree. Ocean cycles with salinity changes and current variations can affect the latent energy. These can be driven by many factors including movement of the ITCZ as Jim Steele has been studying.

Still a long way to go before we understand our climate.

JCM
Reply to  Richard M
February 26, 2023 3:44 pm

I like the ideas in the work.

I would reorganize Figure 1, however “Annual Global Budget”

Net IR = 59 looks good. This is transmitted flux to OLR.

Rain looks fine = 85. Net Latent Flux to OLR. The consensing matter emitting full spectrum OLR into clear sky at altitude.

Sensible heat = 27 OK. But I would caution this does not contribute to the OLR. 

This non radiative flux H is that which is retained in the turbulent boundary layer below the cloud deck, due to the existence of the atmosphere.

We can derive the H using CERES data, once more.

The Greenhouse Gas Effect G = 132.5

The CERES observed Atmospheric Effect = LW up surface – OLR

Atmospheric Effect = 398.75 – 240.2 = 158.6

H = Atmospheric Effect – Greenhouse Gas Effect

H = 158.6 – 132.5 = 26.1

This value, in my opinion, is often confused with a radiative IR effect, sometimes termed ‘LW Cloud Radiative Effect’ by radiation enthusiasts.

It is also scooped into accounting schemes, using radiative equivalent atmospheric absorption concepts.

Untitled.png
February 26, 2023 3:05 am

Since GHGs cause the planet to emit from a higher altitude, due to making the atmosphere more opaque to IR radiation”

Surface warming shifts the entire lapse rate line to the right. Temperatures at all altitudes warm.

The radiative balance altitude increases.

The cause is warming. Warming is not the result.

The earth doesnt magically radiate from a cooler altitude just because of CO2.

February 26, 2023 3:05 am

And yes, ERBE has been telling us OLR increases with surface temp, thus the warming is NOT from retained energy .

vinceram
February 26, 2023 3:46 am

Planets use whatever physical/chemical means available in an attempt to to equlibrate surface temperatures.The surface would be significantly colder on the night-side, and much hotter on the day-side without IR active gases in the atmosphere, especially without water present. The surface temperature would heat up until extreme convective heat transfer conditions accelerated surface wind speeds that would whip-up giant clouds of dust in order to regulate earth’s temperature by the increased reflection and absorption of solar energy by the dust in the atmosphere, thus blocking out a good percentage of the solar energy at the surface.

DWM
February 26, 2023 7:09 am

All that is required for a change in the Earth’s OLR radiation is for the Albedo to change. CO2 we know warms the surface of the planet. How can we ignore the probability that the slight warming resulting from GHGs causes a slight change in the Albedo resulting in an increase in OLR radiation?

Reply to  DWM
February 26, 2023 8:28 am

You have to get the mechanism right. Truth is that CO2 causes more greening of the planet, both on land and sea.
more green = more black = more heat is trapped.
That is what is causing albedo to change….

https://breadonthewater.co.za/wp-content/uploads/2023/01/albedo1674915190404.webp

DWM
Reply to  Henry Pool
February 26, 2023 10:09 am

That would be another reason that CO2 could be blamed for the increase in OLW radiation, namely increased Sun’s radiation absorption at the surface requires increased OLW radiation to achieve energy balance.

I would have used the fact that warmer surface from GHE has less ice and snow thus more absorption of energy at the surface. More absorption at the surface requires more OLW radiation to achieve an energy balance.

Reply to  DWM
February 26, 2023 11:55 am

I invalidated ‘another reason’
See comment below.

February 26, 2023 7:32 am

I calculated that the net cooling effect of CO2 (by deflecting SW) is more or less equal to the warming effect (by trapping LW)
Summary of analysis CO2 spectrum NIST (1)

Scroll to the first 3 rows and look in columns K,L and M

An evaluation of the greenhouse effect by carbon dioxide | Bread on the water

Reply to  Henry Pool
February 26, 2023 9:08 am

Those who follow my work know that I’m a data guy. Here’s some important information regarding the ideas in the head post. I offer this without comment, I’ll leave that to Javier and to the vox pop.

comment image

w.

Reply to  Willis Eschenbach
February 26, 2023 12:00 pm

Ja. Ja. Like the cooling effect caused by the CO2 by deflecting SW does not exist.

Crisp
February 26, 2023 10:05 pm

Seim and Olsen (University of Oslo, 2021) did actual experiments (as opposed to theoretical blah blah blah) and found:(a) 100% (1,000,000 ppm) CO2 “heats” air to about the same temperature that non-greenhouse gases (N2, O2 [air], Ar) do; and (b) no significant temperature difference in containers with 0.04% vs. 100% CO2.
This confirms experiments carried out over 100 years ago by Prof John Wood (and repeated since) that greenhouses do NOT operate by trapping IRR via CO2 and converting it to LWR.

And there are good physical reasons for this. Didn’t anybody here do physical chemistry at university, particularly with respect to the ways and manner in which gases can absorb and transport energy? Hint: there are 4 ways, 3 of which are controlled by the laws of quantum mechanics, not statistical mechanics, and therefore have nothing to do with the Stefan Boltzmann equation. These 3 ways are a function of the wavelength of the input energy and IR absorption falls in that range. We should appreciate that the temperature of gases is a macroscopic measure of what is happening to energy levels at the molecular and sub-atomic scales, but importantly, temperature measures only some of that energy. Hint: it measures only 1 of these 4 means by which gases hold energy, and it is not the quantum ones.

Reply to  Crisp
February 27, 2023 3:10 pm

Crisp, let me refer you to the discussion/dissection of the Seim/Olsen experiment by Kevin Kilty here on WUWT. Seim/Olsen have a number of problems in their experimental setup.

Then you can read my discussion/dissection of the R. W. Wood (not “John Wood”) experiment here.

In my opinion, there are many problems with the “CO2 roolz the temperature” hypothesis. But those two experiments are not among them.

Best regards,

w.

February 27, 2023 6:31 am

Regarding “The truth is that after adjusting to a change in GHG levels, the planet still returns all the energy it receives from the Sun”. This is after equilibrium between energy income and energy outgo gets restored by the planet having a surface temperature increase.

Richard M
Reply to  donklipstein
February 27, 2023 7:09 am

There is no increase in the surface temperature from increases in CO2 because we get reductions in high altitude water vapor. In fact, without increased absorption in the wings of the CO2 absorption bands we would get cooling.

Reply to  donklipstein
February 27, 2023 8:11 am

This is after equilibrium between energy income and energy outgo gets restored by the planet having a surface temperature increase.”

Because the higher surface temp implies a higher radiation level, right?

Reply to  donklipstein
February 27, 2023 3:27 pm

donklipstein February 27, 2023 6:31 am

Regarding “The truth is that after adjusting to a change in GHG levels, the planet still returns all the energy it receives from the Sun”. This is after equilibrium between energy income and energy outgo gets restored by the planet having a surface temperature increase.

Don, I fear you’re repeating the common error of the climate alarmists, which is the assumption that TOA radiation imbalance can only be rebalanced by increasing surface temperature.

In fact, there are a number of ways for the balance to be restored:

  • Changes in surface albedo
  • Changes in cloud albedo
  • Changes in the advection of thermal energy at a variety of atmospheric levels
  • Changes in the advection of ocean heat content via El Nino etc.
  • Changes in atmospheric absorption of sunlight by clouds
  • Changes in atmospheric absorption of upwelling surface LW by clouds
  • Changes in the rate of surface latent and sensible energy losses to the atmosphere
  • Changes in the percentage of atmospheric LW radiation going upwards vs downwards

So no, there’s no law saying surface temperature must change to restore TOA balance.

w.

JCM
Reply to  Willis Eschenbach
February 27, 2023 4:47 pm

good stuff

Reply to  Willis Eschenbach
March 4, 2023 10:57 am

Changes in surface albedo: Currently reinforcing temperature increase.

Changes in cloud albedo: Currently reinforcing temperature increase (low confidence).

Changes of advection: Mostly a small negative feedback, caused by a positive feedback that is disproportionately happening in/around the Arctic.

Advection changes in the ocean such as with El Nino: ENSO indices have flat long tertm trend.

Changes in atmospheric absorption of sunlight by clouds: Repeat of an above point.

Changes in atmospheric absorption of upwelling surface LW by clouds: At least half of the outgoing radiation from clouds is downward. Increase of cloud absorption of radiation from the surface is at most a negative feedback that only partially negates a surface temperature increase from GHG-change-caused radiation imbalance.

Changes in the rate of surface latent and sensible energy losses to the atmosphere: Sounds to me like change of latent vs. sensible is a positive feedback to change of global average surface temperature while being a negative feedback to global peak surface (where there is water) temperature. Also, John Christy demonstrated that most climate models overconsider this factor, because increased latent heat transport is the main cause of the tropical upper tropospheric warming hotspot that has been happening very greatly less than predicted.

Changes in the percentage of atmospheric LW radiation going upwards vs downwards: Increase of greenhouse gases increases the downward percentage and decreases the upward percentage.

Richard M
February 27, 2023 6:41 am

This is how the GHE actually works. An increase in CO2 means an increase in the height of emission. Since the temperature of emission must remain the same, the temperature from the surface to the new height of emission must increase. The increase is small but significant. 

This view is wrong for CO2. Looking at emission height is just plain bad science. Don’t get sucked in. What you really want to know is what changes in the total energy emitted when you change the concentration of a gas. Averaging out the emission heights for all GHGs is like averaging out the number of dollars and pennies you have and saying increases in the number of any denomination you have has the same effect.

For CO2 the average emission height is in the Stratosphere. If all else were held equal and you raised that height, you should emit more energy and thus cool the planet. However, even that is the wrong answer for other reasons.

GHGs both absorb and emit energy. As a result opacity looks entirely different than we are used to with stuff than only absorbs energy (like mist). In addition, the atmosphere exists in a gravitational field which changes the concentration of these absorbers/emitters as you go higher. You can’t apply what might seem like common sense.

For well mixed GHGs the ability to absorb/emit stays balanced at all altitudes as you increase their concentrations. This means the flow of energy stays constant. The effective opacity stays the same.

It’s the combination of absorption and emittance that determines the final result. Absorption does increase slowing down the flow of energy. However, emittance also increases which means more energy is flowing. The two values cancel out and the energy flow per unit time remains constant.

For CO2 the only increase in energy from increasing the concentration comes from the fact that more surface energy is absorbed low in the Troposphere at the edges of the 15 mm frequency band.

DWM
Reply to  Richard M
February 27, 2023 8:30 am

If you calculate the Earth’s total thermal energy released to space for the case of no atmosphere and an albedo of 30% you will get 240 w/m2. If you now add an atmosphere of any concentration of CO2, water vapor, and other gasses the total thermal energy released to space will remain 240 w/m2.

Reply to  Richard M
February 27, 2023 9:11 am

While I agree with most of what you say I think it is even more complicated than this. You would be correct if there weren’t other molecules floating around. But what gets absorbed doesn’t always get emitted as radiation. Sometimes it get passed on as kinetic energy to other molecules (e.g. oxygen, nitrogen, water vapor, other CO2, etc). How that gets handled via conduction and convection also plays a part in the heat profile as well as the absorption and emitting of IR by oxygen and nitrogen. O2 and N2 don’t need to have the same IR efficiency as CO2 in order to have an impact since there is so much more O2 and N2 in the atmosphere.

Whether all that can be measured adequately, especially since it is a time function, leaves me with a lot of doubt.

/sarc/ perhaps we should focus on getting some O2 and N2 out of the atmosphere instead of CO2?

February 27, 2023 6:57 am

I just Googled for NASA statements of CERES trends, and saw that incoming absorbed radiation and outgoing longwave IR are both increasing, but incoming has been increasing more than outgoing, and the imbalance is in the direction of warming the surface and increasing. Income of absorbed radiation is expected to be increasing because of decreasing surface albedo.

MichaelMoon
February 27, 2023 9:20 pm

So, all this endless discussion about Convection is meaningless. The Earth orbits in a vacuum, heated by a star. The amount of energy in the atmosphere is determined by two things, energy in and energy out. Energy in seems relatively constant, although the amount of UV seems to vary a bit. Energy out is determined by how much radiation goes to Space, from the Land, mostly dirt and rock, from the Ocean liquid, from reflectance from Albedo, and from the Atmosphere. Albedo is not constant, mostly determined by clouds, can be measured by reflectance from the New Moon, this data is hard to find. It is called Earth-Shine, try to find the records.

The only thing CO2 changes is the opacity of the Atmosphere to outgoing radiation. CO2 absorbs in the 15-micron band. A bit more CO2 at the surface does not change much, as it already absorbs all it can in the first 10 meters from the surface. It rarely re-radiates down here, as collisions with Air absorb this energy as it has for the entire history of the Earth.

The absorbance band of CO2 is around 15 microns wavelength. This corresponds to a temperature of emittance and of absorption of -80 C. So, more CO2, high in the sky where it is much colder that at the Surface, does exactly what Javier says, raises the altitude at which the Atmosphere is freely able to radiate to Space, thus lowering the temperature at which the Atmosphere is freely able to radiate to Space, thus retaining some more energy equals Heat in the Atmosphere.

But, as Javier also says, no one knows how much, this cannot be calculated from First Principles.

I will not reply to Dr. Eschenback again, he does not understand the first word of this, never passed any classes in Thermo nor Heat Transfer, all he knows is what NASA and NOAA say…

Moon

DWM
Reply to  MichaelMoon
February 28, 2023 6:25 am

Speaking of energy in and energy out. The surface of the Earth radiates about 390 w/m2. Of that 240 w/m2 is radiated to space. At all times 390 w/m2 must be returned to the surface to maintain equilibrium. Estimates from NASA and others suggest the thermal radiation from the Sun is about 160 w/m2 so that leaves 230 w/m2 that must be returned to the surface from some other source.

Without CO2 and water vapor it would be difficult to find that level of energy returning to the surface because without GHGs most of the radiated energy would pass directly to space. So if GHGs can return that much energy why not a few more w/m2 further heating the surface?

JCM
Reply to  MichaelMoon
February 28, 2023 8:56 am

Energy out is determined by how much radiation goes to Space, from the Land, mostly dirt and rock, from the Ocean liquid, from reflectance from Albedo, and from the Atmosphere.

In a 1D conceptual model:

The Total OLR is the combination of Cloudy Sky OLR and Clear Sky OLR.

The Total OLR is the sum of transmitted flux from the active planetary surface and that emitted from atmosphere.

The active planetary surface observed from space includes the land “dirt and rock” and that from the condensed matter surfaces (ocean and cloud).

At any given time the active radiating planetary surface observed from space is the 2/3ds cloud fraction. The remaining 1/3 is the Earth surface land and ocean.

The gap between the Earth surface and the active radiating cloud surface, of which occupies 2/3s of total radiating surface at any given time, is completely dominated by non radiative flux K.

The amount of energy in the atmosphere is determined by two things, energy in and energy out.

The dominance of non-radiative process delivering power to the active radiating surface in atmospheric budgets does not displace the Ein and Eout paradigm.

The error is reducing the internal power dynamics to radiative process. These dynamics are known to meteorologists as weather.

Atmospheric Ein = +Solar +Oceanic Flux
Atmospheric Eout = -OLR – Oceanic Flux
Earth System Ein = Solar
Earth System Eout = -OLR

The weighted effective planetary radiating surface temperature can be thought of as:

1/3 289K + 2/3 273K (roughly 2km cloud radiating height).

Planetary effective radiative surface temperature = 279K

279K delivering 240 Wm-2 power density suggests a total system emissivity about 0.7.

The weighted effective planetary emissivity:

1/3 x Blackbody Surface + 2/3 Cloud Emissivity (~0.6).

Planetary effective emissivity ~0.7

JCM
Reply to  JCM
February 28, 2023 9:39 am

Further still – these ratios are absolutely everywhere in the earth system.

Take Schmidt’s diagram posted upthread by Javier

comment image

The surface budget components reflect the ratios:

170 = Net SW
16 + 100 = K
Net LW = 53

Surface Net SW *1/3 = Surface Net LW
Surface Net SW * 2/3 = K

This causes dissonance for people who wish to reduce the Earth system climate dynamics to radiative flux mechanisms within atmosphere, but in fact there is no net flux of LW within atmosphere whatsoever. The only LW flux is that which is emitted to space (more or less). This last statement often gets me into trouble here.

JCM
Reply to  JCM
February 28, 2023 10:04 am

If one wishes to get even more daring, the albedo can be derived as a simple reciprocal of emissivity for the opaque effective radiating surface (i.e. earth surface + cloud):

where the albedo = (1-0.7)

Phil.
Reply to  MichaelMoon
March 2, 2023 4:31 pm

The absorbance band of CO2 is around 15 microns wavelength. This corresponds to a temperature of emittance and of absorption of -80 C.”

It has absolutely nothing to do with -80ºC